You are on page 1of 191

Sebabnya Modul Ini Menitikberatkan Penggunaan Unit S.I.

Pada Setiap Langkah Penghitungan Dalam Fizik

* Sebagai ahli Fizik, penggunaan unit amat penting. Unit memberikan


makna kepada nombor dalam setiap penghitungan dalam Fizik.

* Nilai nombor menjadi berbeza apabila unit yang berlainan digunakan,


(misalnya, 9.2 m dan 9.2 mm mewakili panjang yang berbeza).

* Unit merupakan sebahagian penting dalam ‘bahasa Fizik’ yang kita


sering gunakan. Unit mesti disebut dengan jelas apabila kuantiti fizik
dikemukakan. Unit dapat ‘menceritakan kisah Fizik’.

* Unit menggambarkan konsep fizik dengan lebih jelas. Unit merupakan


blok-blok pembinaan dalam Fizik.

* Penggunaan unit dalam pengiraan membolehkan murid


mengelakkan kesilapan secara automatik.

* Dengan menggunakan unit secara teliti dan lengkap dalam


pengiraan, seseorang murid akan mencapai kejayaan dalam
peperiksaan Fizik dan juga membina batu asas dalam pendidikan
Fizik.

Walau bagaimanapun, cikgu-cikgu yang mengajar berhak untuk memilih


sama ada untuk menggunakan unit dalam pengiraan setiap langkah atau
mengikuti format peperiksaan SPM di mana murid hanya perlu meletakkan
unit yang betul pada akhir jawapan sahaja.

© Nilam Publication Sdn. Bhd.


MODUL • Fizik TINGKATAN 4

KANDUNGAN
CONTENTS

TEMA: ASAS FIZIK TEMA: MEKANIK NEWTON


THEME: ELEMENTARY PHYSICS THEME: NEWTONIAN MECHANICS

UNIT Pengukuran I
UNIT Kegravitian 62
1 Measurement 3 Gravitation

1.1 Kuantiti Fizik 3.1 Hukum Kegravitian Semesta Newton


Physical Quantities Newton’s Universal Law of Gravitation
1.2 Penyiasatan Saintifik 3.2 Hukum Kepler
Scientific Investigation Kepler’s Laws
Praktis SPM 3.3 Satelit Buatan Manusia
SPM Practice Man-made Satellites
Praktis SPM
SPM Practice

TEMA: MEKANIK NEWTON TEMA: HABA


THEME: NEWTONIAN MECHANICS THEME: HEAT

UNIT Daya dan Gerakan I 12


UNIT Haba 81
2 Force and Motion I 4 Heat

2.1 Gerakan Linear 4.1 Keseimbangan Terma


Linear Motion Thermal Equilibrium
2.2 Graf Gerakan Linear 4.2 Muatan Haba Tentu
Linear Motion Graphs Specific Heat Capacity
2.3 Gerakan Jatuh Bebas 4.3 Haba Pendam Tentu
Free Fall Motion Specific Latent Heat
2.4 Inersia 4.4 Hukum Gas
Inertia Gas Laws
2.5 Momentum Praktis SPM
Momentum SPM Practice
2.6 Daya
Force
2.7 Impuls dan Daya Impuls
Impulse and Impulsive Force
2.8 Berat
Weight
Praktis SPM
SPM Practice

I © Nilam Publication Sdn. Bhd.


MODUL • Fizik TINGKATAN 4

TEMA: GELOMBANG, CAHAYA


DAN OPTIK
THEME: WAVES, LIGHT AND OPTIC

UNIT Gelombang 115


UNIT Cahaya dan Optik 155
5 Waves 6 Light and Optics

5.1 Asas Gelombang 6.1 Pembiasan Cahaya


Fundamental of Waves Refraction of Light
5.2 Pelembapan dan Resonans 6.2 Pantulan Dalam Penuh
Damping and Resonance Total Internal Reflection
5.3 Pantulan Gelombang 6.3 Pembentukan Imej oleh Kanta
Reflection of Waves Image Formation by Lenses
5.4 Pembiasan Gelombang 6.4 Formula Kanta Nipis
Refraction of Waves Thin Lens Formula
5.5 Pembelauan Gelombang 6.5 Peralatan Optik
Diffraction of Waves Optical Instruments
5.6 Interferens Gelombang 6.6 Pembentukan Imej oleh Cermin Sfera
Interference of Waves Image Formation by Sperical Mirrors
5.7 Gelombang Elektromagnet Praktis SPM
Electromagnetic Waves SPM Practice
Praktis SPM
SPM Practice

Standard Prestasi
Tahap
Tidak
Penguasaan Tafsiran Menguasai
menguasai
(TP)
Mengingat kembali pengetahuan dan kemahiran sains mengenai
1
Pengukuran.
Memahami Pengukuran serta dapat menjelaskan kefahaman
2
tersebut.
Mengaplikasikan pengetahuan mengenai Pengukuran untuk
U 3 menerangkan kejadian atau fenomena alam dan melaksanakan
tugasan mudah.
n
i Menganalisis pengetahuan mengenai Pengukuran dalam konteks
4
t penyelesaian masalah mengenai kejadian atau fenomena alam.
Menilai pengetahuan mengenai Pengukuran dalam konteks
1 5 penyelesaian masalah dan membuat keputusan untuk melaksanakan
satu tugasan.
Mereka cipta menggunakan pengetahuan dan kemahiran sains
mengenai Pengukuran dalam konteks penyelesaian masalah atau
6 membuat keputusan atau dalam melaksanakan aktiviti/ tugasan
dalam situasi baharu secara kreatif dan inovatif dengan mengambil
kira nilai sosial/ ekonomi/ budaya masyarakat.

Untuk mendapatkan senarai lengkap ‘Standard


Prestasi’, anda boleh mengimbas kod QR ini.
To obtain a complete list of 'Standard Prestasi', you may
scan this QR code.

© Nilam Publication Sdn. Bhd. II


MODUL • Fizik TINGKATAN 4

Unit
PENGUKURAN
1 MEASUREMENT

1.1 KUANTITI FIZIK / PHYSICAL QUANTITIES SK 1.1

SP 1.1.1 Menerangkan kuantiti fizik

Apakah itu kuantiti fizik? Kuantiti yang terdiri daripada kuantiti asas dan kuantiti terbitan .
What is physical quantity? derived U
Quantity that consists of base quantities and quantities. N
I
T
Kuantiti fizik yang tidak boleh ditakrifkan dalam istilah kuantiti asas yang
Apakah itu kuantiti asas?
lain. 1
What is a base quantity?
A physical quantity which cannot be defined in terms of other base quantities.

Kuantiti fizik yang diterbitkan daripada kuantiti asas melalui


Apakah itu kuantiti pendaraban atau pembahagian atau kedua-duanya.
terbitan?
What is derived quantity? A physical quantity which is derived from base quantities through
multiplication or division or both.

Unit metrik Unit imperial


Metric unit Imperial unit
Meter, kilometer, gram, kilogram dan Ela, inci, kaki, batu, gelen dan
saat. lain-lain.
Metre, kilometre, gram, kilogram and Yard, inch, foot, mile, gallon and
seconds. others.
Sistem metrik yang mewakili satu Unit imperial ini dikenali
unit tunggal bagi sebarang kuantiti sebagai ukuran yang pernah
fizik digunakan oleh semua negara. digunakan di Britain dan
Metric system that represents one single Amerika Syarikat.
Nyatakan perbezaan unit for any physical quantity that is Imperial unit is known as a
antara unit metrik dan used by all countries. measurement that has been used in
unit imperial. Britain and United States.
State the differences between
metric unit and imperial unit. Isikan tempat kosong di bawah dengan unit imperial yang betul:
Fill in the blanks for the correct imperial units below:
(a) (b) (c)

Inci / Inch Kaki / Foot Ela / Yard

1 © Nilam Publication Sdn. Bhd.

B1FizikF4(p1-10)csy6p.indd 1 09/01/2020 8:48 AM


MODUL • Fizik TINGKATAN 4

SP 1.1.2 Menerangkan dengan contoh kuantiti asas dan kuantiti terbitan

1 Lengkapkan jadual di bawah tentang senarai kuantiti asas serta simbol dan unit S.I.nya.
Complete the table of base quantities with its symbols and S.I units.

Simbol untuk
Kuantiti asas Unit S.I. Simbol untuk unit S.I.
kuantiti asas
Base quantity S.I. unit Symbol for S.I. unit
Symbol for base quantity
Panjang meter
l m
Length metre

Mass
m kilogram kg
Jisim

Masa saat
t s
U Time second
N
I Suhu
T termodinamik
T kelvin K
1 Thermodynamic
temperature
Arus elektrik
I ampere A
Electric current

Keamatan
berluminositi
I candela cd
Luminous
intensity

Kuantiti bahan mol


Amount of n mol
mole
substance

SP 1.1.3 Memerihalkan kuantiti terbitan dalam sebutan kuantiti asas dan unit asas S.I.

2 Tentukan unit terbitan bagi kuantiti terbitan yang berikut.


Determine the derived unit for the following derived quantities.

Kuantiti Unit terbitan dari


terbitan Hubungan dengan kuantiti asas unit asas
Derived Relationship with base quantity Derived unit
quantity from base units

Luas / Area Luas = panjang × panjang Area = length × length m × m = m2

Isi padu Isi padu Volume


m × m × m = m3
Volume = panjang × panjang × panjang = length × length × length

Ketumpatan Ketumpatan Density kg


Density jisim mass = kg m–3
= = m3
panjang × panjang × panjang length × length × length

© Nilam Publication Sdn. Bhd. 2

B1FizikF4(p1-10)csy6p.indd 2 09/01/2020 8:48 AM


MODUL • Fizik TINGKATAN 4

Kuantiti Unit terbitan dari


terbitan Hubungan dengan kuantiti asas unit asas
Derived Relationship with base quantity Derived unit
quantity from base units

Halaju sesaran displacement m –1


Halaju = Velocity =
Velocity masa time s =ms

m s–1
Pecutan Pecutan = perubahan halaju Acceleration = change in velocity = m s–2
Acceleration masa time s

Berat Weight
Berat = jisim × pecutan graviti = mass × gravitational acceleration
kg m s–2
Weight U
N
I
T
Momentum
Momentum
Momentum = jisim × halaju Momentum = mass × velocity kg m s–1 1
Daya
Daya = jisim × pecutan Force = mass × acceleration kg m s–2; newton (N)
Force

kg m–1 s–2 ; N m–2 ;


Tekanan Tekanan = daya
force
Pressure =
Pressure luas area pascal (Pa)

Kerja
Kerja = daya × sesaran Work = force × displacement kg m2 s–2; joule (J)
Work

Kuasa kerja Power =


work
Kuasa = time kg m2 s–3 ; watt (W)
Power masa

Tenaga kinetik Kinetic energy


Tenaga kinetik 1 1
= × mass × (velocity)2 kg m2 s–2 ; joule (J)
Kinetic energy = × jisim × (halaju)2
2 2

Tenaga
keupayaan Tenaga keupayaan graviti Gravitational potential energy
graviti = jisim × pecutan graviti × = mass × gravitational kg m2 s–2 ; joule (J)
Gravitational tinggi acceleration × height
potential energy

Cas
Cas = arus × masa Charge = current × time A s ; coulomb (C)
Charge

Voltan = kerja
Voltan work
Voltage = J C–1 ; volt (V)
Voltage cas charge

Rintangan = voltan
Rintangan voltage
Resistance = V A–1 ; ohm (Ω)
Resistance arus current

3 © Nilam Publication Sdn. Bhd.

B1FizikF4(p1-10)csy6p.indd 3 09/01/2020 8:48 AM


MODUL • Fizik TINGKATAN 4

SP 1.1.4 Menerangkan dengan contoh kuantiti skalar dan kuantiti vector

Kuantiti fizik yang mempunyai magnitud sahaja.


A physical quantity which has magnitude only.
Berikan definisi kuantiti skalar.
[suhu, masa, laju, jarak, isi padu] • Contoh : Seorang budak perempuan berjalan sejauh 4 meter.
Define scalar quantity. Example : A girl walks 4 metres.
[temperature, time, speed, distance, • Magnitud : 4 meter
volume] Magnitude : 4 metres
• Arah : Tiada arah
Direction : No direction

Kuantiti fizik yang mempunyai kedua-dua magnitud


U
N
I dan arah .
T
magnitude
1 Berikan definisi kuantiti vektor.
A physical quantity which has both
direction
and
.
[halaju, pecutan, sesaran,
momentum, daya] • Contoh : Seorang budak perempuan berjalan sejauh
Define vector quantity. 4 meter ke arah Timur.
[velocity, acceleration, displacement,
Example : A girl walks 4 metres towards East.
momentum, force]
• Magnitud : 4 meter
Magnitude : 4 metres
• Arah : Timur
Direction : East

Kuantiti Kategori
Quantity Category

Kuantiti skalar (tiada arah disertakan pada jarak)


5m Scalar quantity (there is no direction listed for the
distance)

Kuantiti vektor (terdapat arah disertakan pada


30 cm s–1,
Pertimbangkan kuantiti-kuantiti kelajuan)
Timur
yang disenaraikan di dalam Vector quantity (there is direction listed for the
East 30 cm s–1
jadual. Kategorikan setiap speed)
kuantiti sama ada kuantiti vektor
atau kuantiti skalar. Kuantiti vektor (terdapat arah disertakan pada
Consider the following quantities listed 5 km, Utara jarak)
5 km, North Vector quantity (there is direction listed for the
in the table. Categorise each quantity
as being either a vector quantity or a distance)
scalar quantity.
Kuantiti skalar (arah tidak terlibat)
20 °C
Scalar quantity (there is no direction involved)

256 bit Kuantiti skalar (arah tidak terlibat)


256 bytes Scalar quantity (there is no direction involved)

4 000 kalori Kuantiti skalar (arah tidak terlibat)


4 000 calories Scalar quantity (there is no direction involved)

© Nilam Publication Sdn. Bhd. 4

B1FizikF4(p1-10)csy6p.indd 4 09/01/2020 8:48 AM


MODUL • Fizik TINGKATAN 4

1.2 PENYIASATAN SAINTIFIK / SCIENTIFIC INVESTIGATION SK 1.2

SP 1.2.1 Mentafsir bentuk-bentuk graf untuk menentukan hubungan antara dua kuantiti fizik

Lengkapkan jadual di bawah bagi bentuk graf yang menunjukkan hubungan antara dua kuantiti fizik.
Complete the table below on the shape of the graphs that shows a relationship between two physical quantities.

1 y
Kecerunan graf 2 y
Kecerunan graf
(x2 , y2) Gradient of graph Gradient of graph
(x2 , y2) Δy
∆y = y2 – y1 Δy ∆y m = Δ x
m = Δ x (x1 , y1)
(x1 , y1) ∆x y2 – y1
∆x = x2 – x1 y2 – y1 =
= (0, c) x2 – x1
0 x x2 – x1 0 x U
y berkadar terus dengan x y = mx c = Pintasan paksi-y N
y bertambah secara linear I
y is directly proportional to x dengan x y-intercept T

y increases linearly with x Persamaan linear 1


Linear equation
y = mx + c

3 y 4 y
Kecerunan graf
(0, c) (x1 , y1) Gradient of graph
Δy
∆y
(x2 , y2) m = Δ x
0 ∆x x y2 – y1 0
= x
x2 – x1
y berkurang secara linear y bertambah dengan x
dengan x c = Pintasan paksi-y y increases with x
y decreases linearly with x y-intercept
Persamaan linear
Linear equation
y = mx + c

5 y 6 y

0 x 0 x
y berkadar songsang dengan x y berkurang dengan x
y is inversely proportional to x y decreases with x

5 © Nilam Publication Sdn. Bhd.

B1FizikF4(p1-10)csy6p.indd 5 09/01/2020 8:48 AM


MODUL • Fizik TINGKATAN 4

SP 1.2.2 Menganalisis graf untuk mendapatkan rumusan siasatan

Contoh / Example

Seorang murid telah menjalankan satu eksperimen untuk menyiasat hubungan antara jisim, m bagi duit syiling
dengan isi padunya, V. Duit syiling yang berjisim 20.0 g dimasukkan secara perlahan-lahan ke dalam sebuah
silinder penyukat yang mengandungi air satu demi satu. Bacaan yang diperoleh ditunjukkan di bawah.
A student carried out an experiment to investigate the relationship between mass, m for coins with its volume, V. A coin of
mass of 20.0 g has been put slowly into a measuring cylinder of water one by one. The readings measured are shown below.

• Sekeping duit syiling serupa menyesar 3.0 cm3 air


One coin displaced 3.0 cm3 of water
• Dua keping duit syiling serupa menyesar 6.0 cm3 air
Two identical coins displaced 6.0 cm3 of water
• Tiga keping duit syiling serupa menyesar 9.0 cm3 air
U Three identical coins displaced 9.0 cm3 of water
N
I • Empat keping duit syiling serupa menyesar 12.0 cm3 air
T Four identical coins displaced 12.0 cm3 of water
1 • Lima keping duit syiling serupa menyesar 15.0 cm3 air
Five identical coins displaced 15.0 cm3 of water

(a) Rekod keputusan yang diperoleh bagi semua nilai m dan V di dalam jadual di bawah.
Record the results obtained for all values of m and V in the table below.

Jisim / Mass, m (g) 20.0 40.0 60.0 80.0 100.0


3
Isi padu / Volume, V (cm ) 3.0 6.0 9.0 12.0 15.0

(b) Lukis graf V melawan m. / Draw the graph of V against m.

Isi padu, V / cm3


Volume, V / cm3

16

14

12

10

6 Δy = y2 – y1

2
Δx = x2 – x1
Jisim, m / g
0 20 40 60 80 100 Mass, m / g

© Nilam Publication Sdn. Bhd. 6

B1FizikF4(p1-10)csy6p.indd 6 09/01/2020 8:48 AM


MODUL • Fizik TINGKATAN 4

(c) Berdasarkan graf tersebut, nyatakan hubungan antara V dan m.


Based on the graph, state the relationship between V and m.
V berkadar terus dengan m.
V is directly proportional to m.

(d) Tentukan nilai V apabila m = 50.0 g. Tunjukkan pada graf tersebut bagaimana anda menentukan nilai V.
State the value of V when m = 50.0 g. Show in the graph how you obtained the value of V.
V = 7.4 cm3

(e) Hitungkan kecerunan, m bagi graf isi padu, V melawan jisim, m. Tunjukkan pada graf tersebut bagaimana
anda menghitung nilai m.
Calculate the gradient, m for the graph of volume, V against mass, m. Show in the graph how you calculate the value U
of the gradient. N
Δy I
m = T
Δx
y –y 1
= 2 1
x2 – x1
(15 – 0) cm3
=
(100 – 0) g
= 0.15 cm3 g–1

(f) Hitungkan luas di bawah graf berserta dengan unitnya.


Calculate the area under the graph with its unit.
Luas di bawah graf / Area under the graph
1
= × 100 g × 15 cm3
2
= 750 cm3 g

(g) Sekiranya anda memasukkan duit syiling serupa yang keenam ke dalam selinder air tersebut berapakah isi
padu enam duit syiling itu?
If you now insert the sixth identical coin into the above cylinder of water, what is the total volume of the six coins?
V = 18.0 cm3

7 © Nilam Publication Sdn. Bhd.

B1FizikF4(p1-10)csy6p.indd 7 09/01/2020 8:48 AM


MODUL • Fizik TINGKATAN 4

SP 1.2.3 Menjalankan penyiasatan saintifik dan menulis laporan lengkap melalui eksperimen bandul ringkas

Eksperimen Bandul ringkas / Simple pendulum


Experiment

Inferens Panjang bandul ringkas mempengaruhi tempoh ayunan bandul ringkas.


Inference The length of a simple pendulum affects the period of oscillation of the simple pendulum.

Hipotesis Semakin panjang benang bandul ringkas, semakin panjang tempoh ayunan.
Hypothesis The longer the length of the simple pendulum, the longer its period of oscillation.

Untuk menyiasat hubungan antara panjang bandul ringkas dengan tempoh ayunan.
Tujuan
To investigate the relationship between the length of a simple pendulum and its period of
Aim
U
N oscillation.
I
T
Pemboleh ubah dimanipulasikan / Manipulated variable:
1 Panjang bandul, l / The length of the pendulum, l

Pemboleh ubah Pemboleh ubah bergerak balas / Responding variable:


Variables Tempoh ayunan bandul ringkas, T / The period of the simple pendulum, T

Pemboleh ubah dimalarkan / Constant (fixed) variable:


Jisim ladung bandul / Mass of pendulum bob

Ladung bandul, benang 70 cm panjang, kaki retort dan pengapit, jangka sudut, jam
Bahan dan radas
Materials and randik dan pembaris meter. / Pendulum bob, 70 cm length of thread, retort stand and clamp,
apparatus
protractor, stopwatch and metre rule.

(a) Semua radas disusun seperti yang ditunjukkan dalam rajah.


All the apparatus are set up as shown in the diagram.
(b) Panjang bandul dilaraskan supaya panjang, l = 10.0 cm. Benang
The pendulum is adjusted so that its length, l = 10.0 cm. Thread
l
(c) Bandul diayun untuk memulakan ayunan melalui sudut
yang kecil (tidak lebih daripada 10o). Ladung bandul
The pendulum bob is displaced through a small angle (not Pendulum bob
more than 10o). Kaki retort
Retort stand
(d) Masa yang diambil untuk 10 ayunan lengkap, t, diambil
menggunakan jam randik. / The time taken for 10 complete
Susunan radas
oscillations, t, is taken by using the stopwatch.
dan prosedur
Arrangement of (e) Tempoh ayunan bandul ringkas, T (iaitu masa yang diambil untuk satu ayunan
the apparatus and lengkap) dikira dengan menggunakan: / The period of oscillation of the simple pendulum,
procedure T (i.e. the time taken for one complete oscillation) is calculated by using:
Tempoh, T = Masa untuk 10 ayunan lengkap
10
Time for 10 complete oscillations
Period, T =
10
(f) Langkah (c) hingga (e) diulangi untuk panjang bandul l = 20.0 cm, 30.0 cm, 40.0 cm,
50.0 cm dan 60.0 cm.
Steps (c) to (e) are repeated with length of pendulum, l = 20.0 cm, 30.0 cm, 40.0 cm, 50.0
cm and 60.0 cm.
(g) Semua data direkodkan di dalam jadual. / All data are recorded in the table.

© Nilam Publication Sdn. Bhd. 8

B1FizikF4(p1-10)csy6p.indd 8 09/01/2020 8:48 AM


MODUL • Fizik TINGKATAN 4

Masa yang diambil untuk


10 ayunan lengkap, t (s) Tempoh ayunan
Panjang,
The time taken for 10 complete bandul, T (s)
l (cm)
oscillations, t (s) The period of T2 (s2)
Length,
oscillation of the
l (cm) tpurata
t1 t2 pendulum, T (s)
taverage
Keputusan 10.0
Results
20.0
30.0
40.0
50.0
U
60.0 N
I
T
1 (a) Graf T2 melawan l diplot pada kertas graf.
A graph of T2 against l is plotted by using a graph paper. 1
T2 (s2)

Analisis data l (cm)


Analysis of data 0

2 Berdasarkan graf yang diplot, nyatakan hubungan antara dua pemboleh ubah itu.
Based on the graph plotted, state the relationship between the two variables.
3 Tentukan kecerunan graf, m, bagi graf T2 melawan l. Tunjukkan cara pengiraan
dengan jelas.
Determine the gradient of the graph, m, for T2 against l. Show the calculation clearly.
4 Adakah hipotesis diterima? / Is hypothesis accepted?
Ya / Yes

Semakin panjang benang bandul ringkas, semakin panjang tempoh ayunan bertambah.
Kesimpulan
Conclusion The longer the length of the simple pendulum, the longer its period of oscillation.

Praktis SPM / SPM Practice


Soalan Objektif / Objective Questions
1 Antara ukuran berikut, yang manakah paling 2 Arus elektrik, I, diberi oleh
pendek? Electric current, I, is given by
Which of the following measurements is the shortest? Q
I=
A 2
2.45 × 10 mm t
B 2.45 × 10–3 cm Apakah unit bagi I? / What is the unit of I?
C 2.45 × 10–2 μm A C s–1 C J C–1
D 2.45 × 103 nm B C s D J C

9 © Nilam Publication Sdn. Bhd.

B1FizikF4(p1-10)csy6p.indd 9 09/01/2020 8:48 AM


MODUL • Fizik TINGKATAN 4

3 Simbol manakah menunjukkan perkaitan imbuhan


A F berkadar songsang dengan 1
yang tidak betul? t
Which symbol is not correct in representing its F is inversely proportional to 1
t
B F berkadar terus dengan 1
corresponding prefix?
t
Imbuhan Simbol F is directly proportional to 1
Prefix Symbol t
C F berkadar secara linear dengan 1
A tera T t
1
F is linearly proportional to t
B kilo L
D F berkadar secara linear dengan t
C mikro / micro μ F is linearly proportional to t
D senti / centi c
7 Rajah 2 menunjukkan graf T2 melawan l.
Diagram 2 shows a graph of T2 against l.
4 Antara berikut, yang manakah kesemuanya kuantiti
U T 2 / s2
N terbitan?
I Which of the following are all derived quantities?
T 2.5

1 A Daya, panjang, masa


Force, length, time
2.0
1.5
B Momentum, halaju, daya
1.0
Momentum, velocity, force
C Panjang, halaju, suhu 0.5
Length, velocity, temperature 0 l/m
D Arus elektrik, masa, suhu 0.1 0.2 0.3 0.4 0.5
Electric current, time, temperature Rajah 2 / Diagram 2
Persamaan yang manakah betul bagi graf ini?
5 Unit S.I. manakah yang betul bagi kuantiti yang Which equation is correct for the graph?
diberi? A T2 = 4l C T2 = 0.2l
Which of the S.I. units of the quantities given is correct?
B l = 4T2 D l = 0.2T2
Kuantiti Unit S.I.
Quantity S.I. unit 8 Pasangan kuantiti manakah adalah benar?
Which pair of quantities is correct?
Panjang Sentimeter
A
Length Centimeter Kuantiti skalar Kuantiti vektor
Scalar quantity Vector quantity
Jisim Gram
B A
Mass Gram Mempunyai arah Mempunyai
sahaja. magnitud sahaja.
Masa Minit Has direction only. Has magnitude only.
C
Time Minute
B Mempunyai Mempunyai
Suhu Kelvin
D magnitud sahaja. magnitud sahaja.
Temperature Kelvin
Has magnitude only. Has magnitude only.

6 Berdasarkan keputusan graf pada Rajah 1, hubungan C Mempunyai Mempunyai


yang manakah adalah benar? / Based on the result of magnitud sahaja. magnitud dan arah.
the graph in Diagram 1, which relationship is correct? Has magnitude only. Has magnitude and
direction.
F/N
D Mempunyai Mempunyai arah
magnitud dan arah. sahaja.
Has magnitude and Has direction only.
1 direction.
/ s–1
0 t

Rajah 1 / Diagram 1

© Nilam Publication Sdn. Bhd. 10

B1FizikF4(p1-10)csy6p.indd 10 09/01/2020 8:48 AM


MODUL • Fizik TINGKATAN 4

Soalan Struktur / Structure Question


1 Rajah 1 menunjukkan suatu eksperimen untuk menentukan hubungan antara daya, F, dengan pemanjangan, x, bagi
sebuah spring.
Diagram 1 shows an experiment to determine the relationship between the force, F, and the extension of a spring, x.

Spring

Penunjuk / Pointer
U
Daya / Force, F N
I
T

Rajah 1 / Diagram 1 1
(a) Tulis satu hipotesis yang sesuai yang melibatkan daya, F, dan pemanjangan, x.
Write a suitable hypothesis involving the force, F, and the extension, x.

x bertambah apabila F bertambah.


x increases with F.

(b) Bagaimanakah kejituan pengukuran bagi x dapat ditingkatkan?


How can the accuracy of the measurement of x be increased?

Bagi setiap nilai F, ambil sekurang-kurangnya dua bacaan bagi x. Kemudian ambil nilai purata bagi x.
Take at least two measurements of x for each value of F. Then take the average of x.

(c) Graf di bawah menunjukkan keputusan eksperimen itu.


The graph below shows the results of the experiment.
x / cm

0 F/N

Berdasarkan graf di atas, tulis satu kesimpulan terhadap hubungan antara F dan x.
Based on the graph above, write a suitable conclusion on the relationship between F and x.

x adalah berkadar terus dengan F.


x is directly proportional to F.

(d) Tulis satu langkah berjaga-jaga yang harus diambil dalam eksperimen di atas.
Write one precaution that should be taken in the above experiment.

Elakkan ralat paralaks dengan memastikan aras mata adalah sama dengan aras penunjuk.
Avoid parallax errors by making sure that the eye-level is the same as the pointer level.

11 © Nilam Publication Sdn. Bhd.

B1FizikF4(p1-10)csy6p.indd 11 09/01/2020 8:48 AM


MODUL • Fizik TINGKATAN 4

Unit
DAYA DAN GERAKAN I
2 FORCE AND MOTION I

2.1 GERAKAN LINEAR / LINEAR MOTION SK 2.1

SP 2.1.1 Menghuraikan jenis gerakan linear bagi objek yang berada dalam keadaan: (i) pegun, (ii) halaju seragam, (iii) halaju tidak seragam

Jumlah panjang laluan yang dilalui dari satu lokasi ke satu lokasi yang lain.
Terangkan
The total path length travelled from one location to another.
jarak
Explain Kuantiti / Quantity: Skalar / Scalar
distance meter (m)
Unit S.I. / S.I. unit:

(a) Jarak dalam arah tertentu. / The distance in a specific direction.


(b) Jarak antara dua lokasi yang diukur sepanjang laluan yang paling pendek yang
U menghubungkannya dalam arah tertentu. / The distance between two locations measured along
N
I
Terangkan the shortest path connecting them in a specific direction.
T sesaran (c) Jarak kedudukan akhir dari kedudukan awal dalam arah tertentu.
2 Explain
displacement
The distance of its final position from its initial position in a specified direction.

Kuantiti / Quantity: Vektor / Vector

Unit S.I. / S.I. unit: meter (m)

Contoh / Example Utara/North

Rahim berjalan dari rumahnya ke simpang K sejauh 1.5 km. Warung Pak Din Rumah Rahim
Pak Din’s stall Rahim’s house
Kemudian, dia berpatah balik dan berhenti di warung Pak Din
yang sejauh 0.5 km dari rumahnya.
Rahim walked from his house to the junction K which is 1.5 km. Then, B O
0.5 km 1.5 km
he turned back and stopped at Pak Din’s stall which is 0.5 km from
K
his house.
(a) Berapakah sesaran Rahim dari rumahnya
What is Rahim’s displacement from his house
(i) apabila dia sampai di simpang K? / when he reached the junction K?
1.5 km ke timur / 1.5 km to the east

(ii) apabila dia berada di warung Pak Din? / when he was at Pak Din’s stall?
0.5 km ke barat / 0.5 km to the west
(b) Selepas bersarapan pagi, Rahim berjalan pulang ke rumahnya. Apabila dia sampai di rumahnya,
After breakfast, Rahim walked back to his house. When he reached home,
(i) berapakah jumlah jarak yang dilalui oleh Rahim? / what was the total distance travelled by Rahim?
1.5 km + 1.5 km + 0.5 km + 0.5 km = 4 km

(ii) berapakah jumlah sesaran Rahim dari rumahnya? / what was Rahim’s total displacement from his house?
0 km

© Nilam Publication Sdn. Bhd. 12


MODUL • Fizik TINGKATAN 4

1 Laju ialah kadar perubahan jarak. / Speed is the rate of change of distance.

Jarak dilalui Speed, v =


Distance travelled
Laju, v = Time taken
Masa yang diambil

Terangkan Kuantiti / Quantity: Skalar / Scalar


laju Unit S.I. / S.I. unit: m s–1
Explain
2 Laju purata / Average speed
speed
Jumlah jarak dilalui, d Total distance travelled, d
v= v =
Jumlah masa yang diambil, t Total time taken, t

Laju seragam Laju yang magnitudnya kekal sama tanpa mempertimbangkan


arahnya.
Uniform speed Speed that remains the same in magnitude regardless of its direction.

1 Halaju, v ialah kadar perubahan sesaran. / Velocity, v is the rate of change of displacement.

Sesaran Velocity, v =
Displacement
Halaju, v = Time taken
Masa yang diambil
U
U
N
N
Kuantiti / Quantity: Vektor / Vector II
T
T
Unit S.I. / S.I. unit: m s–1 12
2 Halaju purata / Average velocity
Terangkan Sesaran, s Displacement, s
halaju v= v =
Jumlah masa yang diambil, t Total time taken, t
Explain
velocity Halaju seragam Halaju yang magnitud dan arahnya kekal sama.
Uniform velocity Velocity that remains the same in magnitude and direction.

3 Suatu objek mempunyai halaju tidak seragam jika:


An object has a non-uniform velocity if:
(a) arah gerakan berubah atau gerakan tidak linear.
the direction of motion changes or the motion is not linear.
(b) magnitud halaju berubah.
the magnitude of its velocity changes.

13 © Nilam Publication Sdn. Bhd.


MODUL • Fizik TINGKATAN 4

Terangkan pecutan / Explain acceleration

1 Pecutan, a, didefinisikan sebagai kadar perubahan halaju.


Acceleration, a, is defined as the rate of change of velocity.
2 Formula dan unit S.I: / Formula and S.I unit:

Pecutan, a Acceleration, a
Perubahan halaju Change in velocity
= Masa yang diambil = Time taken
Halaju akhir, v – Halaju awal, u Final velocity, v – Initial velocity, u
= =
Masa yang diambil, t Time taken, t
v–u v–u
= t = t

Unit S.I. / S.I. unit: m s–2

Pecutan sifar bermaksud suatu objek berada dalam keadaan pegun atau bergerak pada halaju malar, a = 0
Zero acceleration means an object is at rest or is moving at a constant velocity, a = 0

U
N Contoh / Example
I
T
Perhatikan gerakan sebuah kereta dalam Rajah (a) dan Rajah (b) yang merupakan gambar stroboskop.
2 Observe the motion of the car in Diagram (a) and Diagram (b) which are stroboscopic pictures.
Arah gerakan
Direction of motion

(a) (b)

Arah gerakan Arah gerakan


Direction of motion Direction of motion

(b)

(a) Huraikan perubahan halaju sebuah kereta dalam Rajah (a). Adakah kereta memecut atau menyahpecut?
Describe the changes in velocity of the car in Diagram (a). Is the car accelerating or decelerating?
Jarak antara dua gambar berturut-turut bertambah. Halaju kereta itu bertambah. Kereta itu memecut.
The distance between two consecutive images increases. Velocity of the car is increasing. The car accelerates.

(b) Huraikan perubahan halaju sebuah kereta dalam Rajah (b). Adakah kereta memecut atau menyahpecut?

Describe the changes in velocity of the car in Diagram (b). Is the car accelerating or decelerating?
Jarak antara dua gambar berturut-turut berkurang. Halaju kereta itu berkurang. Kereta itu mengalami
nyahpecutan.
The distance between two consecutive images decreases. Velocity of the car is decreasing. The car decelerates.

© Nilam Publication Sdn. Bhd. 14


MODUL • Fizik TINGKATAN 4

SP 2.1.2 Menentukan: (i) jarak dan sesaran, (ii) laju dan halaju, (iii) pecutan/nyahpecutan

Latihan / Exercises
400 km j–1
1 Sebuah kapal terbang menuju ke utara selama 1 jam dengan halaju 300 km j–1. A B
Kemudian, kapal terbang itu menuju ke timur selama 1 jam dengan halaju
400 km j–1. 300 km j–1
An aeroplane flies towards the north for 1 hour with a velocity of 300 km h–1.
Then, the aeroplane flies towards the east for 1 hour with a velocity of 400 km h–1.
(a) Berapakah laju purata kapal terbang itu? O
What is the average speed of the aeroplane?
Jumlah jarak Total distance
Laju purata = Average speed = Total time
Jumlah masa
Jarak OA = LajuOA × MasaOA Distance OA = SpeedOA × TimeOA
= 300 km j–1 × 1 jam = 300 km h–1 × 1 hour
= 300 km = 300 km
Jarak AB = LajuAB × MasaAB Distance AB = SpeedAB × TimeAB
= 400 km j–1 × 1 jam = 400 km h–1 × 1 hour
= 400 km = 400 km
(300 km + 400 km) (300 km + 400 km)
∴ Laju purata = 2 jam ∴ Average speed = 2 hours
700 km 700 km U
U
= –1
2 jam = 350 km j = 2 hours = 350 km h–1 N
N
II
T
T
12
(b) Berapakah halaju purata kapal terbang itu?
What is the average velocity of the aeroplane?
Dari (a); From (a);
JarakOA = 300 km DistanceOA = 300 km
JarakAB = 400 km DistanceAB = 400 km
∴ SesaranOB = (300 km)2 + (400 km)2 ∴ DisplacementOB = (300 km)2 + (400 km)2
= 500 km = 500 km
Sesaran Displacement
∴ Halaju purata = Masa ∴ Average velocity = Time
500 km 500 km
= = 2h
2j
= 250 km h–1
= 250 km j–1

2 Bacaan meter laju bagi sebuah kereta yang bergerak ke arah utara menunjukkan 80 km j–1. Sebuah kereta
yang lain bergerak pada 80 km j–1 menuju ke selatan. Adakah kelajuan kedua-dua kereta itu sama? Adakah
halaju kedua-dua kereta itu sama? Terangkan jawapan anda.
The speedometer reading for a car travelling towards north shows 80 km h–1. Another car is travelling at 80 km h–1
towards the south. Is the speed of both cars the same? Is the velocity of both cars the same? Explain your answer.
Kelajuan kedua-dua kereta itu adalah sama iaitu 80 km j–1 tetapi halaju adalah tidak sama kerana arah
kedua-dua kereta itu berbeza.
The speed of both cars is the same, that is, 80 km h–1 but the velocity is not the same because the cars are in different
directions.

15 © Nilam Publication Sdn. Bhd.


MODUL • Fizik TINGKATAN 4

3 Seorang penunggang basikal bermula dari keadaan rehat dan menambahkan halajunya pada kadar seragam
sehingga dia mencapai halaju 4.0 m s–1 dalam 5.0 s. Berapakah purata pecutannya?
A cyclist starts from rest and increases his velocity at a constant rate until he reaches a velocity of 4.0 m s–1 in 5.0 s.
What is his average acceleration?
v–u
Halaju awal / Initial velocity = 0 pecutan / acceleration, a = t
Halaju akhir / Final velocity = 4.0 m s–1 . (4.0 – 0) m s–1
a =
5.0 s 5.0 s
Masa yang diambil / Time taken = .
= 0.8 m s–2

4 Isikan tempat kosong: / Fill in the blanks:


(a) Laju malar 10 m s–1 Jarak sejauh 10 m dilalui setiap saat .
A constant speed of 10 m s–1 A distance of 10 m travelled every second .

(b) Halaju malar –10 m s–1 Sesaran sejauh 10 m yang dilalui setiap saat dalam arah bertentangan.
A constant velocity of –10 m s–1 A displacement of 10 m travelled every second in the opposite
direction.

(c) Pecutan malar 4 m s–2 Halaju meningkat dengan 4 m s–1 setiap saat .
A constant acceleration of 4 m s –2
Velocity increases by 4 m s –1
every second .
U
(d) Nyahpecutan malar 4 m s–2 Halaju berkurang dengan 4 m s–1 setiap saat .
N
I
T A constant deceleration of 4 m s–2 Velocity decreases by 4 m s–1 every second .
2
Hubung kait Sesaran, Halaju, Pecutan dan Masa
Relating Displacement, Velocity, Acceleration and Time

Jangka masa detik: / Ticker timer:


6 – 12 V a.c.
• Ia disambung ke bekalan kuasa arus ulang-alik 50 Hz.
Apabila dihidupkan, bilah pengetuk akan bergetar 50
kali sesaat. / It is connected to an alternating current supply
of 50 Hz. When it is switched on, the iron strip will vibrate
50 times per second.
Jangka masa detik
• Masa diambil untuk membuat 50 titik pada pita detik Ticker timer
Troli dinamik
ialah 1 saat. Jadi, selang masa antara dua titik yang Dynamic trolley
1
berturutan ialah s = 0.02 s. / The time taken to make 50
50
dots on the ticker tape is 1 second. Hence, the time interval
1
between 2 consecutive dots is s = 0.02 s.
50
• 1 detik didefinisikan sebagai selang masa antara 2 titik.
1 tick is defined as the time interval between 2 dots.

© Nilam Publication Sdn. Bhd. 16


MODUL • Fizik TINGKATAN 4

Aktiviti 1: Kaedah pengiraan / Activity 1: Method of calculation

Menyiasat gerakan dalam makmal untuk menentukan jarak/sesaran, kelajuan/halaju, masa dan pecutan/
nyahpecutan / To investigate motion in laboratory to determine distance/displacement, speed/velocity, time and
acceleration/deceleration

10 detik / ticks

Arah
gerakan
Direction of
motion A 8.0 cm B

LANGKAH 1: Menentukan masa diambil untuk 1 detik. / STEP 1: Determine the time taken for 1 tick.
(a) Masa yang diambil untuk 50 detik = 1 saat (a) Time taken for 50 ticks = 1 second
(b) Masa yang diambil untuk 1 detik = 0.02 s (b) Time taken for 1 tick = 0.02 s
(c) Masa yang diambil dari A ke B = 10 detik (c) Time taken from A to B = 10 ticks
= 0.2 s = 0.2 s
U
U
LANGKAH 2: Menentukan sesaran / STEP 2: Determine the displacement N
N
II
Sesaran suatu objek ditentukan dengan mengukur panjang pita detik yang ditarik melalui jangka masa detik T
T
The displacement of the object is determined by measuring the length of the ticker tape that is pulled through the ticker timer 12
Sesaran A ke B = 8.0 cm / Displacement from A to B = 8.0 cm
LANGKAH 3: Menentukan halaju / STEP 3: Determine the velocity
Sesaran Displacement
Velocity, v =
Halaju, v = Time
Masa
8.0 cm 8.0 cm
= =
0.2 s 0.2 s
40.0 = 40.0 cm s–1
= cm s–1

LANGKAH 4: Menentukan pecutan / STEP 4: Determine the acceleration

1.5 cm 3.5 cm 5.5 cm 7.5 cm

Arah gerakan
Direction of
A u B C D ν E
motion

Dari jalur pertama: Halaju awal, u, pada AB Selang masa, bagi perubahan halaju, t
From the first strip: Initial velocity, u, at AB The time interval, t, for the change in the velocity
1.5 cm t = (4 – 1) × 0.2 s = 0.6 s
uAB = = 7.5 cm s–1
0.2 s

Dari jalur terakhir: Halaju akhir, v, pada DE Pecutan / Acceleration,


From the final strip: Final velocity, v, at DE
(37.5 – 7.5) cm s–1
7.5 cm a= = 50.0 cm s–2
vAB = = 37.5 cm s–1 0.6 s
0.2 s

17 © Nilam Publication Sdn. Bhd.


MODUL • Fizik TINGKATAN 4

Latihan / Exercises

1 Berdasarkan rajah di bawah, hitungkan pecutan 2 Rajah di bawah menunjukkan suatu pita detik yang
objek itu. / Based on the diagram below, calculate mengandungi 5 detik untuk setiap selang AB, BC,
the acceleration of the object. CD dan DE. Hitungkan pecutan objek itu.
The diagram below shows a ticker tape contains 5 ticks
Arah pergerakan for every interval AB, BC, CD and DE. Calculate the
Direction of motion
acceleration of the object.
Arah pergerakan
Direction of motion
A 2.0 cm B 4.0 cm C 6.0 cm D 8.0 cm E

0.2 cm 1.4 cm

Penyelesaian / Solution: Penyelesaian / Solution:


0.2 cm –1 8.0 cm
u= 0.02 s = 10 cm s u= –1
0.1 s = 80.0 cm s
1.4 cm
v = 0.02 s = 70 cm s–1 2.0 cm –1
v= 0.1 s = 20.0 cm s
t = (5 – 1) × 0.02 s = 0.08 s
t = (4 – 1) × 0.1 s = 0.3 s
U (70 – 10) cm s–1
N a= 0.08 s = 750 cm s–2 = 7.5 m s–2 (20.0 – 80.0)
I a= 0.3 s = –200 cm s–2 = –2.0 m s–2
T

2
3 Rajah di bawah menunjukkan carta pita detik bagi sebuah troli yang bergerak. Frekuensi bagi jangka masa
detik ialah 50 Hz. Setiap jalur pita mempunyai panjang 10 detik.
The diagram below shows a ticker tape chart for a moving trolley. The frequency of the ticker-timer used is 50 Hz.
Each strip of the tape is a 10-ticks length. Panjang 10 detik / cm
(a) Berapakah selang masa antara dua titik? 10-tick length / cm
What is the time interval between two dots?
12.0
0.02 s Jalur pita
Strip of the tape
10.0
(b) Berapakah selang masa untuk satu jalur?
What is the time interval for one strip? 8.0
0.02 × 10 = 0.2 s
6.0
(c) Berapakah halaju awal? / What is the initial velocity?
2.0 cm
u= = 10.0 cm s–1 4.0
0.2 s
2.0
(d) Berapakah halaju akhir? / What is the final velocity?
12.0 cm –1
v= 0.2 s = 60.0 cm s 0
Detik / Ticks

(e) Berapakah selang masa yang diambil untuk berubah dari halaju awal kepada halaju akhir?
What is the time interval to change from its initial velocity to its final velocity?
t = (11 – 1) × 0.2 s = 2.0 s

(f) Berapakah pecutan objek itu? / What is the acceleration of the object?
(60.0 – 10.0) cm s–1
a= 2.0 s = 25.0 cm s–2

© Nilam Publication Sdn. Bhd. 18


MODUL • Fizik TINGKATAN 4

Aktiviti 2: Untuk mengenal pasti jenis gerakan / Activity 2: To identify the types of motion.

Pita Detik dan Carta / Ticker Tape and Charts Penerangan / Explanation
(i)
Arah
gerakan
• Jenis gerakan: Halaju seragam
Direction
Panjang / Length (cm)
of motion
Type of motion: Uniform velocity

• Jarak dilalui antara dua titik berturutan adalah sama


Distance between two consecutive dots is equal

Masa / Time (s)

(ii) • Jarak antara dua titik berturutan bertambah


Arah
gerakan secara seragam.
Panjang / Length (cm) Direction increases
of motion The distance between two consecutive dots
4.0
uniformly.
3.3
2.6 • Halaju objek itu bertambah secara seragam.
1.9 The velocity of the object increases uniformly.
1.2 U
U
• Objek itu bergerak pada pecutan seragam. N
N
0.5 II
Masa / Time (s) acceleration T
T
The object moves at a uniform .
12
1 2 3 4 5 6

(iii) • Jarak antara dua titik berturutan berkurang


Arah secara seragam.
gerakan
Panjang / Length (cm) Direction The distance between two consecutive dots decreases
of motion
uniformly.

• Halaju objek itu berkurang secara seragam.


The velocity of the object decreases uniformly.
• Objek itu bergerak pada nyahpecutan seragam.
Masa / Time (s)
The object moves at a uniform deceleration .

SP 2.1.3 Menyelesaikan masalah gerakan linear dengan menggunakan persamaan:

Formula Penting / Important Formulae


Menyelesaikan masalah gerakan linear dengan pecutan seragam
Solve problems on linear motion with uniform acceleration

(1) v = u + at (2) s = u + v t
2
1
(3) s = ut + 2 at2 2 2
(4) v = u + 2as
di mana / where:
s: sesaran / displacement v: halaju akhir / final velocity
u: halaju awal / initial velocity a: pecutan / acceleration
t: masa / time

19 © Nilam Publication Sdn. Bhd.


MODUL • Fizik TINGKATAN 4

Latihan / Exercises
1 Sebuah kereta memecut dari keadaan rehat ke 25 m s–1 dalam 4 s. Cari pecutan kereta itu.
A car accelerates from rest to 25 m s–1 in 4 s. Find the acceleration of the car.

Penyelesaian / Solution:
u = 0, v = 25 m s–1, t = 4 s, a = ?
v–u 25 m s–1 – 0
v = u + at a= t
= 4s
= 6.25 m s–2

2 Sebuah kereta memecut dari keadaan rehat pada 3 m s–2


sepanjang suatu jalan lurus. Berapakah sesaran yang 0 m s–1
dilalui oleh kereta itu selepas 4 s? a = 3 m s–2
A car accelerates from rest at 3 m s–2 along a straight road.
How far has the car travelled after 4 s?

Penyelesaian / Solution: Sesaran / Displacement


u = 0, a = 3 m s–2, t = 4 s, sesaran / displacement = ?
1
s = ut + 2 at2
t=4s
1
U = 0 + 2 (3 m s–2)(4 s)2 = 24 m
N
I
T

2
3 Sebuah kereta bergerak dengan halaju 20 m s–1 sepanjang jalan lurus. Pemandu itu menekan brek selama 5 s. Ia
menyebabkan nyahpecutan 3 m s–2, berapakah halaju akhir kereta itu?
A car is travelling at 20 m s–1 along a straight road. The driver brakes for 5 s. This causes a deceleration of 3 m s–2.
What is the final velocity of the car? 20 m s–1 v
20 m s–1 v

Penyelesaian / Solution: a = –3 m s–2


a = –3 m s–2
u = 20 m s–1, t = 5 s, a = –3 m s–2, v = ?
v = u + at
= 20 m s–1 + (–3 m s–2)(5 s)
= 5 m s–1

t=5s
t=5s

4 Sebuah kereta bergerak dengan halaju malar 40 m s–1. Pemandu ternampak suatu penghalang di hadapannya
dan dia segera menekan brek. Dia dapat memberhentikan keretanya dalam masa 8 s. Jarak antara penghalang
itu dari kereta apabila pemandu ternampak penghalang itu ialah 180 m. Berapakah jarak penghalang itu dari
kereta selepas ia berhenti?
A car was moving at a constant velocity of 40 m s–1. The driver saw an obstacle in front and he immediately stepped on
the brake pedal. He managed to stop the car in 8 s. The distance of the obstacle from the car when the driver spotted
it was 180 m. How far was the obstacle from the car after it stopped?

Penyelesaian / Solution:
u = 40 m s–1, v = 0, t = 8 s, s = ?
u+v (40 m s–1 + 0)(8 s)
s = 2
t = 2
= 160 m
Jarak penghalang dari kereta itu selepas berhenti / The distance of the obstacle from the car after it stopped
= 180 m – 160 m
= 20 m  

© Nilam Publication Sdn. Bhd. 20


MODUL • Fizik TINGKATAN 4

2.2 GRAF GERAKAN LINEAR / LINEAR MOTION GRAPHS SK 2.2

SP 2.2.1 Mentafsir jenis gerakan dari graf: (i) sesaran-masa, (ii) halaju-masa, (iii) pecutan-masa

Pentafsiran jenis gerakan daripada graf sesaran-masa


Interpretation of types of motion for displacement-time graph

1 Objek pada keadaan rehat


Object at rest
• Objek berada dalam keadaan rehat kerana ia berada pada kedudukan
s/m yang sama pada bila-bila masa.
The object is at rest because it is at the same position at any time.

• Halaju = kecerunan graf


Velocity = gradient of the graph
= 0 m s–1
0 t/s

2 Objek bergerak dengan U


U
halaju seragam N
N
II
Object moving at uniform • Objek bergerak pada halaju seragam kerana sesaran bertambah secara T
T
velocity seragam dalam setiap saat.
Object travels at uniform velocity because displacement increases constantly in
12
s/m
every second.
• Garis lurus graf mempunyai kecerunan yang tetap.
20 The straight line of the graph has a constant gradient.
• Halaju = kecerunan
Velocity = gradient
= 4 m s–1
0 t/s
5

3 Objek bergerak dengan


pecutan seragam
Object moving with uniform • Objek bergerak dengan pecutan seragam kerana kadar perubahan sesaran
acceleration bertambah.
Object moves with uniform acceleration because the rate of change of displacement
s/m
is increasing.
• Kecerunan lengkungan bertambah menunjukkan halaju bertambah.
The gradient of the curve is increasing showing that velocity is increasing.
• Objek mengalami pecutan seragam.
The object experiences uniform acceleration.
• Kecerunan t2 lebih tinggi daripada kecurunan t1.
Gradient of t2 is greater than gradient of t1
0 t/s
t1 t2

21 © Nilam Publication Sdn. Bhd.


MODUL • Fizik TINGKATAN 4

Pentaksiran jenis gerakan daripada graf halaju-masa


Interpretation of types of motion for velocity-time graph

1 Objek berada dalam


keadaan rehat • Kecerunan = pecutan = 0
Object at rest 0
Gradient = acceleration =
v / m s-¹
• Luas di bawah graf = sesaran = 0

Area under the graph = displacement = 0

• Objek berada dalam keadaan rehat .


Object is at rest .
0 t/s

2 Objek bergerak dengan 0


halaju seragam • Kecerunan / Gradient = pecutan / acceleration =
Object moves with uniform • Luas di bawah graf = sesaran = 10 m s–1 × 2 s = 20 m
velocity
Area under the graph = displacement = 10 m s–1 × 2 s = 20 m
v / m s-¹
• Sesaran dilalui = 20 m
Displacement travelled = 20 m
U 10
N
• Objek bergerak dengan halaju seragam
I
T uniform
t/s Object moves with velocity
2
0
2

3 Objek bergerak dengan 18 m s–1


• Kecerunan / Gradient = malar / constant = = 6 m s–2
pecutan seragam 3s
Object moving with uniform –2
acceleration • Pecutan / Acceleration = 6 m s
v / m s-¹ • Luas di bawah graf / Area under the graph
1 –1
18 = 2 (18 m s )(3 s) = 27 m

• Sesaran dilalui / Displacement travelled = 27 m


• Objek bergerak dengan pecutan seragam
0 t/s
Object moves with uniform acceleration
3

4 Objek bergerak dengan


nyahpecutan seragam • Kecerunan graf adalah malar dan negatif
Object moves with uniform
deceleration The gradient of the graph is constant and negative
v / m s-¹
• Objek itu bergerak dengan nyahpecutan seragam
The object is moving with uniform deceleration

• Simbol negatif menunjukkan nyahpecutan


Negative sign indicates deceleration
0 t/s

© Nilam Publication Sdn. Bhd. 22


MODUL • Fizik TINGKATAN 4

SP 2.2.2 Menganalisis graf sesaran-masa untuk menentukan jarak, sesaran dan halaju

SP 2.2.3 Menganalisis graf halaju-masa untuk menentukan jarak, sesaran, halaju dan pecutan

Latihan / Exercises
1 Rajah di bawah menunjukkan graf sesaran-masa bagi suatu objek. Berdasarkan graf,
The diagram below shows a displacement-time graph of an object. Based on the graph,
(a) Hitungkan halaju objek antara Sesaran / m
Displacement / m
Calculate the velocity of the object between
(i) A dan / and B (ii) B dan / and C (iii) C dan / and D B C
20 m 20 m 20
v= v = 0 v=–
10 s 5s
= 2 m s–1 = –4 m s–1
A D
(b) Gambarkan gerakan objek itu antara 0 10 30 35
Masa / s
Describe the motion of the object between Time / s

A dan B: Halaju malar bergerak ke hadapan


(i)
A and B: Constant velocity
(ii) B dan C: Dalam keadaan rehat U
U
N
B and C: At rest
N
II
T
T
(iii) B dan C: Halaju malar tetapi objek bergerak dalam arah bertentangan atau bergerak semula ke 12
B and C: belakang
Constant velocity but the object moves in opposite direction or going backward

(c) Cari / Find


(i) jumlah jarak / total distance (ii) jumlah sesaran / total displacement

= (20 + 0 + 20) m = 40 m = (20 + 0 – 20) m = 0

(d) Hitungkan / Calculate


(i) laju purata (ii) halaju purata gerakan zarah itu.
the average speed the average velocity of the moving particle.
jumlah jarak total distance sesaran displacement
= masa =
time = masa =
time
40 m 40 m
= 35 s = = 0 = 0
35 s
–1
= 1.14 m s
= 1.14 m s–1

2 Berikut menunjukkan graf halaju-masa bagi sebuah Halaju / Velocity / m s-1


kereta. Berdasarkan graf, / The following shows the
velocity-time graph of a car. Based on the graph, 20
K

(a) Hitungkan pecutan kereta itu bagi


calculate the acceleration of the car for L M
10
(i) JK
20 m s–1
a= = 2 m s–2 J N Q Masa / s
10 s Time / s
0 10 20 30 35 40 45 50
(ii) KL
(10 – 20) m s–1 –10
a = = –1 m s–2 P
(20 – 10) s

23 © Nilam Publication Sdn. Bhd.


MODUL • Fizik TINGKATAN 4

LM
(iii)
a = 0

(iv) MN
–1
a = – 10 m s = –2 m s–2
5s

(b) Nyatakan jenis gerakan kereta itu bagi


State the types of motion of the car for

(i) JK : pecutan malar (iii)


KL : nyahpecutan malar (tetap)
constant acceleration constant deceleration

LM : halaju tetap / malar / pecutan sifar (iv)


(ii) MN : nyahpecutan malar (tetap)
constant velocity / zero acceleration constant deceleration

(c) Hitungkan jumlah sesaran yang dilalui oleh kereta itu semasa
Calculate the total displacement travelled by the car during

U (i) gerakan bagi 10 s yang pertama (ii) kereta bergerak dengan halaju seragam
N the first 10 s of motion the car moves with uniform velocity
I
= 1 (20 m s–1)(10 s)
T Sesaran
Sesaran (LM) = 10 m s–1 × 10 s = 100 m
2 Displacement 2 Displacement
= 100 m
(iii) gerakan bagi 10 s yang terakhir
the last 10 s of motion
Sesaran / Displacement = 1 (10 m s–1)(10 s) = 50 m (pada arah bertentangan / in opposite direction)
2

(d) Hitungkan / Calculate


(i) jumlah jarak bagi keseluruhan perjalanan / the total distance for the whole journey
Jarak / Distance
= 1 (10)(20) + 1 (10 + 20)10 + (10)(10) + 1 (5)(10) + 1 (10)(10)
2 2 2 2
= (100 + 150 + 100 + 25 + 50) m
= 425 m

(ii) jumlah sesaran bagi keseluruhan perjalanan.


the total displacement for the whole journey.
Sesaran / Displacement

2
[
= 1 (10)(20) + 1 (10 + 20)10 + (10)(10) + 1 (5)(10) – 1 (10)(10)
2 2 2
]
= (100 + 150 + 100 + 25) m – 50 m
= 325 m

(e) Hitungkan / Calculate


(i) laju purata / the average speed (ii) halaju purata / the average velocity
Laju purata / Average speed Halaju purata / Average velocity
425 m 325 m
= 45 s = 9.44 m s–1 = 45 s = 7.22 m s–1

© Nilam Publication Sdn. Bhd. 24


MODUL • Fizik TINGKATAN 4

Graf sesaran-masa kepada graf halaju-masa dan sebaliknya


SP 2.2.4
Graf halaju-masa kepada graf pecutan-masa dan sebaliknya

Ringkasan bagi bentuk graf gerakan


Summary of shapes of motion graphs

Graf s melawan t v melawan t a melawan t


Graph s against t v against t a against t

s v

Halaju sifar
Zero velocity

0 t 0 t

s
v

Halaju negatif U
U
0 t N
Negative velocity N
II
0 t T
T
12

s v a

Halaju seragam
Uniform velocity
0 t 0 t 0 t

s a
v

Pecutan seragam
Uniform acceleration
0 t t
0 t 0

v a
s
t
Nyahpecutan seragam 0
Uniform deceleration
0 t
0 t

25 © Nilam Publication Sdn. Bhd.


MODUL • Fizik TINGKATAN 4

SP 2.2.5 Menyelesaikan masalah melibatkan graf gerakan linear

Latihan / Exercises
Sesaran / Displacement, 1 Hitungkan / Calculate
s/m
(i) halaju bagi OP, QR dan RS
velocity of OP, QR and RS
P Q
20 (ii) sesaran / displacement
Penyelesaian / Solution:
10
(i) OP: halaju / velocity = 20 m = 10 m s–1
2s
O R QR: halaju / velocity = – 20 m = –10 m s–1
0 1 2 3 4 5 6 7 8 t/s 2s
RS: halaju / velocity = – m = –10 m s–1
10
–10 1s
S (ii) s = (20 + 0 – 20 – 10) m = –10 m

U
N Halaju / m s-¹ 2 Hitungkan / Calculate
I
T
Velocity / m s-¹ (i) pecutan bagi OA dan BC
2 acceleration of OA and BC
(ii) jumlah sesaran / total displacement
A B
10
Penyelesaian / Solution:
–1
5 (i) OA: pecutan = 10 m s = 1 m s–2
acceleration 10 s
–1
O C BC: pecutan = – m s = –2 m s–2
10
t/s acceleration 5s
0
(ii) Jumlah sesaran = 1 (25 + 10)s (10 m s–1)
5 10 15 20 25

Total displacement 2
= 175 m

Kesimpulan / Conclusion
1 Kecerunan graf s melawan t memberikan halaju suatu objek.
Gradient of the graph s against t gives the velocity of an object.

2 Kecerunan graf, v melawan t memberikan pecutan suatu objek.
Gradient of the graph v against t gives the acceleration of an object.

3 Luas di bawah graf v melawan t memberikan sesaran yang dilalui oleh objek.
Area under the graph v against t gives the displacement travelled by the object.

© Nilam Publication Sdn. Bhd. 26


MODUL • Fizik TINGKATAN 4

2.3 GERAKAN JATUH BEBAS / FREE FALL MOTION SK 2.3

SP 2.3.1 Menjelaskan gerakan jatuh bebas dan pecutan graviti melalui contoh

Apakah jatuh bebas?


What is free fall?
• Objek dikatakan 'jatuh bebas' apabila ia jatuh di bawah kekuatan medan graviti sahaja.
An object is free falls when it is falls under the gravitational field strength only.
• Sehelai kertas tidak jatuh bebas kerana kejatuhannya dipengaruhi oleh rintangan udara.
A piece of paper does not fall freely because its fall is affected by air resistance.
• Objek hanya jatuh bebas di dalam vakum . Ketiadaan udara bermaksud tiada rintangan udara yang
menentang pergerakan objek.
An object falls freely only in vacuum . The absence of air means there is no air resistance to resist the motion
of the object.
• Di dalam vakum, kedua-dua objek yang ringan dan berat jatuh bebas. Ia jatuh dengan pecutan graviti
iaitu pecutan disebabkan oleh graviti, g.
In vacuum, both light and heavy objects fall freely. They fall with the gravitational acceleration, that is the acceleration
due to gravity , g. U
U
N
N
II
T
T
12
Contoh / Examples

1 Yang mana satukah akan mencecah tanah dahulu? / Which one will reach the ground first?

Bola golf Kertas Bola golf Kertas yang direnyukkan


Golf ball Paper Golf ball Paper which is crumpled

(a) Bola golf dan sehelai kertas dipegang pada (b) Ulangi dengan bola golf dan sehelai kertas yang
ketinggian yang sama dan dijatuhkan serentak. direnyukkan. / Repeat with a golf ball and a piece of
Hold a golf ball and a piece of paper at the same paper which is crumpled.
height and drop them simultaneously. (i) Objek yang manakah mencecah tanah dahulu?
(i) Objek yang manakah mencecah tanah Which object reaches the floor first?
dahulu? Kedua-duanya mencecah tanah pada masa
Which object reaches the floor first? yang sama.
Bola golf. / The golf ball.
Both reach the floor at the same time.

(ii) Terangkan mengapa. / Explain why. (ii) Terangkan mengapa. / Explain why.
Kertas mempunyai luas permukaan yang Kedua-dua objek mempunyai saiz dan luas
besar. Jadi lebih banyak rintangan udara permukaan yang sama. Jisim tidak memberi

yang bertindak ke atasnya. kesan kepada pecutan graviti. / Both objects


have same size and surface area. Mass does not
The paper has large surface area. As such, the
affect gravitational acceleration.
air resistance acting on it is greater.

27 © Nilam Publication Sdn. Bhd.


MODUL • Fizik TINGKATAN 4

2 Perbezaan antara jatuh bebas di atmosfera (udara) dan jatuh bebas di dalam vakum bagi duit syiling dan bulu
pelepah. / The difference between free fall in atmosphere and free fall in a vacuum of a coin and a feather.

Bulu pelepah
Feather
Duit syiling Bulu pelepah
Coin Feather
Duit syiling
Coin

(a) (b)

(a) Duit syiling dan bulu pelepah dilepaskan dari ketinggian yang sama secara serentak di dalam makmal.
A coin and a feather are released from the same height simultaneously in the laboratory.
Pemerhatian / Observation
Duit syiling jatuh lebih cepat daripada bulu pelepah. / The coin falls faster than the feather.
U
N
I Penjelasan bagi (a) / Explanation on (a)
T
Rintangan udara yang besar bertindak ke atas bulu pelepah kerana ia mempunyai luas permukaan
2
yang besar. / A bigger air resistance acts on the feather because it has a large surface area .

Daya graviti pada duit syiling mampu untuk mengatasi rintangan udara lebih baik daripada bulu
pelepah.
The gravitational force on the coin is able to overcome air resistance better than the feather.

(b) Duit syiling dan bulu pelepah yang sama diletakkan di dalam satu tiub vakum dan kemudian dijatuhkan
serentak pada ketinggian yang sama.
The same coin and feather are put into a vacuum tube and then dropped simultaneously from the same height.
Pemerhatian / Observation
Kedua-dua objek mencecah ke bawah silinder pada masa yang sama.
Both objects reach the bottom of the cylinder at the same time.

Penjelasan bagi (b) / Explanation on (b)


Di dalam keadaan vakum, tiada rintangan udara . Hanya terdapat satu daya yang bertindak ke atas objek

iaitu daya graviti .

In vacuum, there is no air resistance . The only force acting on both objects is the force of gravity .

Kedua-dua objek jatuh bebas dengan pecutan yang disebabkan graviti walaupun berbeza dari segi
jisim dan bentuk .
Both objects free fall with acceleration due to gravity despite the differences in their mass and
shapes .

© Nilam Publication Sdn. Bhd. 28


MODUL • Fizik TINGKATAN 4

Terangkan pecutan yang disebabkan oleh graviti, g / Explain acceleration due to gravity, g

Pecutan disebabkan oleh graviti, g, ialah pecutan bagi objek yang disebabkan oleh kekuatan medan graviti .
Acceleration due to gravity, g, is the acceleration of an object due to the pull of the gravitional field strength .
-2 –2
Nilai piawai bagi pecutan graviti, g, ialah 9.81 m s . Nilai g yang sering digunakan ialah 10 m s . Magnitud
bagi pecutan yang disebabkan oleh graviti bergantung pada kekuatan medan graviti .
The standard value of the gravitational acceleration, g, is 9.81 m s-2. The value of g is often taken to be 10 m s–2 for simplicity.
The magnitude of the acceleration due to gravity depends on the gravitional field strength .

Contoh / Examples

1 Rajah di sebelah menunjukkan gambar foto stroboskop bagi bola yang jatuh bebas Bola
Ball
dan graf halaju, v melawan masa, t bagi gerakannya. / The diagram on the right shows
a stroboscopic photograph of a free falling ball and a graph of velocity, v against time, t.
(a) Perhatikan gambar foto dan terangkan halaju bola.
Observe the photograph and describe the velocity of the ball.
Halaju bola itu meningkat dengan seragam U
U
N
N
The velocity of the ball increases uniformly Lantai II
T
T
Floor
(b) Apakah yang boleh anda simpulkan daripada graf kecerunan v – t? 12
What can you deduce from the gradient of the v – t graph? v
Kecerunan ialah pecutan bola itu
The gradient is the acceleration of the ball

(c) Terangkan gerakan bola tersebut. 0 t


Describe the motion of the ball.
Bola tersebut bergerak dengan pecutan seragam
The ball moves with uniform acceleration

2 Pegang dua biji batu yang berbeza saiz pada ketinggian yang sama, kemudian kedua-dua batu itu dijatuhkan
serentak daripada ketinggian yang sama.
Hold two stones of different sizes at the same height and then drop both stones simultaneously from the same height.
(a) Huraikan bagaimana halaju berubah. / Describe how the velocity changes.
Halaju meningkat dengan seragam.
The velocity increases uniformly.

(b) Bandingkan masa yang diambil untuk batu mencecah lantai.


Compare the time taken for the stones to reach the floor.
Sama / Same

(c) Adakah pecutan batu dipengaruhi oleh jisimnya?


Is the acceleration of each stone influenced by its mass?
Jisim tidak mempengaruhi pecutan.
Mass does not affect the acceleration.

29 © Nilam Publication Sdn. Bhd.


MODUL • Fizik TINGKATAN 4

SP 2.3.2 Mengeksperimen untuk menentukan nilai pecutan graviti

Eksperimen Pecutan yang disebabkan graviti Inkuiri / Inquiry


Experiment Acceleration due to gravity

Tujuan / Aim
Menentukan pecutan disebabkan graviti. / To determine the acceleration due to gravity.

Radas / Apparatus
Jangka masa detik, bekalan kuasa 12 V, bangku, pengapit-G, pemberat, pita detik
Ticker timer, 12 V ac power supply, stool, G-clamp, slotted weight, ticker tape.

Prosedur / Procedure
Pita detik
1 Potong sekeping pita detik lebih kurang 2.5 m panjang dan lalukan Ticker tape
Bekalan Pengapit-G
melalui jangka masa detik yang diapit kepada kerusi oleh pengapit-G. kuasa, 12 V G-clamp
Cut a piece of ticker tape about 2.5 m long and pass through the ticker A.C. Power Jangka masa
timer which is clamped to a stool using G-clamp. supply, 12 V detik
2 Sambungkan satu hujung pita pada pemberat 100 g. Ticker timer
Attach one end of the tape to the 100 g slotted weight. Bangku
3 Hidupkan jangka masa detik dan pemberat dilepaskan supaya ia jatuh Stool

U
bebas.
N Switch on the ticker timer and release the slotted weight so that it falls freely. Pemberat
I 4 Kaji pita itu untuk menentukan nilai bagi pecutan disebabkan oleh Weight
T
graviti, g. Kepingan polistirena
2 Analyse the tape to determine the value of the acceleration due to gravity, g.
Polystyrene sheet

Perbincangan / Discussion
1 Mengapakah sukar untuk menentukan pergerakan objek yang jatuh dengan hanya memerhatikannya jatuh?
Why is it difficult to describe the motion of a falling object by just observing it fall?
Objek bergerak sangat laju. / The object moves very fast.

2 Apakah jenis pergerakan objek jika ia jatuh di bawah tarikan graviti?


What is the type of motion of an object falling under the pull of gravity?
Pecutan seragam. / Uniform acceleration.

3 Mengapakah pergerakan pemberat boleh diandaikan sebagai jatuh bebas?


Why is it that the motion of the slotted weight can be assumed to be a free fall?
Rintangan udara yang kecil boleh diabaikan.
The small air resistance is negligible.

4 Apakah langkah yang akan anda ambil untuk mengurangkan geseran antara pita dan jangka masa detik?
What steps did you take to minimise the friction between the ticker tape and the ticker timer?
Pegang pita detik dalam keadaan menegak dan lepaskannya. Pastikan ia jatuh melalui jangka masa detik
dengan lancar.
Hold the ticker tape vertically when releasing it. Make sure it slips through the ticker timer smoothly.

5 Terangkan mengapa perlu menjatuhkan pemberat daripada kedudukan yang tinggi.


Explain the need for the slotted weight to be dropped from a high position.
Pengiraan akan menjadi lebih tepat kerana ralat eksperimen dikurangkan.
The calculation will be more accurate because experimental errors are reduced.

© Nilam Publication Sdn. Bhd. 30


MODUL • Fizik TINGKATAN 4

6 Tunjukkan bagaimana anda mengira g daripada pita.


Show how you would calculate g from the tape.

s1 s2
u= t v= t
1 2
v–u
a= t

7 Bandingkan nilai pecutan disebabkan oleh graviti daripada aktiviti ini dengan nilai yang sebenar. Berikan
alasan yang munasabah bagi perbezaan di antara dua nilai tersebut.
Compare the value of the acceleration due to gravity from this activity with the actual value. Give possible reasons for
any difference in these two values.
Nilai daripada eksperimen adalah lebih rendah berbanding dengan nilai sebenar. Alasannya ialah rintangan
yang disebabkan oleh jangka masa detik.
The value from the experiment is lower than the actual value. The reason is the resistance caused by the ticker timer.

8 Bandingkan nilai bagi g daripada eksperimen jika anda mengulangi eksperimen dengan menggunakan pemberat
200 g dan 300 g.
Compare the values of g from the experiment if you repeat the experiment using 200 g and 300 g weights. U
U
N
N
Keputusan sepatutnya sama. II
T
T
The result should be the same.
12
9 Apakah yang boleh anda simpulkan tentang hubungan antara g dan jisim bagi objek yang jatuh?
What can you conclude about the relationship between g and the mass of the falling object?
Jisim tidak mempengaruhi pecutan graviti, g.
Mass does not affect the gravitational acceleration, g.

SP 2.3.3 Menyelesaikan masalah yang melibatkan pecutan graviti bumi bagi objek yang jatuh bebas

Formula Penting / Important Formulae

Untuk objek yang jatuh dengan pecutan, g, berikut adalah persamaan-persamaan yang berkaitan:
For an object falling with acceleration g, the following equations apply:
1 v = u + gt di mana / where s = sesaran / displacement titik tertinggi, v = 0
2
2 s = ut + ½ gt u = halaju awal / initial velocity highest point, v = 0
3 s = ½ (u + v)t v = halaju akhir / final velocity
4 v2 = u2 + 2gs t = masa / time
a = g, pecutan graviti / acceleration due to gravity,
Nota / Notes:
1 Apabila suatu objek jatuh bebas: a = g = 10 m s-2 (pecutan)
When an object fall freely: a = g = 10 m s-2 (acceleration)
2 Apabila suatu objek dilambung ke atas: a = –g = –10 m s-2 (nyahpecutan)
When an object is thrown upwards: a = –g = –10 m s-2 (deceleration)
3 Pada kedudukan yang tertinggi, v = 0.
At the highest point, v = 0. Objek dilambung
4 Jatuh ke bawah, v adalah positif. / Downward direction, v is positive. ke atas
Object thrown
5 Arah ke atas, v adalah negatif. / Upward direction, v is negative. upwards

31 © Nilam Publication Sdn. Bhd.


MODUL • Fizik TINGKATAN 4

Latihan / Exercisess
Andaikan g = 10 m s–2 dan tiada rintangan udara. Andaikan nilai g = 10 m s–2.
–2
Assume g = 10 m s and there is no air resistance. Assume the value of g = 10 m s–2.

1 Sebiji batu jatuh daripada 3 Amir menjatuhkan batu ke dalam perigi. Jika jarak
ketinggian 45 m. / A rock falls antara bahagian atas perigi dan permukaan air ialah
from a height of 45 m. 20 m,
(a) Berapa lamakah masa Amir releases a stone into a well. If the distance between
the top of the well and the water surface is 20 m,
yang diambil oleh batu
itu untuk mencecah ke g (a) berapakah masa yang diambil oleh batu itu
h t
tanah? / How long does it untuk sampai ke permukaan air?
take to reach the ground? what is the time required for the stone to reach the
(b) Berapakah halaju surface of the water?
batu itu semasa ia (b) berapakah halaju batu itu apabila ia terkena
menghentam lantai? permukaan air?
What is its velocity as it hits the ground? what is the velocity of the stone when it strikes the
surface of the water?
Penyelesaian / Solution:
(a) u = 0, s = 45 m, Penyelesaian / Solution:
g = 10 m s–2, t = ? (a) u = 0, s = 20 m , g = 10 m s–2 , t = ?
U
1 1
N s = ut + 2 gt2 s = ut + 2 gt2
I
T 1 1
20 m = 0 + 2 (10 m s–2)(t2)
2 45 m = 0 + 2 (10 m s–2)(t2)
t2 = 4 s2
t2 = 9 s2
t = 2 s
t = 3 s
(b) v2 = u2 + 2gs
(b) v = u + gt
v2 = 0 + 2(10 m s–2)(20 m)
= 0 + (10 m s–2)(3 s)
∴v = 20 m s–1
= 30 m s–1

2 Sebiji bola dilambung 4 Suatu objek yang berjisim 5 kg dilepaskan dari
v=0
ke atas daripada tanah sebuah bangunan setinggi 500 m. Berapakah
–1
dengan halaju 30 m s . An object of mass 5 kg is released from a tall building
Selepas beberapa lamakah g = –10 m s–2 of height 500 m. What is the
bola itu akan menyentuh (a) berat objek itu? / weight of the object?
tanah semula? u = 30 m s–1 (b) kekuatan medan graviti? / gravitational field
A ball is thrown upwards
strength?
from the ground with a
(c) masa yang diambil untuk sampai ke tanah?
velocity of 30 m s–1. After
how many seconds will it time taken to reach the ground?
strike the ground again? Penyelesaian / Solution:
(a) W = 5 kg × 10 m s–2
Penyelesaian / Solution: = 50 N
Untuk gerakan ke atas,/For the upward motion, (b) g = 10 N kg–1 atau / or 10 m s–2
u = 30 m s–1, v = 0, g = –10 m s–2, (c) u = 0, s = 500 m, g = 10 m s–2
v = u + gt 1
∴0 = 30 m s–1 + (–10 m s–2)(t) s = ut + 2 gt2
10t = 30 s 1
500 m = 0 + 2 (10 m s–2)(t2)
t = 3 s (gerakan ke atas / upward motion)
Maka, masa untuk gerakan ke bawah juga mengambil t2 = 100 s2
3 s. t = 10 s
Oleh itu, ia mengambil masa 6 saat.
The time taken for the downward motion is also 3 s.
So it takes a total of 6 seconds.

© Nilam Publication Sdn. Bhd. 32


MODUL • Fizik TINGKATAN 4

2.4 INERSIA / INERTIA SK 2.4

SP 2.4.1 Menerangkan konsep inersia melalui contoh

• Inersia suatu objek ialah kecenderungan objek untuk kekal dalam keadaan rehat atau
terus bergerak dalam keadaan gerakannya.
Berikan maksud
The inertia of an object is the tendency of the object to remain at rest or if moving to continue
inersia.
its motion.
Give the definition
of inertia.
• Inersia bukan suatu kuantiti fizik.
Inertia is not a physical quantity.

Apakah Hukum • Menyatakan bahawa sesuatu objek akan kekal dalam keadaan pegun atau bergerak
Gerakan Newton dengan halaju malar jika tiada daya luar bertindak ke atasnya.
Pertama? State that an object continue in its state of rest or uniform speed if no
What is the external
force acted upon it.
Newton’s First
Law of Motion?
U
U
N
N
Aktiviti yang melibatkan inersia / Activities involving inertia II
T
T

Syiling Kadbod 12
Coin Cardboard Apabila kadbod ditarik keluar dengan cepat, duit syiling itu terus
jatuh ke dalam gelas. Inersia duit syiling itu mengekalkannya
dalam keadaan rehat walaupun kadbod itu ditarik keluar.
When the cardboard is quickly pulled away, the coin drops straight
into the glass. The inertia of the coin maintains it in rest even when the
cardboard is withdrawn.

Gelas berisi air


Glass filled with water Apabila kertas di bawah gelas itu ditarik dengan sangat cepat,
Kertas
Paper gelas air itu kekal dalam keadaan rehat. Inersia gelas yang berisi
air itu cenderung mengekalkan gelas air dalam keadaan rehat.
When the paper at the bottom of the glass is pulled very quickly, the
glass of water remains at rest. The inertia of the glass of water tends
to remain at rest.

Buku Sebuah buku ditarik keluar dari kedudukan tengahnya. Buku di


Books
atasnya akan jatuh ke bawah secara terus. Inersia cuba menentang
perubahannya dari keadaan rehat, iaitu, apabila buku ditarik keluar,
buku-buku di atas tidak akan bergerak bersamanya.
A book is pulled out from its central position. The books on top will drop
straight downwards. Inertia tries to resist the change from rest, that is,
when the book is pulled out, the books on top not follow suit.

33 © Nilam Publication Sdn. Bhd.


MODUL • Fizik TINGKATAN 4

SP 2.4.2 Mengeksperimen untuk mengenal pasti hubungan antara inersia dan jisim

Eksperimen Inersia dan Jisim


Experiment Inertia and Mass

Inferens Inersia bergantung kepada jisim objek


Inference
The inertia depends on mass of object

Hipotesis Apabila jisim objek bertambah, inersianya juga bertambah.


Hypothesis
When the mass of an object increases, its inertia also increases.

Tujuan Untuk mengkaji hubungan antara jisim dengan inersia (tempoh ayunan).
Aim To study the relationship between mass and inertia (period of oscillation).
Pemboleh ubah dimanipulasikan / Manipulated variable:
Jisim plastisin, m
Mass of plasticine, m
Pemboleh ubah bergerak balas / Responding variable:
U Pemboleh ubah Tempoh ayunan, T
N Variables
I
T Period of oscillation, T
2 Pemboleh ubah dimalarkan / Constant variable:
Panjang bilah Hacksaw / amplitud ayunan / kekerasan bilah Hacksaw
Length of the Hacksaw blade / amplitude of oscillation / stiffness of Hacksaw blade

Bahan dan radas Bilah gergaji, pengapit-G, jam randik dan plastisin.
Materials and
apparatus Hacksaw blade, G-clamp, stopwatch and plasticine.

1 Letakkan sejumlah plastisin (berbentuk sfera)


dengan jisim 30 g pada hujung bilah Hacksaw.
Pengapit-G / G-clamp
Place a lump of plasticine (sphere-shaped) with a mass
of 30 g at the free end of the Hacksaw blade.
Plastisin / Plasticine
2 Sesarkan sedikit bilah Hacksaw dan lepaskannya Bilah Hacksaw / Hacksaw blade
supaya ia berayun secara mengufuk.
Displace the Hacksaw blade slightly and release it so
that it oscillates horizontally.

Prosedur 3 Tentukan dan rekodkan masa yang diambil untuk 10 ayunan lengkap, t saat.
Procedure Determine and record the time taken for 10 complete oscillations, t seconds.
t
4 Hitungkan tempoh ayunan, T = saat.
10
t
Calculate period of oscillation, T = seconds.
10
5 Ulangi langkah 1 – 4 eksperimen dengan jisim 40 g, 50 g, 60 g dan 70 g.
Repeat steps 1 – 4 of the experiment with mass of 40 g, 50 g, 60 g and 70 g.
6 Lakarkan graf tempoh ayunan melawan jisim.
Plot the graph of period of oscillation against mass.

© Nilam Publication Sdn. Bhd. 34


MODUL • Fizik TINGKATAN 4

Masa untuk 10 ayunan, t / s Tempoh ayunan


Jisim, m / g Time for 10 oscillation, t / s Period of osillation
Mass, m / g
t1 t2 tpurata / average T/s
30
Keputusan
Results 40
50
60
70

Lakarkan graf T melawan jisim, m.


Plot the graph T against mass, m.
T/s

Analisis
Analysis

0 m/g U
U
N
N
II
T
T
1 Nyatakan kuantiti yang digunakan untuk mewakili inersia dalam aktiviti ini.
State the quantity used to represent inertia in this activity. 12
Tempoh ayunan

Period of oscillation

2 Apakah hubungan antara tempoh ayunan suatu objek dengan inersianya?


What is the relationship between the period of oscillation of an object and its inertia?
Semakin panjang tempoh ayunan, semakin besar inersia.
The longer the period of oscillation, the larger the inertia.

Perbincangan 3 Daripada graf, nyatakan hubungan antara


Discussion From the graph, state the relationship between
(a) tempoh ayunan dengan jisim objek.
period of oscillation and mass of object.
Semakin besar jisim, semakin panjang tempoh ayunan.
The larger the mass, the longer the period of oscillation.
(b) inersia suatu objek dan jisimnya.
inertia of an object and its mass.
Semakin besar jisim objek, semakin besar inersianya.
The larger the mass of the object, the larger its inertia.

Kesimpulan Apabila jisim objek bertambah, inersianya juga bertambah.


Conclusion
When the mass of an object increases, its inertia also increases.

35 © Nilam Publication Sdn. Bhd.


MODUL • Fizik TINGKATAN 4

SP 2.4.3 Mewajarkan kesan inersia dalam kehidupan seharian

Contoh-contoh situasi yang melibatkan inersia / Examples of situations involving inertia

Apabila bas itu bergerak ke hadapan secara tiba-tiba dari keadaan rehat,
inersia badan penumpang cenderung untuk kekal dalam keadaan rehat.
Ini menyebabkan badan penumpang terhumban ke belakang.
When the bus suddenly moves forward from rest, the inertia of the passenger's
body tends to keep them at rest. This causes their body to be thrown backwards.
Bas bergerak ke hadapan
secara tiba-tiba
The bus moves suddenly forward

Penumpang berada dalam keadaan gerakan apabila bas itu sedang bergerak.
Apabila bas itu berhenti secara tiba-tiba, inersia badan penumpang
cenderung untuk terus bergerak ke hadapan. Ini menyebabkan badan
penumpang terhumban ke hadapan.
The passengers are in a state of motion when the bus is moving. When the bus
Bas berhenti secara tiba-tiba suddenly stops, the inertia of the passengers tends to continue in its forward
The bus suddenly stops motion. This causes their body to be thrown forward.
Sos cili dalam botol boleh dituang keluar dengan senang jika botol
U digerakkan ke bawah dengan cepat dan berhenti secara tiba-tiba.
N
I Gerakan ke bawah yang cepat Chili sauce in the bottle can be easily poured out if the bottle is moved downward
T Fast downward motion fast with a sudden stop.
2 • Sos dalam botol bergerak bersama-sama dengan botol semasa
Botol sos cili pergerakan ke bawah.
Bottle of chili The sauce in the bottle moves with the bottle during the downward movement.
sauce
• Apabila botol itu berhenti secara tiba-tiba, inersia sos menyebabkan
ia terus bergerak ke bawah dan mengakibatkan sos dituang keluar
dari botol itu.
When the bottle is stopped suddenly, the inertia of the sauce causes it to
continue in its downward movement. Thus, the sauce is poured out of the
bottle.
• Titisan air pada payung yang basah akan jatuh apabila budak perempuan
itu memusingkan payung itu.
The water droplets on a wet umbrella will fall when the girl rotates the
umbrella.
• Ini adalah disebabkan titisan air pada permukaan payung itu bergerak
secara serentak apabila payung itu dipusingkan.
This is because the water droplets on the surface of the umbrella move
simultaneously as the umbrella is rotated.
• Apabila payung itu berhenti berpusing, inersia titisan air akan terus
mengekalkan pergerakannya.
When the umbrella stops rotating, the inertia of the water droplets will
continue in its original motion.

Sebuah kapal minyak yang besar mengambil masa yang lebih panjang untuk memecut kepada laju maksimumnya
dan ia mengambil beberapa kilometer untuk berhenti walaupun propelernya telah diterbalikkan. Mengapa?
A massive oil tanker (a very big ship) takes a long time to accelerate to its full speed and a few kilometers to come to a
stop even though the engine has reversed its propeller to slow it down. Why?
Kapal minyak yang besar mempunyai jisim yang lebih besar, jadi inersianya juga lebih besar. Oleh itu,
ia adalah lebih sukar untuk memberhentikan kapal minyak.
The massive oil tanker has larger mass, so it has a larger inertia. So it is more difficult to stop the oil tanker.

© Nilam Publication Sdn. Bhd. 36


MODUL • Fizik TINGKATAN 4

Kesan baik dan buruk inersia / Good and negative effect of inertia

Peta Pemikiran / Thinking Maps


Inersia (Punca dan Kesan)/Inertia (Cause and Effect)
Inersia menjadi besar
Jisim objek besar Inertia becomes large
Larger mass of object 1 1

Inersia Objek yang berada dalam


Inertia 2 keadaan rehat akan cenderung
Hukum Gerakan 2 kekal dalam keadaan rehat
Newton Pertama Object at rest will tend to
Newton's first Law
remain at rest
of Motion 2

Objek yang berada dalam keadaan gerakan cenderung


untuk kekal dalam keadaan gerakannya
An object in motion tends to remain in motion

Kaedah mengurangkan kesan buruk inersia / Method to reduce the negative effects of inertia
U
U
N
N
1 Tali pinggang keledar di dalam kereta dapat mengekalkan II
T
pemandu pada tempat duduknya. Apabila kereta berhenti secara T
mendadak, tali pinggang itu mengelakkan pemandu daripada 12
terhumban ke hadapan .
A safety belt in a car secures a driver to his seat. When the car
stops suddenly, the seat belt prevents the driver from being thrown
forward .

2 Perabot yang dibawa oleh lori biasanya perlu diikat dengan tali Tali
kepada bahagian-bahagian lori yang tertentu supaya apabila Rope
lori bergerak atau berhenti dengan tiba-tiba, perabot itu tidak
akan jatuh atau tidak akan terhumban ke hadapan.
Furniture carried by a lorry is normally tied by ropes to certain fixed
parts of the lorry so that when the lorry moves or stops suddenly, the
furniture will not fall or will not be thrown forward.
Lori
Lorry
3 Lori tangki mempunyai empat tangki kecil di mana jisim
muatan dibahagi antara tangki-tangki tersebut akan mempunyai Kepala lori
inersia yang lebih kecil. Ini akan mengurangkan impak pada Tractor
setiap tangki yang disebabkan oleh inersia jika lori tangki itu
berhenti dengan tiba-tiba.
Four small tanks with distributed mass will have smaller inertia. This
will greatly reduce the inertial impact on each tank if the tanker stops
Treler dengan 4
suddenly. tangki kecil
Lori tangki Trailer with 4 small
Tanker tanks
Maklumat tambahan:
Additional information:

37 © Nilam Publication Sdn. Bhd.


MODUL • Fizik TINGKATAN 4

2.5 MOMENTUM / MOMENTUM SK 2.5

SP 2.5.1 Menerangkan momentum, p sebagai hasil darab jisim, m dan halaju, v. (p = mv)

• Momentum adalah hasil darab jisim dan halaju.


Momentum is the product of mass and velocity.
• Momentum, p dihitung menggunakan rumus berikut:
Berikan definisi Momentum, p can be calculated using the following formula:
momentum. p = mv : p = momentum / momentum
Define momentum. m = jisim / mass
Apakah unit S.I. bagi v = halaju / velocity
momentum? • Unit S.I: kg m s–1 atau N s (Newton saat)
What is the S.I. unit for
momentum? S.I unit: kg m s–1 or N s (Newton second)

• Momentum adalah suatu kuantiti vektor. Arah momentum mengikut arah halaju.
Momentum is a vector quantity. The direction of the momentum follows the direction
of the velocity.

U
N
I
T Contoh / Examples
2
1 Dalam permainan ragbi, seorang pemain berjisim 70 kg bergerak Pemain 1 Pemain 2
dengan halaju 4 m s-1 dan seorang pemain yang lain yang berjisim Player 1 Player 2
75 kg bergerak dengan 3 m s-1 menghala antara satu sama lain
seperti yang ditunjukkan. Hitungkan momentum kedua-dua pemain
itu masing-masing.
In a rugby game, a player of mass 70 kg is moving with velocity of 4 m s-1
and the other player of mass 75 kg is moving with 3 m s-1 towards each
other as shown. Calculate the momentum of the two players respectively.

Penyelesaian / Solution:
Momentum pemain 1 / Momentum of player 1, p1 = m1v1 = (70 kg)(4 m s–1) = 280 kg m s-1
Momentum pemain 2 / Momentum of player 2, p2 = m2v2 = (75 kg)(–3 m s–1) = –225 kg m s-1

2 Seorang nenek (m = 80 kg) meluncur mengelilingi gelanggang dengan


halaju 6 m s–1. Tiba-tiba dia berlanggar dengan Bobby (m = 40 kg) Nenek Bobby
Granny
yang berada dalam keadaan rehat. Hitungkan momentum bagi nenek
dan Bobby.
A granny (m = 80 kg) skating around the ring with a velocity of 6 m s–1.
Suddenly she collides with Bobby (m = 40 kg) who is at rest. Calculate the
momentum of granny and Bobby respectively.

Penyelesaian / Solution:
Momentum nenek / Momentum of granny, p1 = m1v1 = (80 kg)(6 m s–1) = 480 kg m s–1
Momentum Bobby / Momentum of Bobby, p2 = m2v2 = (40 kg) × (0 m s–1)
= 0 kg m s–1 (dalam keadaan rehat / at rest)

© Nilam Publication Sdn. Bhd. 38


MODUL • Fizik TINGKATAN 4

SP 2.5.2 Mengaplikasi Prinsip Keabadian Momentum dalam perlanggaran dan letupan

Berikan • Tanpa kehadiran daya luar, jumlah momentum dalam suatu sistem kekal tidak berubah.
definisi Prinsip In the absence of an external force, the total momentum of a system remains unchanged.
Keabadian
Momentum. • Jumlah momentum sebelum perlanggaran = Jumlah momentum selepas perlanggaran
Total momentum before collision = Total momentum after collision
Give the definition
of Principle of
Conservation of
Momentum.

Situasi-situasi yang melibatkan Prinsip Keabadian Momentum


Situations that involved the Principle of Conservation of Momentum

1 Rajah di bawah menunjukkan dua orang adik-beradik yang sedang meluncur. Abang bergerak dan berlanggar
dengan adiknya yang berada dalam keadaan rehat.
The diagram below shows two brothers are skating. The elder brother moves and collides with his younger brother who
is at rest.
(a) Apakah gerakan mereka selepas perlanggaran? / What is their movement after the collision?
Selepas perlanggaran, laju abang berkurang, laju adik bertambah. Momentum abang berkurang,
momentum adik bertambah. U
U
N
N
After collison, the speed of the elder brother decreases, the speed of the younger brother II
increases, momentum of the elder brother decreases, momentum of the younger brother T
T
increases. 12
(b) Adakah jumlah momentum sebelum perlanggaran sama dengan jumlah momentum
selepas perlanggaran?
Is the total momentum before collision equal to the total momentum after collision?
Ya / Yes

2 Rajah di bawah menunjukkan Aziz menjentik sekeping duit syiling, A secara terus kepada sekeping, B.
The diagram below shows Aziz is flicking a coin, A, directly to coin, B.

A B
(a) Apakah yang berlaku kepada gerakan kedua-dua duit syiling tersebut selepas perlanggaran?
What happens to the motion of both coins after collision?
Duit syiling A berhenti, duit syiling B bergerak.
Coin A stops, coin B moves.

(b) Apakah yang berlaku kepada momentum duit syiling A selepas perlanggaran?
What happens to the momentum of coin A after collision?
Momentum duit syiling A dipindahkan kepada duit syiling B selepas perlanggaran.
Momentum of coin A is transferred to coin B after collision.

39 © Nilam Publication Sdn. Bhd.


MODUL • Fizik TINGKATAN 4

3 Rajah di bawah menunjukkan sebiji bola keluli, E ditarik dan dilepaskan.


The diagram below shows a steel ball, E is pulled and released.

(a) Bola itu akan berlanggar dengan empat biji bola yang lain. Ini akan menyebabkan bola, A bergerak ke
ketinggian yang sama dengan ketinggian bola E dilepaskan.
The ball will collide with the other four balls. This will cause the ball, A to move
to the same height as ball E is released.
(b) Adakah momentum diabadikan? / Is the momentum conserved?
Ya / Yes A B C D E

(c) Apakah yang akan berlaku jika kedua-dua bola D dan E ditarik dan kemudian dilepaskan?
What will happen if two balls D and E are pulled and then released?
Bola A dan B akan bergerak ke ketinggian yang sama dengan bola D dan E masing-masing. Bola C akan
berada dalam keadaan rehat.
Balls A and B will move rise to the same heights of balls D and E respectively. Ball C is at rest.

Perlanggaran kenyal Perlanggaran tak kenyal


Elastic collision Inelastic collision
U
N vv
I u1 u1 u2 u2 v1 v1 v2 v2 u1 u1 u2u2
m1m1+ +m2m2
T

2 m1 m1 m2 m2 m1 m1 m2 m2 m1m1 m2m2 m1m1 m2m2

Sebelum
Sebelum perlanggaran
perlanggaran Selepas
Selepas perlanggaran
perlanggaran
Sebelum
Sebelumperlanggaran
perlanggaran Selepas
Selepas
perlanggaran
perlanggaran Before
Before collision
collision After
After collision
collision
Before
Before collision
collision After
After
collision
collision

• Selepas perlanggaran, kedua-dua objek bergerak • Selepas perlanggaran, kedua-dua objek bergabung
secara berasingan dengan halaju masing-masing. dan bergerak bersama dengan satu halaju sepunya.
After collision, both objects move separately at their After collision, the two objects combine and move together
respective velocities. with a common velocity.
• Jumlah momentum diabadikan. • Jumlah momentum diabadikan.
Total momentum is conserved. Total momentum is conserved.
• Jumlah tenaga diabadikan. • Jumlah tenaga diabadikan.
Total energy is conserved. Total energy is conserved.
• Tenaga kinetik diabadikan. • Tenaga kinetik tidak diabadikan.
Kinetic energy is conserved. Kinetic energy is not conserved.

u1 u2 v1 v2 u1 u2 v

m1 m2 m1 m2 m1 m2 m1 m2
Sebelum perlanggaran Selepas perlanggaran
Sebelum perlanggaran Selepas perlanggaran Before collision After collision
Before collision After collision
Persamaan yang menghubungkaitkan jumlah momentum
Persamaan yang menghubungkaitkan jumlah momentum
sebelum perlanggaran dengan jumlah momentum
sebelum perlanggaran dengan jumlah momentum
selepas perlanggaran:
selepas perlanggaran: Equation which relates the total momentum before collision
Equation which relates the total momentum before collision with the total momentum after collision:
with the total momentum after collision:
m1u1 + m2u2 = (m1 + m2 )v
m1u1 + m2u2 = m1v1 + m2v2

© Nilam Publication Sdn. Bhd. 40


Troli pegun MODUL • Fizik TINGKATAN 4
Stationary trolleys

Letupan Pin
Explosion

m1 m2
Troli pegun
Stationary trolleys
Sebelum letupan / Before explosion
Pin v1 v2

m1 m2 m1 m2

Sebelum letupan / Before explosion Selepas letupan / After explosion


v1 v2
• Sebelum letupan, kedua-dua objek bercantum bersama dan berada dalam keadaan rehat. Selepas letupan,
kedua-dua objek bergerak pada
m1 arah yang
m2 bertentangan.
Before explosion, both of the objects stick together and are at rest. After explosion, both objects move at opposite
directions. Selepas letupan / After explosion
• Jumlah momentum sebelum letupan adalah sifar.
The total momentum before explosion is zero.
Jumlah momentum selepas letupan
Total momentum after explosion
0 = m1v1 + m2v2 U
U
N
N
• Daripada prinsip keabadian momentum: II
From the principle of conservation of momentum: T
T
12
Jumlah momentum Jumlah momentum selepas
sebelum perlanggaran perlanggaran
Total momentum before collision Total momentum after collision

• Persamaan untuk letupan:


Equation for explosion:
0 = m1v1 + m2v2
m1v1 = –m2v2

Catatan: v2 bernilai negatif kerana arah yang bertentangan


Remark: v2 is in negative value because of opposite direction

Video Video

Perlanggaran Kenyal Perlanggaran tak Kenyal


Elastic Collision Inelastic Collision

41 © Nilam Publication Sdn. Bhd.


MODUL • Fizik TINGKATAN 4

Eksperimen Prinsip Keabadian Momentum


Experiment Principle of Conservation of Momentum

Tujuan Untuk menyiasat Prinsip Keabadian Momentum dalam perlanggaran tak kenyal.
Aim To investigate the Principle of Conservation of Momentum in an inelastic collision.
Jangka masa detik, pita detik, plastisin, pita selofan, troli, landasan, bekalan kuasa a.u. 12 V,
Radas
blok kayu. / Ticker timer, ticker tape, plasticine, cellophane tape, trolleys, runway, 12 V ac power
Apparatus
supply, wooden block.
1 Dirikan satu landasan dengan mengubah suai kecerunannya supaya landasan
terpampas geseran di mana troli boleh bergerak turun landasan dengan halaju malar.
Set up a runway and adjust the slope to compensate for friction where the trolley moves down
the runway with constant velocity.
2 Letakkan plastisin pada troli P dan Q supaya mereka akan melekat antara satu sama
lain semasa perlanggaran.
Fix plasticine on trolleys, P and Q so that they can stick together upon collision.

Jangka masa detik


Pita detik Troli P Landasan
Ticker timer Plastisin
Ticker tape Trolley P terpampas
Plasticine Troli Q
geseran
Trolley Q
Friction-
Bekalan kuasa compensated
U Power supply runway
N
I
T Blok kayu/Wooden block
Prosedur
2 Procedure
3 Pita detik diletakkan melalui jangka masa detik dan dilekatkan pada troli P.
A ticker tape is passed through the ticker timer and is attached to trolley P.
4 Hidupkan jangka masa detik dan tolak troli P dengan kuat supaya ia bergerak
menuruni landasan dan berlanggar dengan troli Q, yang berada dalam keadaan rehat.
/ Switch on the ticker timer and push trolley P hard so that it will move down the runway
and collide with trolley Q, which is at rest.
5 Daripada pita detik yang diperoleh, tentukan dan ukurkan halaju berikut.
From the ticker tape obtained, determine and measure the following velocities.
(a) Halaju troli P sebelum perlanggaran, u1 / Velocity of trolley P before collision, u1
(b) Halaju sepunya troli (P + Q) selepas perlanggaran, v
Common velocity of trolley (P + Q) after collision, v
6 Langkah-langkah 2 – 5 diulangi dengan jisim P dan jisim Q yang berbeza seperti
ditunjukkan dalam jadual di dalam ruang keputusan.
Steps 2 – 5 are repeated for different masses of P and Q as shown in the table of results.

Sebelum perlanggaran Selepas perlanggaran


Before collision After collision
Jumlah Jumlah
Troli P Troli Q momentum Troli (P + Q) momentum
Trolley P Trolley Q Total Trolley (P + Q) Total
momentum momentum
Keputusan m1 v
m1 m2 m2 m1u1 + m1 + m2
(m1 + m2)v
(kg) (cm per 10
Results (cm per detik) (kg) (cm per 10 detik)
m2u2 (kg)
10 ticks) (cm per 10 ticks)
1 1 0 2
2 1 0 3
1 2 0 3
2 2 0 4
3 2 0 5

© Nilam Publication Sdn. Bhd. 42


MODUL • Fizik TINGKATAN 4

1 Bandingkan jumlah momentum sebelum perlanggaran dan selepas perlanggaran.


Compare the total momentum before collision and after collision.
Jumlah momentum sebelum dan selepas perlanggaran adalah sama.
The total momentum before collision and after collision are equal.

2 Nyatakan satu kesimpulan daripada eksperimen. / State a conclusion from the experiment.
Tanpa kehadiran daya luar, jumlah momentum sebelum perlanggaran adalah sama
dengan jumlah momentum selepas perlanggaran.
Perbincangan
Discussions In the absence of any external force, the total momentum before collision is equal to total

momentum after collision.


Maklumat tambahan:
Additional information:
3 Apakah tujuan utama mengubah suai landasan supaya menjadi landasan terpampas
geseran?
What is the main purpose of adjusting the runway so that it is friction-compensated?
Troli bergerak dengan halaju malar.
The trolley moves with constant velocity.
U
U
N
N
II
T
T
Latihan / Exercises
12
1 Kereta A yang berjisim 1 000 kg bergerak pada 2 Sebiji bola yang berjisim 5 kg dibaling pada halaju
20 m s–1 berlanggar dengan kereta B yang berjisim 20 km j–1 kepada Lily yang berjisim 60 kg pada
1 200 kg dan bergerak pada 10 m s–1 dalam arah keadaan rehat di atas ais. Lily menangkap bola
yang sama. Akibatnya, kereta B, bergerak ke itu dan kemudian menggelongsor dengan bola di
hadapan pada 15 m s–1. Berapakah halaju, v, bagi atas ais. Tentukan halaju Lily dengan bola selepas
kereta A sebaik sahaja selepas perlanggaran? perlanggaran.
Car A of mass 1 000 kg moving at 20 m s–1 A 5 kg ball is thrown at a velocity of 20 km h–1 towards
collides with car B of mass 1 200 kg moving at Lily whose mass is 60 kg at rest on ice. Lily catches
10 m s-1 in the same direction. The car B is shunted the ball and subsequently slides with the ball across the
forwards at 15 m s–1 by the impact. What is the ice. Determine the velocity of Lily and the ball together
velocity, v, of car A immediately after the collision? after the collision.
uA = 20 m s-¹ uB = 10 m s-¹ m1 = 5 kg m2 = 60 kg m1 = 5 kg m2 = 60 kg
u1 = 20 km j–1 u2 = 0 km j–1
A B

v=?
mA = 1 000 kg mB = 1 200 kg

Penyelesaian / Solution:
mAuA + mBuB = mAvA + mBvB
Penyelesaian / Solution:
(1 000 kg)(20 m s–1) + (1 200 kg)(10 m s–1) m1u1 + m2u2 = m1v1 + m2v2
= (1 000 kg)v + (1 200 kg)(15 m s–1)
(5 kg)(20 km j–1) + (60 kg)(0 km j–1) = (5 + 60) kg × v
20 000 kg m s–1 + 12 000 kg m s–1
(100 + 0) kg km j–1 = (65 kg)v
= (1 000 kg)(v) + 18 000 kg m s–1
∴ v = 1.54 km j–1
(1 000 kg)(v) = 14 000 kg m s–1
∴ v = 14 m s–1

43 © Nilam Publication Sdn. Bhd.


MODUL • Fizik TINGKATAN 4

3 Sebuah trak yang berjisim 1 200 kg bergerak pada 4 Seorang lelaki menembak menggunakan sepucuk
30 m s–1 berlanggar dengan sebuah kereta yang pistol yang berjisim 1.5 kg. Jika peluru itu berjisim
berjisim 1 000 kg yang bergerak dalam arah 10 g dan mempunyai halaju 300 m s–1 selepas
bertentangan pada 20 m s–1. Selepas perlanggaran, tembakan, berapakah halaju sentakan pistol itu?
kedua-dua kenderaan itu bergerak bersama. A man fires by using a gun which has a mass of 1.5 kg.
Berapakah halaju kedua-dua kenderaan itu sebaik If the mass of the bullet is 10 g and it has a velocity of
sahaja selepas perlanggaran? 300 m s–1 after shooting, what is the recoil velocity of
A truck of mass 1 200 kg moving at 30 m s–1 collides the pistol?
with a car of mass 1 000 kg which is traveling in the v 300 m s-1
opposite direction at 20 m s–1. After the collision, the
two vehicles move together. What is the velocity of both
vehicles immediately after collision? 1.5 kg 10 g
Pegun / Stationary
30 m s-1 20 m s-1 v
Sebelum tembakan Selepas tembakan
Before shooting After shooting

Penyelesaian / Solution:
Sebelum perlanggaran Selepas perlanggaran 0 = m1u1 + m2u2
Before collision After collision m1u1 = – m2u2
Penyelesaian / Solution: (1.5 kg) (v1) = – (0.010 kg)(300 m s–1)
m1u1 + m2u2 = (m1 + m2)v ∴ v = –2.0 m s–1
U
N Maklumat tambahan:
I (1 200 kg)(30 m s–1) + (1 000 kg)(–20 m s–1) Additional information:
T
= (1 200 + 1 000)kg × v
2 (36 000 – 20 000) kg m s–1 = (2 200 kg)v
(2 200 kg)v = 16 000 kg m s–1
∴ v = 7.27 m s–1

Aktiviti: Aplikasi Prinsip Keabadian Momentum dalam teknologi pelancaran roket air
Activity: Application of Principle of Conservation of Momentum in water rocket launching technology

• Udara di dalam botol dimampatkan (dipam dengan pam


angin). / Air inside the bottle is compressed (it is pumped
with air pump).

• Tekanan di dalam botol menjadi lebih tinggi.


Muncung Pressure inside the bottle becomes higher.
Nozzle
Botol • Air di dalam botol ditolak keluar dengan halaju yang
plastik pushed
Plastic Pam angin tinggi. / Water inside the bottle is out with high velocity .
Sayap bottle Air pump
Wing • Momentum ke arah bawah dihasilkan.
Air dipaksa keluar
Water forced out Downward
momentum is produced.

• Momentum ke arah atas dengan magnitud yang sama


dihasilkan. / Upward momentum with the same magnitude

is produced.

• Jumlah momentum terabadi .


Total momentum is conserved .

© Nilam Publication Sdn. Bhd. 44


MODUL • Fizik TINGKATAN 4

2.6 DAYA / FORCE SK 2.6

SP 2.6.1 Mendefinisikan daya sebagai kadar perubahan momentum

• Menyatakan bahawa kadar perubahan momentum berkadar terus


dengan daya dan bertindak pada arah tindakan daya.
State that the rate of change of momentum is directly proportional to
force and acted on the direction of applied force.
• Jika objek berjisim m dikenakan suatu daya F, dan halajunya berubah
Nyatakan definisi Hukum dari u ke v, maka
Gerakan Newton Kedua. If an object with mass, m been applied with a force, F, and the velocity is
State the definition of Newton's changing from u to v, thus
Second Law of motion.
Perubahan momentum
Daya α Masa
F α ma
F = kma, k(pemalar / constant) Rate of change of momentum
Force α
Time
F α mv – mu
t U
U
N
F α m ( – u ) iaitu a / which a
v N
II
t T
T

a = v – u 12
t
F α ma
F = ma

Pecutan, a, berkadar terus dengan a


daya yang dikenakan, F.
Hubungan antara a dan F The acceleration, a, is directly proportional
Relationship between a and F to the applied force, F.
a α F: m(pemalar / constant) 0 F
a melawan F
a against F

Pecutan, a, bagi suatu objek berkadar a


songsang dengan jisimnya, m.
Hubungan antara a dan m
Relationship between a and m The acceleration, a, of an object is inversely
1
1 0
a α m : F(pemalar / constant) proportional to its mass, m.
m
1
a melawan m
1
a against m

45 © Nilam Publication Sdn. Bhd.


MODUL • Fizik TINGKATAN 4

Eksperimen Mencari hubungan antara daya, jisim dan pecutan


Experiment Find the relationship between force, mass and acceleration

Daya dan pecutan, a α F 1


Hubungan antara Jisim dan pecutan, a α m
Force and acceleration, a α F
Relationship
1
between Mass and acceleration, a α
m
A A
B B

10 kg 10 kg
10 kg 5 kg
Situasi
Situation
Dua orang pemuda menolak jisim yang Dua orang pemuda mengenakan daya yang
sama tetapi pemuda A menolak dengan sama. Tetapi pemuda B bergerak dengan
daya yang lebih besar. Jadi dia bergerak lebih cepat daripada pemuda A.
dengan lebih cepat. Two men exerted the same force. But man B
Two men pushing the same mass but man A pushes moves faster than man A.
U with a greater force. So, he moves faster.
N
I
T Pecutan, a, bergantung kepada jisim
Pecutan, a, bergantung kepada daya yang
2 Inferens
Inference
dikenakan, F
objek, m
The acceleration, a, depends on mass of the object,
The acceleration, a, depends on applied force, F
m
Semakin besar daya, semakin besar pecutan. Semakin besar jisim, semakin kecil pecutan.
Hipotesis
Hypothesis The larger the force, the greater the acceleration. The greater the mass, the smaller the acceleration.

Untuk mengkaji hubungan antara pecutan, Untuk mengkaji hubungan antara pecutan,
Tujuan a dan daya yang dikenakan, F. a dan jisim objek, m.
Aim To investigate the relationship between acceleration, To investigate the relationship between acceleration,
a and applied force, F. a and object mass, m.
Pemboleh ubah
Daya yang dikenakan ke atas objek, F Jisim, m / Mass, m
dimanipulasikan
Manipulated Force applied on the object, F
variable

Pemboleh ubah Pecutan, a Pecutan, a


bergerak balas Acceleration, a Acceleration, a
Responding variable

Pemboleh ubah Jisim, m Daya yang dikenakan ke atas objek, F


dimalarkan Mass, m Force applied on the object, F
Constant variable

Jangka masa detik dan pita detik, bekalan kuasa, landasan terpampas geseran,
Bahan dan radas pembaris, troli, takal licin (dengan pengapit), tali tak kenyal, pemberat berslot
Materials and
apparatus Ticker timer and ticker tape, power supply, friction-compensated runaway, ruler, trolley,
smooth pulley (with clamp), inelastic string, slotted weights

© Nilam Publication Sdn. Bhd. 46


MODUL • Fizik TINGKATAN 4

Jangka masa detik


Pita detik Ticker timer
Ticker tape Troli
Trolley Tali tak kenyal
Inelastic string
Bekalan kuasa a.u. Takal licin
a.c. power supply Smooth pulley
Rajah
Diagram
Blok kayu Landasan terpampas
geseran Pemberat
Wooden block berslot
Friction-compensated
runway Slotted
weight

1 Radas disusun seperti ditunjukkan 1 Radas disusun seperti ditunjukkan


dalam rajah di atas. dalam rajah di atas.
The apparatus is set up as shown in the The apparatus is set up as shown in the
diagram above. diagram above.
2 Sebuah troli berjisim 1.0 kg (jisim 2 Sebuah troli dengan jisim , m = 1.0 kg
malar) diletakkan di atas landasan. Pita diletakkan di atas landasan. Pita detik
detik dilekatkan pada troli itu. dilekatkan pada troli itu.
A trolley of mass 1.0 kg (constant mass) is A trolley of mass, m = 1.0 kg is placed
placed on the runway. A length of ticker tape on the runway. A length of ticker-tape is
is attached to the trolley. attached to the trolley.
3 Jangka masa detik dihidupkan dan troli 3 Jangka masa detik dihidupkan dan troli U
itu ditarik oleh pemberat yang mempunyai itu ditarik oleh pemberat (daya malar U
N
N
daya, F = 10.0 N. pemberat ini ialah 10 N) II
The ticker timer is switched on and the The ticker timer is switched on and the T
T
Prosedur trolley is pulled by a weight of force, trolley is pulled by a weight of constant 12
Procedure F = 10.0 N. force, 10 N
4 Dari pita detik yang diperoleh, pecutan 4 Dari pita detik yang diperoleh, pecutan
troli dihitung dengan menggunakan troli dihitung dengan menggunakan
(v – u) (v – u)
formula, a = formula, a = .
t t
From the ticker tape obtained, the From the ticker tape obtained, the
acceleration of the trolley is calculated by acceleration of the trolley is calculated by
(v – u) (v – u)
using the formula, a = . using the formula, a = .
t t
5 Langkah-langkah 2 – 4 diulangi dengan 5 Eksperimen diulangi dengan dua troli
menambahkan pemberat berslot supaya, yang bertindan dan seterusnya tiga dan
F = 15.0 N, 20.0 N, 25.0 N dan 30.0 N. empat troli yang bertindan.
Steps 2 – 4 are repeated by adding slotted The experiment is repeated with two trolleys
weights to pull the trolley so that, F = 15.0 that are stacked together and then three and
N, 20.0 N, 25.0 N and 30.0 N. four trolleys that are stacked together.

F/N a / cm s–2
Troli 1
10.0 m / kg
–1 a / cm s–2
Trolleys
Keputusan 15.0 1
Results 20.0 2
25.0 3
30.0 4

a / cm s–2 a / cm s–2

Analisis data
Analysis of data
1 –1
0 F/N 0 m / kg

47 © Nilam Publication Sdn. Bhd.


MODUL • Fizik TINGKATAN 4

SP 2.6.2 Menyelesaikan masalah melibatkan rumus, F = ma

Latihan / Exercises

Nota / Note:
1. Perhatikan arah gerakan objek, F. / Observe the direction of moving object, F.
2. Daya, F ke hadapan (kanan) tolak daya, F ke belakang (kiri). / The forward (right) force, F subtract the backward (left) force, F.
3. Daya yang berlainan arah perlu ditolak, daya yang sama arah ditambah.
The different direction of the force should be subtracted, the same direction of the force should be added.

1 Hitungkan pecutan bagi blok di bawah: / Calculate the acceleration of the block:
(a) m = 2 kg (c) m = 10 kg
F = 8.0 N F=2N F = 18 N

F = ma F = ma
8 N = 2 kg(a) (18 – 2) N = 10 kg(a)
8.0 N (18 – 2) N 16 N
a = 2 kg a = 10 kg = 10 kg

= 4 m s-2 / 4 N kg-1 = 1.6 m s-2 / 1.6 N kg-1

(b) m = 8 kg (d) m = 12 kg
U F=6N F = 14 N F=5N F = 10 N
N R=5N
I
T F = ma F = ma
2 (14 + 6) N = 8 kg(a) 10 N – (5 + 5) N = 12 kg(a)
(14 + 6) N 20 N (10 – 5 – 5) N
a = 8 kg = 8 kg a = =0
12 kg
= 2.5 m s-2 / 2.5 N kg-1 = 0 m s-2

2 Sebuah objek yang berjisim 2 kg ditarik di atas tanah dengan daya 5 N dan
halaju malar. F1 = 5 N
R
An object of mass 2 kg is pulled on the floor by a force of 5 N and has a constant
velocity.
(a) Berapakah daya geseran antara objek dan tanah?
What is the frictional force between the object and the floor?
(b) Hitungkan pecutan objek itu jika objek itu ditarik dengan daya 17 N.
Calculate the acceleration of the object if the object is pulled by a 17 N force.
Penyelesaian / Solution:
(a) R ialah daya geseran / R is the frictional force
F1 – R = ma
∴ R = F1 – ma
Oleh kerana halaju malar / Because the velocity is constant,
a = 0
∴ R = F1 – 0
= F1
= 5 N
(b) F2 – R = ma
17 N – 5 N = (2 kg) (a)
12 N
a = = 6 m s–2
2 kg

© Nilam Publication Sdn. Bhd. 48


MODUL • Fizik TINGKATAN 4

3 Sebuah bas berjisim 2 000 kg bergerak dengan halaju seragam 40 m s–1 sejauh 2 500 m sebelum berehat.
Hitungkan
A bus of mass 2 000 kg travels at a uniform velocity 40 m s-1 for a distance of 2 500 m before it comes to rest. Calculate
(a) purata nyahpecutan bas itu. / the average deceleration of the bus.
(b) purata daya yang dikenakan oleh brek itu untuk membolehkan bas itu berhenti bergerak.
the average force applied by the brakes to bring the bus to stop.
Penyelesaian / Solution:
(a) v2 = u2 + 2as (b) F = ma
0 = (40 m s–1)2 + 2a(2 500 m) = (2 000 kg)(–0.32 m s–2)
∴ 5 000a = –1 600 m s–2 = –640 N
a = –0.32 m s-2 (Negatif bermaksud daya untuk menentang gerakan)
(Negative means force to resist the motion)

U
U
N
N
2.7 IMPULS DAN DAYA IMPULS / IMPULSE AND IMPULSIVE FORCE SK 2.7 II
T
T
12
SP 2.7.1 Berkomunikasi untuk menerangkan impuls dan daya impuls

Peta Pemikiran / Thinking Maps


Impuls (Menerangkan / Mencirikan) / Impulse (Explaining / Characterising)

Perubahan Formula
momentum Ft = mv – mu
Change in momentum
Impuls
Impulse
Unit:/Unit: Hasil darab daya
N s atau / or dan masa, Ft
The product of force
kg m s–1
and time, Ft

Berikan definisi • Hasil darab daya dan masa , Ft.


impuls.
The product of force and time , Ft.
Give the definition
of impulse. • Impuls adalah perubahan momentum.
Impulse is the change of momentun.
• Rumus / Formula, Ft = mv-mu
• Unit: N s / kg m s-1

49 © Nilam Publication Sdn. Bhd.


MODUL • Fizik TINGKATAN 4

Berikan definisi Daya impuls ialah kadar perubahan momentum dalam perlanggaran
daya impuls.
Give the definition atau hentaman dalam masa yang singkat. / An impulsive force is the rate of
of impulsive force. change of momentum in a collision or impact in a short period of time.
Rumus daya impuls: / Formula for impulsive force:
v–u
F = ma = m ( )
t

mv – mu perubahan momentum / change of momentum
F= =
t masa / time
Unit = N = kg m s–2

Berikan definisi Hukum Gerakan Newton Ketiga menyatakan untuk setiap daya tindakan, terdapat satu
Hukum Gerakan
daya tindak balas yang sama magnitud tetapi bertentangan arah.
Newton Ketiga.
Give the definition Newton’s Third Law of Motion states that for every action force, there is reaction
of Newton’s Third force magnitude
Law of Motion. at the same but in opposite direction.

Daripada
Daripada formula, F = mv – mu
formula daya t
U
N impuls, apa mv – mu
From the formula, F = .
I
yang dapat anda t
T
rumuskan? Daya impuls berkadar songsang dengan masa impak.
2 From the formula Impulsive force is inversely proportional to the impact time.
Fα 1
of impulsive force,
what can you t
summarise? Masa impak panjang / Longer impact time Daya impuls kecil / Small impulsive force
mv – mu Masa impak pendek / Shorter impact time Daya impuls besar / Longer impulsive force
F=
t

Contoh-contoh Situasi Daya Impuls perlu Dikurangkan [F kecil, t panjang]


Examples of Situation for Impulsive Force needs to be Reduced [F small, t longer]

1 2 3 4

Melompat ke atas Memakai sarung Membengkokkan Menangkap bola


tilam yang tebal tangan untuk kaki semasa mengikut arah
Jumps on the thick menangkap bola mendarat pergerakan bola
mattress Wearing gloves to Bend the legs upon Catch the ball in the
catch a ball landing motion of the ball
5 Bahan yang mudah pecah seperti telur, kaca dan perkakasan elektrik mestilah dibungkus dalam bahan yang
lembut dan boleh dimampatkan. Mengapa? / Items that are fragile such as eggs, glass and electrical appliances
must be wrapped in materials that are soft and compressible. Why?
Bahan yang lembut dan mudah dimampatkan menghasilkan masa perlanggaran yang panjang. Ia menyerap
hentakan. Jadi ia mengurangkan daya impuls.

Soft and compressible material provides longer time of impact. It absorbs the shock. So it can reduce the impulsive

force.

© Nilam Publication Sdn. Bhd. 50


MODUL • Fizik TINGKATAN 4

Contoh-contoh Situasi Daya Impuls Ditingkatkan [F besar, t pendek]


Examples of Situation for Impulsive Force Needs to be Increased [F larger, t shorter]

Situasi Penjelasan
Situations Explanation

• Peserta karate yang mahir boleh memecahkan kayu yang tebal dengan
menggunakan sisi tangan yang bergerak dengan kelajuan yang sangat tinggi.
A karate expert can break a thick wooden slab with his bare hand that moves at
a very fast speed.

• Tangan tersebut digerakkan pada halaju yang tinggi. Masa tindakan adalah
singkat . Menghasilkan daya impuls yang besar .
The hand is moved at high velocity. Time of impact is shorter . It produce
large impulsive force.

• Kepala penukul yang besar bergerak pada kelajuan yang tinggi untuk
memukul paku.
A massive hammer head moving at a fast speed is brought to rest upon hitting the U
U
nail. N
N
II
T
T

• Masa tindakan adalah pendek apabila kepala penukul memukul pada 12


halaju yang tinggi. Ia akan menghasilkan daya impuls yang besar .
Time of impact is shorter when the hammer head is hit at high velocity.

It produces large impulsive force.

• Bola sepak mempunyai tekanan udara yang tinggi di dalamnya. Apabila bola
Daya impuls
disepak dengan kuat dan diikuti dengan tindakan ikut lajak, bola mengalami
Impulsive force perubahan momentum yang besar dan halaju yang tinggi.
A football has a high air pressure inside it. When the ball is kicked strongly and
followed by a through action, the ball experiences a large change in momentum
and moves at high velocity.

• Masa impak menjadi pendek , daya impuls yang terhasil adalah


besar .
The time of impact becomes shorter , the impulsive force is large .

• Alu digerakkan pada halaju yang tinggi ke arah lesung dan dihentikan pada
Alu masa yang singkat.
Pestle
A pestle moving at high velocity towards a mortar and stopped in a short time.

Daya • Masa impak adalah pendek . Maka, daya impuls adalah besar .
Lesung impuls
Mortar Impulsive Time of impact is shorter . So the impulsive force is large .
force

51 © Nilam Publication Sdn. Bhd.


MODUL • Fizik TINGKATAN 4

Aktiviti: Mereka cipta model kenderaan yang mengaplikasikan Hukum Gerakan Newton
Activity: Create a vehicle by applying Newton's Laws of motion

Beg udara / Airbag


Menyerap hentakan dengan menambahkan masa impak
apabila kepala pemandu terhentak ke stereng. Oleh itu,
daya impuls dikurangkan.
Absorbing impact by increasing the impact time when the
driver’s head is thrown towards the steering. So, the impulsive
force is reduced.

Cermin hadapan / Windscreen


Kaca tahan pecah yang tidak mudah pecah kepada
serpihan yang kecil semasa perlanggaran. Ini

U mengurangkan kecederaan disebabkan oleh serpihan


N
I kaca yang berselerak.
T

2 Shatterproof glass that will not break into small pieces Zon remuk
depan
during collision. This will reduce injuries caused by the Front crumple
zone
scattered glass.

Bumper hadapan / Front bumper


Papan pemuka
Menyerap hentakan akibat daripada kemalangan.
Dashboard
Diperbuat daripada keluli, aluminium, plastik,
getah dan fiber komposit.
Semasa perlanggaran, papan D
Absorb the shock from the accident. Made from steel,
pemuka kemek. Ini akan d
aluminium, plastic, rubber and composite fibres. menyerap kesan hentaman w
dengan meningkatkan masa in
perlanggaran antara kepala b
Sistem brek anti kunci pemandu dan stereng. Jadi th
Anti-lock braking system (ABS) ia mengurangkan daya th
Membolehkan pemandu memberhentikan kereta dengan impuls.
segera tanpa menyebabkan roda terkunci apabila brek
ditekan secara tiba-tiba. Mengelakkan kereta daripada
menggelongsor.
Enables drivers to quickly stop the car without the wheels
locking when the brake is applied suddenly. Prevents the car
from skidding.

© Nilam Publication Sdn. Bhd. 52


MODUL • Fizik TINGKATAN 4

Penyandar kepala / Headrest


Untuk mengurangkan kesan inersia terhadap kepala
pemandu. Mengurangkan kecederaan leher apabila
kereta dilanggar daripada belakang.

To reduce the inertia effect on the driver’s head. Reduce

neck injury when the car is hit from behind.

Zon remuk belakang / Rear crumple zone


Boleh dimampatkan ketika kemalangan. Jadi ia akan
U
U
meningkatkan masa yang diperlukan kereta untuk N
N
II
T
T
berhenti sepenuhnya. Maka, ia akan mengurangkan
12
daya impuls.

Can be compressed during an accident. So it can increase

the time taken by the car for a complete stop. So, it can

reduce the impulsive force.


muka
d Tali pinggang keselamatan / Safety belt
Untuk mengurangkan kesan inersia dengan
mengelakkan pemandu daripada tercampak ke
an During collision, the
dashboard collapses. This hadapan.
n will absorb the impact by
To reduce the inertia effect by preventing the driver from
sa increasing the time of collision
a between the driver's head and being thrown forward.
i the steering. This will reduce
the impulsive force.
Bar hentaman sisi / Side impact bar
Menyerap hentakan dengan menambahkan masa impak
apabila kepala pemandu terhentak ke stereng. Oleh itu,
daya impuls dikurangkan..
Absorbing impact by increasing the impact time when the
driver’s head is thrown towards the steering. So, the impulsive
force is reduced.

53 © Nilam Publication Sdn. Bhd.


MODUL • Fizik TINGKATAN 4

SP 2.7.2 Menyelesaikan masalah melibatkan impuls dan daya impuls

Latihan / Exercises
1 Rosli yang berjisim 60 kg melompat dari tingkat 2 Rooney menyepak bola dengan kekuatan daya
pertama sebuah rumah yang terbakar. Halajunya 1 500 N. Masa tindakan di antara kasut dan bola
sejurus sebelum mendarat ialah 6 m s-1. ialah 0.01 s. Berapakah impuls yang dikenakan
Rosli, with a mass of 60 kg, jumps from the first floor kepada bola itu? Jika jisim bola itu ialah 0.5 kg,
of a burning house. His velocity just before landing on berapakah halaju bola tersebut?
the ground is 6 m s-1. Rooney kicks a ball with a force of 1 500 N. The time of
contact of his boot with the ball is 0.01 s. What is the
(a) Kirakan impuls apabila kakinya mencecah impulse delivered to the ball? If the mass of the ball is
tanah. 0.5 kg, what is the velocity of the ball?
Calculate the impulse when his legs hit the ground.
(b) Berapakah daya impuls yang bertindak ke Penyelesaian / Solution:
atas kaki Rosli jika dia membengkokkan kaki
Impuls/Impulse = Ft = 1 500 N × 0.01 s
ketika mendarat dan mengambil masa 0.5 s
= 15.0 N s
untuk berhenti?
= 15.0 kg m s-1
What is the impulsive force on Rosli’s legs if he
bends upon landing and takes 0.5 s to stop? mv – mu = 15 kg m s-1 u=0
(c) Berapakah daya impuls yang bertindak ke atas (0.5 kg)v = 15 kg m s-1
kaki Rosli jika dia tidak membengkokkan kaki v = 30 m s-1
dan berhenti dalam 0.05 s?
U
N What is the impulsive force on Rosli’s legs if he 3 Dalam satu perlawanan tenis, pemain memukul
I does not bend and stops in 0.05 s? bola yang mempunyai jisim 0.2 kg yang menuju ke
T
(d) Apakah kebaikan membengkokkan kaki arahnya dengan halaju 20 m s-1. Bola itu memantul
2 semasa mendarat? dengan halaju 40 m s-1. Masa yang diambil semasa
What is the advantage of bending his legs upon perlanggaran antara bola dan raket tenis ialah 0.01
landing? s.
In a tennis match, a player hits an on-coming ball with
Penyelesaian / Solution: mass of 0.2 kg and velocity of 20 m s-1. The ball rebounds
(a) mv – mu = m(v – u) with a velocity of 40 m s-1. The time taken in the collision
= (60 kg)(0 – 6) m s–1 between the ball and the tennis racket is 0.01 s.
= –360 kg m s–1 u = –20 m s-1

Impuls adalah –360 kg m s–1 kerana
momentumnya dikurangkan sehingga sifar.
The impulse is –360 kg m s–1 because its momentum is v = 40 m s-1
reduced to zero.
mv – mu –360 kg m s–1 0.2 kg
(b) F = t = 0.5 s t = 0.01 s
= –720 N
(a) Berapakah impuls yang dialami oleh bola itu?
mv – mu –360 kg m s–1
(c) F = = What is the impulse experienced by the ball?
t 0.05 s
(b) Berapakah daya impuls yang dikenakan ke
= –7 200 N atas bola tenis?
(b) dan (c) : Tanda negatif bagi daya bermakna daya
What is the impulsive force exerted on the tennis
ini telah menyebabkan kehilangan momentum
ball?
(b) and (c) : The negative sign to the force means that the
force has caused a loss of momentum Penyelesaian / Solution:
(d) Dengan membengkokkan kaki semasa mendarat, dia (a) Impuls = m (v – u)
akan meningkatkan masa impak dan mengurangkan Impulse = (0.2 kg)[(40 – (–20)] m s–1
daya impuls. Jadi ia dapat mengurangkan kecederaan. = 12.0 kg m s-1
By bending his legs upon landing, he will increase the
= 12.0 N s
time of impact and reduce the impulsive force. So it will 12 N s
(b) F = 0.01 s = 1 200 N
minimise the injuries.

© Nilam Publication Sdn. Bhd. 54


MODUL • Fizik TINGKATAN 4

2.8 BERAT / WEIGHT SK 2.8

SP 2.8.1 Menyatakan berat sebagai daya graviti yang bertindak ke atas suatu objek, W = mg

Berikan definisi berat, W. • Berat, W, bagi sesuatu objek ialah kuantiti vektor yang mempunyai
Give the definition of weight, W. arah yang sama dengan daya graviti yang bertindak ke arah pusat
Bumi. / The weight, W, of an object is a vector quantity which has the same
direction with gravitational force, that is towards the centre of the Earth.
Pecutan = g
Jisim / Mass = m kg
Acceleration = g
• Berat ialah daya dan diukur dalam unit Newton, N.
Weight is a force and is measured in Newton, N.

• Sebiji batu yang berjisim m, dilepaskan jatuh bebas pada pecutan


Berat / Weight = m × 10 N graviti, g. / A stone of mass, m, is released and free falls with a
Bumi / Earth gravitational acceleration of, g.

• Daya yang bertindak ke atas batu hanyalah berat, W, di mana ianya


menuju ke arah bawah.
The only force acting on the stone is its weight, W, which is downward.

Hukum Gerakan Newton Kedua: U


U
N
N
Newton’s Second law of Motion: II
T
F = ma T
12
W = mg

Oleh itu / Therefore:


W = mg ; W = berat / weight
m = jisim / mass
g = pecutan graviti / gravitational acceleration
unit bagi g ialah = N kg–1

the unit of g is = N kg–1

Mengapa angkasawan yang • Di Bulan, berat kita menjadi lebih ringan daripada di Bumi, ini
berada di Bulan terasa lebih kerana medan graviti di Bulan adalah lebih kecil berbanding di
ringan dan terapung-apung Bumi. / On the Moon, our weight is less than that on Earth because the
pada permukaan Bulan? Moon’s gravitational field is smaller than that of the Earth.
Tetapi, apabila pulang ke Bumi, • Malahan di Bumi, berat kita berbeza sedikit dari suatu tempat ke
angkasawan tersebut berasa tempat yang lain, kerana kekuatan medan graviti yang berbeza.
sukar untuk berjalan di atas Semakin jauh dari Bumi, berat kita semakin berkurang.
permukaan Bumi. Even on Earth, our weight can vary slightly from place to place, because
Why an astronaut on the Moon feels the Earth’s gravitational field strength varies. Moving away from the
lighter and floating on the surface of Earth, our weight decreases.
the Moon? But, when come back to • Jika kita boleh pergi lebih jauh ke dalam ruang angkasa lepas dan
the Earth, the astronaut feels difficult
bebas daripada sebarang tarikan graviti, berat kita akan menjadi
to walk on the surface of the Earth.
sifar.
If we could go deep into space, and be free from any gravitational pull,
our weight would become zero.
• Tetapi, jisim kita tetap tidak akan berubah, sama ada di atas Bumi,
Bulan atau di angkasa.
But our mass does not change, whether on the Earth, Moon or deep in
space.

55 © Nilam Publication Sdn. Bhd.


MODUL • Fizik TINGKATAN 4

1
Kekuatan medan graviti di Bulan ialah daripada kekuatan medan
6
graviti di Bumi.
1
Gravitational field strength on the Moon is of the gravitational field
6
strength on Earth.

Di angkasa Di permukaan Di permukaan


lepas Bulan Bumi
Video Deep in On Moon’s On Earth’s
space surface surface
Jisim
100 kg 100 kg 100 kg
Mass
Berat 1
(1 000) N
Daya Graviti 0N 1 000 N
U Gravitational Force Weight 6
N
I
T
Nyatakan perbezaan antara
2 berat dan jisim.
Berat / Weight Jisim / Mass
State the difference between weight Daya graviti yang Jumlah jirim di dalam
and mass.
bertindak ke atas objek. objek.
Definisi
Definition The force of gravity acting The amount of matter in

on the object. the object.

Berat sesuatu objek Jisim sesuatu objek


berubah dengan tidak berubah
Perubahan / walau di mana-mana.
kekuatan medan graviti
Tiada
pada sesuatu tempat. The mass of an object is
perubahan
The weight of an object unchanged anywhere.
Change /
Unchanged changes with the
gravitational field strength
at the location.

Kuantiti Kuantiti vektor Kuantiti skalar


Quantity Vector Scalar
quantity quantity

Unit S.I. Newton, N Kilogram, kg


S.I. unit Newton, N Kilogram, kg

© Nilam Publication Sdn. Bhd. 56


MODUL • Fizik TINGKATAN 4

Praktis SPM / SPM Practice


Soalan Objektif / Objective Questions

1 Rajah 1 menunjukkan troli diletakkan di atas 4 Rajah 3 menunjukkan satu pita detik.
landasan terpampas geseran. / Diagram 1 shows a Diagram 3 shows a ticker tape.
trolley placed on a friction-compensated runway.
Arah gerakan
Direction of motion

Rajah 1 / Diagram 1 Rajah 3 / Diagram 3


Troli itu akan / The trolley will Pernyataan yang manakah adalah benar?
A bergerak dengan halaju bertambah. Which statement is correct?
move with an increasing velocity. A Objek mengalami nyahpecutan
B bergerak dengan halaju berkurang. The object decelerates
move with decreasing velocity. B Halaju objek bertambah
C bergerak dengan halaju malar/tetap. The velocity of the object increases
move with a constant velocity. C Objek bergerak dengan halaju seragam
D kekal tidak bergerak. The object moves with a constant velocity
remain stationary. D Objek mengalami nyahpecutan dan kemudian U
U
N
N
bergerak dengan halaju seragam II
2 Tangki silinder minyak kereta En. Ali bocor. The object decelerates and then moves with a T
T
Tompokan minyak kelihatan sepanjang jalan dari A constant velocity 12
ke B seperti ditunjukkan dalam Rajah 2. / The cylinder
oil tank of En. Ali’s car is leaking. Lubricant spots are seen 5 Rajah 4 menunjukkan dua biji telur serupa masing-
along the road from A to B as shown in Diagram 2.
masing dijatuhkan dari ketinggian yang sama, h, ke
A B
atas lantai simen dan span.
Diagram 4 shows two similar eggs are released from
same height, h onto cement floor and sponge respectively.
10 m 15 m 20 m
Telur Telur
Rajah 2 / Diagram 2 Egg Egg
Ini menunjukkan bahawa kereta Ali bergerak dengan
It can be concluded that En Ali’s car is moving with h h
A pecutan malar. / constant acceleration.
B nyahpecutan malar. / constant deceleration.
C pecutan sifar. / zero acceleration.
Lantai simen Span
D pecutan berkurang dengan malar/tetap. Cement floor Sponge
acceleration that is decreasing constantly.
Rajah 4 / Diagram 4
3 Antara pernyataan berikut, yang manakah Inferens manakah yang benar?
menghuraikan perlanggaran kenyal antara dua Which inference is correct?
objek dengan betul? / Which of following statements A Daya impuls bergantung kepada halaju telur
best describes an elastic collision between two objects?
Impulsive force depends on velocity of egg
A Hanya mementum terabadi.
B Daya impuls bergantung kepada masa hentaman
Only momentum is conserved.
Impulsive force depends on time of impact
B Hanya tenaga kinetik terabadi.
Only kinetic energy is conserved. C Daya impuls bergantung kepada daya graviti
C Kedua-dua momentum dan tenaga kinetik Impulsive force depends on gravitational force
terabadi. D Daya impuls bergantung kepada ketinggian
Momentum and kinetic energy are conserved. telur
D Jumlah tenaga, momentum dan tenaga kinetik Impulsive force depends on height of egg
terabadi. / Total energy, momentum and kinetic
energy are conserved.
57 © Nilam Publication Sdn. Bhd.
MODUL • Fizik TINGKATAN 4

Soalan Struktur / Structure Questions

1 Rajah 1.1 menunjukkan sebuah bas yang bergerak dengan halaju 70 km/j dan menunjukkan kedudukan
penumpang di dalam bas apabila brek tiba-tiba digunakan. Rajah 1.2 menunjukkan tukang masak cuba untuk
menggoncang sos keluar daripada botol.
Diagram 1.1 shows a bus moving with a velocity of 70 km/h and shows the condition of a passenger standing in the bus
when the brakes are suddenly applied. Diagram 1.2 shows a cook trying to shake the sauce out of a bottle.

Rajah 1.1 / Diagram 1.1 Rajah 1.2 / Diagram 1.2


(a) Berdasarkan kedua-dua rajah di atas, nyatakan satu ciri yang dialami oleh penumpang dan sos.
Based on both of the diagrams above, state one characteristic experienced by the passengers and the sauce.
Penumpang bergerak ke hadapan dan sos cili mengalir keluar daripada botol.
The passengers move forward and the chilli sauce comes out from the bottle.
U
N (b) Apakah halaju penumpang
I What is the velocity of the passenger
T
(i) sebelum pemandu bas membrek?
2 before the bus driver brakes?
70 km/j / 70 km/h
(ii) selepas pemandu bas membrek?
after the bus driver brakes?
Halaju yang sama iaitu 70 km/j. / The same velocity that is 70 km/h.
(c) Apakah halaju botol apabila sos mengalir keluar?
What is the velocity of the bottle when the sauce comes out?
Halaju sifar. / Zero velocity.
(d) (i) Berdasarkan jawapan di (b), tulis satu kenyataan pergerakan bas dan penumpangnya.
Based on the answer at (b), write a statement on the motion of the bus and its passengers.
Apabila bas brek, halaju bas berkurangan dengan tiba-tiba, penumpang terus bergerak dengan halaju

awalnya. / When the bus brakes, the velocity of the bus decreases suddenly, the passenger continues to move with

its initial velocity.


(ii) Namakan konsep fizik yang terlibat.
Name the physics concept involved here.
Inersia / Inertia
(e) Jika anda dikejar oleh anak gajah, bagaimana anda berlari untuk menyelamatkan diri? Terangkan.
If you are chased by a baby elephant, how can you run to save yourself? Explain.
Saya perlu berlari secara zig zag. Anak gajah akan terus bergerak dalam garis lurus apabila saya
menukar arah dengan tiba-tiba. / I have to run in a zig zag manner. The baby elephant will continue to move in
a straight line when I change my direction suddenly.

© Nilam Publication Sdn. Bhd. 58


MODUL • Fizik TINGKATAN 4

2 (a) Dengan menggunakan prinsip keabadian momentum, terangkan prinsip kerja roket.
Using the principle of conservation of momentum, explain the working principle of the rocket.
1. Bahan api terbakar di dalam kebuk pembakaran. / Fuel burns in the combustion chamber.
2. Gas panas dipancut keluar dengan halaju tinggi. / Hot gases are expelled at high speed / high momentum.
3. Momentum besar ke bawah terhasil. / A large downwards momentum is produced.
4. Roket memperoleh momentum ke atas yang sama magnitud.
Rocket gains upwards momentum of equal magnitude.

(b) Rajah 2 menunjukkan sebuah roket. / Diagram 2 shows a rocket.

Tangki untuk
bekalan tambahan
Tank for
Hidrogen cecair additional supply
Liquid hydrogen

Kebuk pembakaran
Combustion chamber

U
U
Rajah 2 / Diagram 2 N
N
II
Anda dikehendaki memberi beberapa cadangan untuk mereka bentuk sebuah roket yang boleh bergerak T
T
di angkasa lepas dengan pecutan yang lebih tinggi. Dengan menggunakan pengetahuan tentang daya dan
gerakan serta sifat bahan, terangkan cadangan anda berdasarkan aspek-aspek berikut:
12
You are required to give some suggestions to design a rocket which can travel in the outer space with higher
acceleration. By using the knowledge on forces and motion and the properties of materials, explain the suggestions
based on the following aspects:
(i) Bentuk roket / The shape of the rocket
Bentuk aerodinamik/ larus/ torpedo → Mengurangkan rintangan udara
Aerodynamic shape / stream line/ torpedo → Reduces air resistance

(ii) Bahan untuk membuat roket / The material used to build the rocket
Ketumpatan bahan rendah // Bahan kuat // Takat lebur tinggi → Lebih ringan // Tidak mudah pecah
Low density material // Strong material // High melting point → Lighter // Does not break easily

(iii) Bekalan tambahan yang diperlukan untuk roket bergerak di angkasa lepas
Additional supply needed to enable the rocket to travel in outer space
Ada oksigen cecair → Mencepatkan pembakaran // membekal oksigen untuk pembakaran
Has liquid oxygen → Boosting combustion // supply oxygen for combustion

(iv) Struktur roket untuk meningkatkan pecutan / The structure of the rocket to increase its acceleration
Roket retro. Terdapat beberapa peringkat yang boleh ditanggalkan → Untuk mengurangkan jisim
Retro rocket. Has several stages that can slip/strip off → To decrease mass

(v) Saiz kebuk pembakaran / The size of the combustion chamber


Menambah saiz kebuk pembakaran → Lebih ruang untuk pembakaran bahan api
Increase the size of combustion chamber → More space for combustion of fuel

59 © Nilam Publication Sdn. Bhd.


MODUL • Fizik TINGKATAN 4

Teknik Menjawab [Format Kertas 2: Perbandingan] / Answering Technique [Paper 2 Format: Comparison]

1 Rajah 1 menunjukkan dua situasi Ah Chong sedang menolak troli berisi buah pada jisim berbeza. Permukaan
jalan adalah kasar dan daya geseran antara permukaan jalan dengan permukaan roda troli ialah 2 N.
Diagram 1 shows two situations of Ah Chong pushing a trolley filled with fruits at different masses. The surface of the road
is rough and the frictional force between the surface of the road and the surface of the wheel of trolley is 2 N.
(a) Tentukan daya bersih dan pecutan troli bagi setiap rajah.
Determine the net force and acceleration of the trolley for each diagram.

a a
22 N 22 N

20 kg 25 kg

2N 2N
(a) (b)
Rajah 1 / Diagram 1
U
N Daya bersih / Net force = Daya bersih / Net force =
I
T
22 + (–2) = 20 N 22 + (–2) = 20 N
2 F = ma F = ma
20a = 20 25a = 20
a = 1 m s–2 a = 0.8 m s–2

(b) Bandingkan jisim troli, pecutan troli dan daya bersih yang bertindak pada kedua-dua troli di dalam jadual
di bawah.
Compare the mass of trolley, acceleration of trolley and net force acting on both trolleys in the table below.
Aspek / Aspect Rajah (a) / Diagram (a) Rajah (b) / Diagram (b)

Jisim troli Lebih kecil Lebih besar


Mass of trolley Smaller Larger

Pecutan troli Lebih besar Lebih kecil


Acceleration of trolley Larger Smaller

Daya bersih yang bertindak pada troli Sama Sama


Nett force acting on the trolley Same Same

(c) Nyatakan hubungan antara jisim troli dengan pecutan troli.


State the relationship between the mass of trolley and acceleration of the trolley.
Semakin berkurang jisim troli, semakin bertambah pecutan troli.

As the mass of trolley decreases, the acceleration of trolley increases.

© Nilam Publication Sdn. Bhd. 60


MODUL • Fizik TINGKATAN 4

2 Rajah 2 menunjukkan dua orang sedang menaiki basikal dengan jisim yang sama dalam dua situasi yang
berbeza. Rajah (a) menunjukkan kedua-dua mengayuh basikal manakala Rajah (b) menunjukkan hanya
seorang mengayuh basikal tersebut. Halaju basikal di Rajah (a) bertambah dengan kadar yang lebih cepat
daripada halaju basikal di Rajah (b). / Diagram 2 shows two people are cycling with the same mass in two different
situations. Diagram (a) shows both are cycling while Diagram (b) shows only one is cycling. The velocity of bicycle in
Diagram (a) increases with higher rate than the bicycle in Diagram (b).

(a) (b)
Rajah 2 / Diagram 2

(a) Lengkapkan jadual di bawah dengan membandingkan aspek-aspek berikut:


Complete the table below by comparing the following aspects:
Aspek / Aspect Rajah (a) / Diagram (a) Rajah (b) / Diagram (b)
Jumlah jisim basikal Sama Sama
Total mass of bicycle Same Same
U
Bilangan penunggang yang terlibat dalam Dua orang Satu orang U
N
N
tindakan pengayuhan II
Number of cyclist involved in action of cycling Two One T
T

Lebih besar Lebih kecil


12
Daya tujah ke depan yang terhasil
Thrust force produced Larger Smaller

Pecutan basikal Lebih besar Lebih kecil


Acceleration of bicycle Larger Smaller

(b) Nyatakan hubungan antara daya tujah ke depan dan pecutan basikal.
State the relationship between the thrust force and acceleration of bicycle.
Semakin bertambah daya tujah ke depan, semakin bertambah pecutan basikal.

As the thrust force increases, the acceleration of bicycle increases.

(c) Berdasarkan jawapan anda di (b), namakan hukum fizik yang terlibat.
According to your answer in (b), name the physics law involved.
Hukum Newton Kedua / Newton's Second Law

(d) Kenal pasti pemboleh ubah: / Identify the variables:


(i) Dimanipulasikan / Manipulated : Daya, F / Force, F
(ii) Bergerak balas / Responding : Pecutan, a / Acceleration, a
(iii) Dimalarkan / Constant : Jisim basikal, m / Mass of bicycle, m

Teori: / Theory:
a
• Apabila jisim, m adalah malar, pecutan, a, bergantung pada daya, F.
As mass, m, is constant, acceleration, a, depends on force, F . F = ma
• Semakin bertambah daya, F, semakin bertambah pecutan, a.
As the force, F increases, the acceleration, a increases. a # = F#
• a berkadar terus dengan F (a α F). m (malar/constant)
a is directly proportional with F (a α F). 0 F

61 © Nilam Publication Sdn. Bhd.


MODUL • Fizik TINGKATAN 4

Unit
KEGRAVITIAN
3 GRAVITATION

HUKUM KEGRAVITIAN SEMESTA NEWTON


3.1
NEWTON'S UNIVERSAL LAW OF GRAVITATION SK 3.1

SP 3.1.1 Menerangkan Hukum Kegravitian Semesta

Hukum Kegravitian Semesta Newton / Newton’s Universal Law of Gravitation


Menurut hukum ini, satu jasad menarik satu jasad yang lain di alam semesta ini menggunakan
satu daya yang berkadar terus dengan hasil darab jisim kedua-duanya dan berkadar
songsang dengan kuasa dua jarak antara mereka. Daya ini dikenali sebagai daya graviti .
According to this law, one body attracts the other body in the universe with a force which is directly proportional

to the product of masses of two bodies and inversely proportional to the square of the distance between the centre
of the two bodies. This force is known as gravitational force.

Jasad pertama Jasad kedua


First body Second body

F F
U m1 m2
N
I
T

3 r

Gm1m2
F=
r2

F = Daya graviti antara dua jasad / The gravitational force between two bodies
m1 = Jisim bagi jasad pertama / Mass of first body
m2 = Jisim bagi jasad kedua / Mass of second body
r = Jarak di antara pusat jasad pertama dan pusat jasad kedua
The distance between the centre of the first body and the centre of the second body
G = Pemalar kegravitian / Gravitational constant (G = 6.67 × 10-11 N m2 kg-2 )

Video

Hukum Kegravitian
Law of Gravitation

© Nilam Publication Sdn. Bhd. 62


MODUL • Fizik TINGKATAN 4

Menurut Hukum Newton Ketiga, / Under Newton’s Third Law,


Bagi setiap tindakan, wujud tindak balas yang sama tetapi bertentangan arah. Maka, bagi setiap daya
yang Bumi kenakan kepada setiap objek di atasnya, daya tindak balas yang sama tetapi bertentangan arah
dikenakan oleh objek-objek tersebut kepada Bumi. Meskipun kita sering melihat objek jatuh ke Bumi, kita
belum pernah menyaksikan Bumi “jatuh” ke arah objek. Hal ini kerana jisim Bumi jauh lebih besar dari
F
objek tersebut dan walaupun dikenakan daya yang besar, daripada persamaan F = ma, maka a = ; iaitu
m
pecutan menjadi sangat kecil apabila jisim, m menjadi sangat besar. Kesan graviti lebih besar jika kita melihat
pada tarikan antara Bumi dan Bulan. Kedua-duanya saling menarik antara satu sama lain dan bukan Bumi
menarik Bulan semata-mata.
For each action, there is an equal reaction but in the opposite direction. Thus, for every force that the Earth imparts
to any object on it, an equal reaction force but in the opposite direction is imposed by those objects on the Earth.
Although we often see objects falling to the Earth, we have never seen the Earth "fall" towards the object. This is
because the mass of the Earth is much larger than the object and although a great force is exerted from the equation
F
F = ma, then a = ; the acceleration becomes very small when the mass becomes very large. The impact of gravity
m
is greater if we look at the attraction between the Earth and the Moon. Both mutually attract each other and it is not
as if the Earth is attracting the Moon alone.

• Bulan dapat beredar mengelilingi Bumi kerana wujudnya daya graviti antara Bulan dan Bumi.
The moon can orbit around the Earth as there is gravitational force between Moon and Earth.

Lintasan Bulan jika tiada


tarikan graviti U
U
Pathway of the Moon if no N
N
gravitational force II
T
T
Graviti
Gravity 13
Bulan
Moon

Bumi / Earth

• Planet-planet dapat beredar mengelilingi Matahari kerana daya graviti yang dikenakan antara planet-
plane dan Matahari . / The planets can orbit around the Sun because of the gravitational force
imposed between the planets and the Sun .

Planet Daya graviti


Gravitational force Planet
Matahari
Sun

Daya graviti
Gravitational force Planet

63 © Nilam Publication Sdn. Bhd.


MODUL • Fizik TINGKATAN 4

Menyelesaikan masalah melibatkan Hukum Kegravitian Semesta Newton bagi:


SP 3.1.2
(i) dua jasad pegun di Bumi (ii) jasad di atas permukaan Bumi (iii) Bumi dan satelit (iv) Bumi dan Matahari

Latihan / Exercises
1 Kirakan paduan daya graviti pada Bulan (mb = 7.35 × 1022 kg) akibat daya Bulan Bumi
Moon Earth
graviti Bumi (mB = 5.97 × 1024 kg) dan Matahari (mM = 1.99 × 1030 kg), FbB
dengan mengandaikan kedudukan ketiga-tiganya membentuk sudut tegak
antara satu sama lain (seperti di dalam rajah).
(Jarak antara Bulan-Bumi, rbB = 3.84 × 108 m) FbM
(Jarak antara Bulan-Matahari, rbM = 1.50 × 1011 m) Fb
Calculate the resultant gravitational force of the Moon (mb = 7.35 × 1022 kg) due
to the gravitational force of the Earth (mB = 5.97 × 1024 kg) and the Sun
(mM = 1.99 × 1030 kg), assuming that FbB is perpendicular to FbM Matahari
Sun
(as in the diagram).
(Distance between Moon-Earth, rbB = 3.84 × 108 m)
(Distance between Moon-Sun, rbM = 1.50 × 1011 m)

FbB = Daya graviti pada Bulan oleh Bumi / The gravitational force of the Moon by the Earth
FbM = Daya graviti pada Bulan oleh Matahari / The gravitational force of the Moon by the Sun
Fb = Paduan daya graviti pada Bulan / Resultant gravitational force on the Moon

Daya graviti antara Bulan dengan Bumi. / The gravitational force between Moon and Earth.
GmbmB
FbB =
rbB2
U (6.67 × 10–11 N m2 kg–2)(7.35 × 1022 kg)(5.97 × 1024 kg)
N
=
I (3.84 × 108 m)2
T
= 1.98 × 1020 N
3
Daya graviti Bulan dengan Matahari. / The gravitational force between the Moon and the Sun.
GmbmM
FbM =
rbM2
(6.67 × 10–11 N m2 kg–2)(7.35 × 1022 kg) (1.99 × 1030 kg)
=
(1.50 × 1011 m)2
= 4.34 × 1020 N

Kerana daya kedua-dua ini saling bersudut tegak maka daya paduan graviti yang bertindak pada bulan ialah:
Because these two forces are perpendicular to each other, the resultant gravitational force that acts on the moon is:

Fb = √FbM2 + FbB2

= √(1.98 × 1020 N)2 + (4.34 × 1020 N)2


= 4.77 × 1020 N

© Nilam Publication Sdn. Bhd. 64


MODUL • Fizik TINGKATAN 4

2 Sebuah satelit berjisim, m = 200 kg mengorbit Bumi dengan jarak 30 000 km diukur dari pusat Bumi.
Tentukan daya graviti pada satelit tersebut. (Jisim Bumi = 5.97 × 1024 kg)
A satellite of mass, m = 200 kg orbits the Earth at a distance of 30 000 km measured from the centre of the Earth.
Determine the gravitational force of the satellite. (Mass of the Earth = 5.97 × 1024 kg)
Diberi, / Given, (G = 6.67 × 10–11 N m2 kg–2)
Satelit
Jisim satelit, / Mass of satellite, ms = 200 kg Satellite
Jarak di antara satelit dengan pusat Bumi,
Distance between satellite and the entre of the Earth, 30 000 km
rs = 30 000 km
= 3 × 107 m
GmBms
F =
r2
Bumi
(6.67 × 10–11 N m2 kg–2)(5.97 × 1024 kg) × (200 kg)
F = Earth
(3 × 107m)2
= 88.49 N

SP 3.1.3 Menghubung kait pecutan graviti, g di permukaan Bumi dengan pemalar kegravitian semesta, G

Hubung Kait antara Pecutan Graviti, g di Permukaan Bumi dengan Pemalar Kegravitian
Semesta, G / Relationship between Gravitational Acceleration, g on the Surface of the Earth with U
U
Universal Gravitational Constant, G N
N
II
T
T
1 Daripada Hukum Gerakan Newton Kedua dan Hukum Kegravitian Semesta Newton, terbitkan hubungan 13
antara g dan G.
From Newton's Second Law of Motion and Newton's Law of Gravitation, derive the relationship between g and G.

Daya graviti yang menyebabkan objek


Hukum Gerakan Newton Kedua,
Newton's Second Law of Motion, F = mg jatuh dengan pecutan graviti Bumi, g
Gravitational force that causes object to fall
with acceleration, g

Daya graviti yang menarik objek ke arah


Hukum Kegravitian Semesta Newton, GMm
F= pusat Bumi
Newton's Universal Law of Gravitation, r2 Gravitational force that pulls object to the centre
of the Earth

➀ = ➁,
GMm g = pecutan graviti / gravitational acceleration
mg = G = pemalar kegravitian semesta / gravitational constant
r2 M = jisim Bumi / mass of Earth
GM m = jisim objek / mass of the object
g= 2 G = jarak di antara pusat Bumi dengan pusat sesuatu objek
r
distance between the centre of the Earth and the centre of the object

65 © Nilam Publication Sdn. Bhd.


MODUL • Fizik TINGKATAN 4

2 Berdasarkan rajah di sebelah, ungkapkan gh dalam sebutan G, M, R dan h, di mana


gh ialah pecutan akibat graviti pada ketinggian, h dari permukaan Bumi,
G ialah pemalar Kegravitian,
M ialah jisim Bumi,
R ialah jejari Bumi. h
r
From the diagram on the right, express gh in terms of G, M, R and h, whereby
gh is the acceleration due to gravity at height, h from the Earth's surface,
G is the Gravitational constant, R
M is the mass of the Earth,
R is the radius of the Earth.
GMm
F = 2
r
Di kedudukan pada ketinggian, h dari permukaan Bumi, jarak dari pusat Bumi ialah
At height, h from the surface of the Earth, the distance from the centre of the Earth is

r = R+ h
GMm
F = –––––– ➀
(R + h)2
F = mgh ––––––––––– ➁

➀ = ➁,
GMm
mgh =
(R + h)2
GM
∴ gh =
(R + h)2
U
N
I Semakin jauh daripada permukaan Bumi, pecutan graviti, g semakin kecil.
T The further from the Earth's surface, the smaller is the gravitational acceleration, g.
3

SP 3.1.4 Mewajarkan kepentingan mengetahui nilai pecutan graviti planet-planet dalam Sistem Suria

Aktiviti / Activity

Arahan: Secara berkumpulan, jalankan satu kajian tentang kepentingan pengetahuan tentang pecutan
graviti planet-planet dalam penerokaan angkasa dan kelangsungan kehidupan. Bentangkan hasil kajian
kumpulan anda dalam bentuk persembahan multimedia di dalam kelas anda.
Instructions: In a group, conduct a research about the importance of knowledge on gravitational acceleration of
planets in space exploration and continuity of life. Present you research in multimedia form in your classroom.

© Nilam Publication Sdn. Bhd. 66


MODUL • Fizik TINGKATAN 4

mv2
SP 3.1.5 Memerihalkan daya memusat dalam sistem gerakan satelit dan planet daya memusat, F =
r

Apakah itu daya Daya memusat merupakan daya yang bertindak ke atas suatu jasad bagi membolehkan
memusat? ia bergerak secara membulat. Daya ini sentiasa menuju ke pusat bulatan ketika jasad
What is centripetal bergerak. / Centripetal force is defined as the force acting on a body that is moving in a circular
force? path that is directed towards the centre of the circle.

Contoh Sebiji bola yang diikat pada seutas tali dan diputarkan, ketegangan (tension) tali
Example gerakan
bertanggungjawab untuk memastikan bola tersebut kekal dalam
membulat . Maka, dalam situasi ini, daya ketegangan tali adalah
daya memusat .
When a ball is tied to a rope and is rotated, the tension of the rope is responsible
for ensuring that the ball remains in circular motion . Therefore, in this situation, the

tension of the rope is the centripetal force .


Pusat bulatan
Centre of circle
v
T = tegangan tali
T tension in rope
v = laju linear
linear speed

Laluan yang dilalui oleh bola


Path travelled by a ball

U
Contoh 1 Apabila kereta bergerak pada lengkung, daya U
N
N
situasi yang geseran yang bertindak ke atas tayar kereta ialah
II
T
T
mengaplikasikan Geseran
daya memusat
daya memusat yang diperlukan untuk gerakan membulat. Friction 13
Examples of As a car moves along a curve, the frictional force
situations which acting upon the tyres of the car provides the centripetal force
apply centripetal required for circular motion.
force
Bulan
2 Apabila Bulan mengorbit Bumi, daya graviti v
Moon
yang bertindak ke atas Bulan memberikan daya memusat Daya memusat
yang diperlukan untuk gerakan membulat. Centripetal force
As the Moon orbits the Earth, the gravitational force Bumi
acting upon the Moon provides the centripetal force required Earth
for circular motion.

Rumus mv2 F = daya memusat / centripetal force, m = jisim / mass,


F = v = laju linear / linear speed, r = jejari bulatan / radius of circle
Formula r
mv2
Daripada rumus daya, F = ma dan rumus daya memusat, F = .
r
mv2
From formula of force, F = ma and formula of centripetal force, F = .
r
mv2
ma =
r
v2 iaitu a ialah pecutan memusat
a= which a is centripetal acceleration
r

67 © Nilam Publication Sdn. Bhd.


MODUL • Fizik TINGKATAN 4

SP 3.1.6 Menentukan jisim Bumi dan Matahari menggunakan rumus Hukum Kegravitian Semesta Newton dan daya memusat

F1 = daya graviti / gravitational force


F2 = daya memusat / centripetal force

Menentukan jisim Bumi dan Matahari / Determine the mass of the Earth and the Sun

Rajah di bawah menunjukkan orbit Bulan mengelilingi Bumi.


The diagram below shows the orbit of the Moon around the Earth.

v Bulan/Moon
m

Orbit Bulan r
Orbit of
the Moon
M
Bumi/Earth

Jarak Bulan membuat satu orbit lengkap mengeliling Bumi = 2πr (lilitan)
Distance by the Moon to make one complete orbit around the Earth = 2πr (circumference)

Laju linear Bulan, v = 2πr


Linear speed of the Moon, T

U Daripada Hukum Kegravitian Semesta Newton dan daya memusat, rumus bagi jisim Bumi, M dapat ditentukan.
N From Newton's Universal Law of Gravitation and centripetal force, formula of mass of the Earth, M can be determined.
I
T GMm
3 F=
r2
–––––– ➀

mv 2
F= –––––– ➁
r2
➀=➁
GMm mv 2
=
r2 r2
GM
r2
=
v2
r2
gantikan
substitute (v = 2πr
T )
GM (2πr/T) 2
=
r2 r
2 2
GM 4π r
=
r2 T 2r
4π2r 3
GM =
T2
4π2r 3 rumus untuk menentukan jisim Bumi atau Matahari
∴M =
GT 2 formula to determine the mass of the Earth or the Sun

© Nilam Publication Sdn. Bhd. 68


MODUL • Fizik TINGKATAN 4

3.2 HUKUM KEPLER / KEPLER’S LAWS SK 3.2

SP 3.2.1 Menjelaskan Hukum Kepler I, II dan III

Semasa di Tingkatan Rendah, anda telah mengetahui mengenai Kepler, seorang ahli astronomi, matematik dan
astrologi Jerman yang menemui 3 Hukum Kepler. / When you were in Lower Form, you knew about Kepler, a German
astronomist, mathematician and astrologist who came up with 3 Kepler's Laws.

Hukum Kepler Orbit bagi setiap planet adalah elips dengan Matahari berada di satu daripada fokusnya.
Pertama All planets move in elliptical orbits with the Sun at one focus (Law of Orbits).
Kepler's First Law
Hukum Kepler Garis yang menyambungkan planet dengan Matahari akan mencakupi luas yang sama
Kedua / Kepler's dalam selang masa yang sama apabila planet bergerak dalam orbitnya.
Second Law A line that connects a planet to the Sun sweeps out equal areas in equal times (Law of Areas).
Hukum Kepler Kuasa dua tempoh orbit planet adalah berkadar terus dengan kuasa tiga jejari orbitnya.
Ketiga The square of the orbital period of any planet is directly proportional to the cube of the radius
Kepler's Third Law of its orbit (Law of Periods).

Setiap planet bergerak dengan lintasan berbentuk elips dengan Matahari berada
pada salah satu titik fokusnya . Akibatnya jarak planet ke Matahari berubah-ubah
sehingga terdapat suatu tempat di mana planet paling dekat dengan Matahari (titik
Aphelion) dan paling jauh dari Matahari (titik Perihelion).
Penjelasan Each planet moves in an ellipse orbit with the Sun at one of its focal points . As
tentang Hukum a result, the distance of the planet to the Sun varies so that there is a place where the planet is
Kepler I the closest to the Sun (the Aphelion point) and the farthest from the Sun (the Perihelion point).
Explanation of
U
Kepler's First Law U
N
N
Matahari / Sun II
Titik Aphelion T
T
Aphelion point
Planet
Planet
Titik Perihelion
Perihelion point
13

Garis hubung planet dengan Matahari mencakupi luas yang sama dalam
selang masa yang sama. Dengan kata lain, pecutan planet nilainya berubah-ubah
sesuai dengan posisinya. / The planet's connecting line with the Sun sweeps out equal areas
in equal times . In other words, the acceleration of a planet varies according to its position.
Planet
A Jika luas AMB = luas CMD, maka
Penjelasan D selang masa yang diperlukan planet
tentang Hukum Matahari untuk bergerak dari A ke B sama
Sun
Kepler II M dengan selang masa dari C ke D.
Explanation of If area of AMB = area of CMD, then the
Kepler's Second amount of time it takes for the planet
C
Law to move from A to B equals to the
B amount of time from C to D.

Perhatikan bahawa garis AB lebih panjang daripada garis CD. Oleh kerana itu, laju linear
planet di AB lebih besar daripada laju linear di CD. Kedua-dua luas ini mempunyai
luas yang sama. Pada selang masa yang sama, garis penghubung planet dan Matahari
mencakupi luas yang sama.
Observe that the line AB is longer than the line CD. Therefore, the linear speed of the planet
in AB is greater than its linear speed in CD. The two areas 'swept out' are equal.

69 © Nilam Publication Sdn. Bhd.


MODUL • Fizik TINGKATAN 4

Planet yang mengorbit dengan jejari orbit yang lebih besar mempunyai tempoh orbit
yang lebih besar.
A planet orbiting with a bigger radius of orbit has a bigger orbital period.
Penjelasan
tentang Hukum Secara matematik, / Mathematically,
Kepler III
Explanation of
T2 =
4π2 3
GM ( )
r
Kepler's Third Law
T 2 ∝ r3
T = tempoh orbit planet / orbital period of a planet
r = jejari orbit / radius of orbit

SP 3.2.2 Merumuskan Hukum Kepler III, T2 ∝ r3

T1 = Tempoh planet pertama. / The period of the first planet.


T2 = Tempoh planet kedua. / The period of the second planet.
r1 = Jarak planet pertama dari Matahari. / Distance of first planet from the Sun.
r2 = Jarak planet kedua dengan Matahari. / Distance of second planet from the Sun.
k = Pemalar yang nilainya sama untuk semua planet. / The constant with the same value for all planets.

Misalnya, dua planet mempunyai jarak purata dari Matahari ialah r1 and r2 dan tempoh ialah T1 dan T2 masing-
masing. / Suppose two planet have an average distance from the Sun, r1 and r2 , and the periods are T1 and T2 respectively.
Daripada Hukum Kepler Ketiga, / From Kepler's Third Law,

T12 =
GM ( )
4π2 3
r

U
Planet 1, T12 = ( )
4π2 3
GM 1
r –––––– ➀
k(pemalar / constant) =
4π2
N
I
T
Planet 2, T22 =
GM ( )
4π2 3
r 2 –––––– ➁
GM

3 ➀÷➁∴
T12
=
r13 rumus untuk menghitung tempoh orbit, T atau jejari orbit, r
T22 r23 formula to calculate orbital period, T or radius of orbit, r

SP 3.2.2 Menyelesaikan masalah menggunakan rumus Hukum Kepler Ketiga

Latihan / Exercises
K J
1 Perhatikan rajah lintasan orbit suatu planet mengelilingi Matahari L I
di sebelah. Pada posisi apakah planet bergerak paling cepat? H
Observe the diagram of the orbit of a planet around the Sun on the
right. At what position is the planet moving the fastest? Planet O Matahari
A G
A A ke B / A to B
B B ke C / B to C
C H ke I / H to I F
D I ke J / I to J E
B D
C

Penerangan / Explanation
Planet bergerak lebih cepat saat posisinya lebih dekat ke Matahari, jadi posisi
planet dari L ke A atau posisi planet dari A ke B merupakan posisi di mana planet
bergerak paling cepat.
The planet moves faster when its position is closer to the Sun, so the position of the planet
from L to A or the planet from A to B is the position where the planet moves the fastest.

© Nilam Publication Sdn. Bhd. 70


MODUL • Fizik TINGKATAN 4

2 Perhatikan rajah lintasan orbit suatu planet mengelilingi Matahari di atas. Antara pernyataan berikut, yang
manakah benar tentang luasan antara titik A-B-O dan H-I-O?
Observe the diagram of the orbit of a planet around the Sun above. Which of the following statements is true about the
area between points A-B-O and H-I-O?
A Luasan A-B-O lebih besar dari H-I-O. / Area A-B-O is wider than H-I-O.
B Luasan H-I-O lebih besar dari A-B-O. / Area H-I-O is wider than A-B-O.
C Keduanya memiliki luas yang sama. / Both have the same area.
D Kurang cukup maklumat untuk menentukan luasan. / Not enough information to determine the areas.

Penerangan / Explanation
Berdasarkan Hukum Kepler II, nilai luasan titik garis khayal pada orbit adalah malar
di mana planet berada pada orbitnya diukur berdasarkan selang masa yang sama.
According to Kepler's Second Law, the area of an imaginary point line in its orbit is constant
wherever the planet is in its orbit measured by the same time interval.

3 Tempoh Bumi mengelilingi Matahari adalah 1 tahun dan tempoh Jupiter adalah sekitar 12 tahun. Cari jarak
planet Jupiter ke Matahari. Berikan jawapan dalam sebutan rB. / The period of orbit of the Earth around the Sun
is about 1 year and that of Jupiter around the Sun is 12 years. Find the distance of the Jupiter to the Sun.
Penyelesaian / Solution:
TJ = Tempoh Jupiter mengelilingi Matahari. / Period of orbit of the Jupiter around the Sun.
TB = Tempoh Bumi mengelilingi Matahari. / Period of orbit of the Earth around the Sun.
rB = Jarak Bumi dari Matahari. / Distance between the Earth and the Sun.
rJ = Jarak Jupiter dari Matahari. / Distance between Jupiter and the Sun.
TJ2 rJ 3
=
TB2 rB3
(12 tahun / years)2 × rB3
rJ 3 =
(1 tahun / year)2 U
U
N
N
rJ 3 = 144 rB3 II
T
T
3
rJ = √144 rB
13

4 Andaikan jarak dari Bumi ke Bulan adalah 3.84 × 108 m dan tempoh Bulan mengelilingi Bumi ialah 2.5 × 106 s.
Hitung jisim Bumi. / Assume the distance from the Earth to the Moon is 3.84 × 108 m and the period of the Moon
orbiting the Earth is 2.5 × 106 s. Calculate the mass of the Earth.
Rumus / Formula:
T2 4π2
3 =
r GM
4π2r 3
M =
GT 2
4π2 (3.84 × 108 m)3
M=
(6.7 × 10–11 N m2 kg–2)(2.5 × 106 s)2
M = 5.34 × 1024 kg

71 © Nilam Publication Sdn. Bhd.


MODUL • Fizik TINGKATAN 4

3.3 SATELIT BUATAN MANUSIA / MAN-MADE SATELLITES SK 3.3

SP 3.3.1 Menerangkan bagaimana orbit satu satelit dikekalkan pada ketinggian tertentu dengan menggunakan halaju satelit yang sesuai

Satelit buatan diluncurkan ke luar angkasa dan mengorbit mengelilingi planet dengan
tujuan untuk memberi manfaat tertentu. Satelit buatan manusia yang pertama kali
mengorbit di Bumi adalah satelit buatan Rusia bernama Sputnik 1 yang diluncurkan
pada 4 Oktober 1957. Satelit yang mengelilingi Bumi tidak akan menjadi perlahan
atau jatuh ke Bumi. Satelit buatan manusia bergerak dalam orbit pada ketinggian
tertentu dengan laju linear satelit yang sesuai. Rujuk Rajah 3.1. Kedua-dua konsep daya
memusat dan Hukum Kegravitian Semesta Newton digunakan untuk mementukan laju
linear satelit buatan. Rujuk Rajah 3.2 yang menunjukkan sebuah satelit buatan manusia
yang mengelilingi Bumi.
Man-made satellites are launched into space and orbit around the Earth for certain purposes.
The first man-made satellite orbiting the Earth was a Russian-made satellite called Sputnik 1 that
launched on October 4, 1957. The satellites around the Earth will not slow down or collapse.
Man-made satellites move in orbits at specific heights and suitable linear speeds. Refer to
Diagram 3.1. Both concepts of centripetal force and Newton's Law of Universal Gravitation are
used to calculate the linear speed of satellites. Refer to Diagram 3.2 which shows a man-made
satellite orbiting the Earth.
Satelit buatan
manusia Satelit
Satellites
Man-made satellite

160 km
U
N 2 200 km M
I
T 35 786 km
3 Orbit rendah
Bumi
Earth
Low orbit
Orbit sederhana
Medium orbit
Orbit geosegerak Satelit
Geosynchronous orbit Satellites
Rajah 3.1 / Diagram 3.1 Rajah 3.2 / Diagram 3.2

Satelit-satelit pada ketinggian Orbit sebuah satelit


orbit yang berlainan Orbit of a satellite
Satellities at different heights
of orbit

Dengan menggunakan Hukum Kegravitian Semesta Newton, F1, dan daya memusat,
F2, laju linear satelit dapat ditentukan. / Using Newton's Universal Law of Gravitation, F1
and centripetal force, F2 the linear speed of satellite can be determined.
F1 = F2 F1 = Daya graviti antara dua jasad
Rumus laju
mv2
GMm
Gravitational force between two bodies
linear satelit 2 =
Formula of linear r r F2 = Daya memusat / Centripetal force
M = Jisim Bumi / Mass of the Earth
speed of satellite GM
v2 = r = Jarak antara dua jasad / Distance between two bodies
r v = Laju linear satelit / Linear speed of satellite
GM
v =
r √ G = Pemalar kegravitian / Gravitational constant
m = Jisim satelit / Mass of satellite

© Nilam Publication Sdn. Bhd. 72


MODUL • Fizik TINGKATAN 4

Buat masa ini, Malaysia hanya memiliki satelit komunikasi milik swasta,
Berikan contoh
satelit yang Malaysia East Asia Satellite (MEASAT) iaitu MEASAT-1 dan MEASAT-2 yang
dimiliki oleh dilancarkan pada tahun 1996, diikuti MEASAT-3 (2006), MEASAT-3a (2009) dan
Malaysia. terbaharu MEASAT-3b (2014).
Give examples of Currently, Malaysia has only privately owned communications satellites, Malaysia East
satellites owned by Asia Satellite (MEASAT) is MEASAT-1 and MEASAT-2 launched in 1996, followed by
Malaysia.
MEASAT-3 (2006), MEASAT-3a (2009) and the latest MEASAT-3b (2014).

SP 3.3.2 Berkomunikasi untuk menerangkan satelit geopegun dan bukan geopegun

Jenis Satelit Penerangan


Type of satellite Explanation

Orbit geopegun ialah di mana satelit selalunya berada sama dengan posisi dengan
Bumi yang bergerak. Merupakan orbit yang berputar dengan lengkap dalam
24 jam iaitu kadar putarannya sama dengan Bumi. Apabila bergerak dengan halaju
dan arah yang sama dengan Bumi, satelit ini akan kelihatan ‘pegun’. Ia biasanya
berputar mengelilingi Bumi pada garisan khatulistiwa . Jaraknya dari Bumi adalah
35 800 km atau 22 000 batu. Biasanya berbentuk bulat. Kebanyakan satelit yang
dihantar berada pada orbit jenis ini.
Untuk menjadi orbit geopegun, mesti memenuhi 3 kriteria:
• Orbit mesti geosinkronous/geosegerak .
Satelit Geopegun • Orbit mesti bulat.
Geostationary • Orbit mesti berputar pada landasan khatulistiwa Bumi.
satellite Geostationary orbits are orbits where satellites are synchronous with the Earth that is U
U
N
N
II
moving. It is a fully rotating orbit in 24 hours whose period is the same as the Earth's. When T
T
travelling at the same speed and direction as the Earth, this satellite will appear stationary. It
13
usually revolves around the Earth on the equator . Its distance from the Earth is 35 800 km
or 22 000 miles. They are usually round. Most transmitted satellites are in this orbit.

To become a geostationary orbit, it must meet 3 criteria:


• Orbits must be geosynchronous .
• Orbit must be round.
• The orbit must rotate on the equator of the Earth.

Satelit jenis ini biasanya berada dalam orbit lebih rendah atau lebih tinggi daripada
orbit Bumi geopegun. Ia mempunyai tempoh orbit yang lebih pendek atau
Satelit bukan lebih panjang daripada 24 jam. Ia berada di atas kedudukan geografi yang
geopegun berubah-ubah di permukaan Bumi.
Non-geostationary
satellites This kind of satellites normally in a lower or higher orbit than the Geostationary Earth Orbit.
Its orbital period is shorter or longer than 24 hours. It is above different

geographical locations at different times.

73 © Nilam Publication Sdn. Bhd.


MODUL • Fizik TINGKATAN 4

Perbandingan antara satelit geopegun dan satelit bukan geopegun


Comparison between geostationary satellites and non-geostationary satellites

Arah gerakan Arah gerakan tidak


sama dengan arah perlu sama dengan
putaran Bumi arah putaran Bumi
Direction of motion Direction of motion
same as the need not be the same lebih
T
direction of Earth as the direction of
rotation pendek atau
Earth rotation
lebih panjang
daripada 24 jam
24 jam T is shorter
T= Orbit
24 hours mengelilingi or longer
Bumi than 24 hours
Orbits the Earth

Berada di atas Berada di atas


tempat yang tempat yang
sama berubah-ubah
Satelit Satelit bukan


di muka Bumi GM geopegun di muka Bumi
geopegun v= r Above
Above Geostationary Non-geostationary
the same satellite satellite different
U
N geographical geographical
I location
T location

• Pengimejan Bumi


4π 2r3 Earth imaging
Satelit T=
GM • GPS
komunikasi
Communication • Kaji cuaca
satellite
Weather forecast

• TiungSAT
• RazakSAT
MEASAT
• Pipit
• ISS

© Nilam Publication Sdn. Bhd. 74


MODUL • Fizik TINGKATAN 4

SP 3.3.3 Mengkonsepsikan halaju lepas

Halaju lepas, v ialah halaju minimum yang diperlukan oleh satu objek di permukaan
Berikan definisi
Bumi untuk mengatasi daya graviti dan terlepas ke angkasa lepas.
halaju lepas
Give the definition minimum velocity
Escape velocity, v is the required by an object on the Earth's surface to
of escape velocity
overcome the gravitational force and escape to outer space.

Halaju lepas akan dicapai apabila tenaga kinetik minimum yang dibekalkan kepada
objek telah mengatasi tenaga keupayaan graviti. Untuk jasad simetri sfera, halaju
Perkaitan antara lepas pada jarak yang diberikan adalah dikira berdasarkan rumus berikut:
halaju lepas
dengan tenaga Tenaga keupayaan graviti = Tenaga kinetik minimum
Relation between The escape velocity is achieved when the minimum kinetic energy supplied to the object
escape velocity and
energy exceeds the gravitational potential energy. For spherical symmetry bodies, the velocity
travelled at a given distance is calculated based on the following formula:

Gravitational potential energy = Minimum kinetic energy

Magnitud tenaga keupayaan graviti, U, pada satu objek pada jarak dari pusat Bumi, r
adalah:
The magnitude of gravitational potential energy, U, of an object at a distance from the centre
of the Earth, r is:
GMm
U=
r
1
Tenaga kinetik / Kinetic energy = mv2 di mana / where; m = jisim objek / mass of object
2 U
U
Rumus M = jisim Bumi / mass of the Earth N
N
II
Formula v = halaju lepas / escape velocity T
T
G = pemalar kegravitian / gravitational constant (G = 6.67 × 10–11 N m2 kg−2) 13
Hasil tambah tenaga kinetik dan tenaga keupayaan graviti adalah bersamaan sifar.
The addition of kinetic energy and gravitational potential energy equals to zero.
1 GMm
mv2 =
2 r
2GM
v = √r

Mengapa Atmosfera Bumi ialah lapisan yang sangat nipis mengelilingi Bumi. Disebabkan daya
Bumi boleh graviti Bumi yang kuat, ia boleh mengekalkan lapisan atmosferanya.
mengekalkan The atmosphere is a very thin layer around the Earth. Due to the strong gravitational force of
lapisan the Earth, it can maintain its atmospheric layer.
atmosferanya?
Why the Earth
can maintain its
atmospheric layer?

75 © Nilam Publication Sdn. Bhd.


MODUL • Fizik TINGKATAN 4

Mengapakah Bumi berputar pada kelajuan 1 039 batu sejam di garisan khatulistiwa . Kapal
kapal terbang terbang tidak dapat terlepas dari Bumi kerana putaran Bumi sangat perlahan. Bumi
tidak terlepas 1
dari Bumi berputar pada putaran 1 setiap 24 jam. Itulah kelajuan putaran sejam.
24
–1
berdasarkan Halaju lepas Bumi = 11.2 km s .
halaju lepas
The Earth spins at a velocity of 1 039 miles per hour at the equator . The plane cannot
Bumi?
Why airplanes did escape from the Earth because the rotation of the Earth is very slow. Earth spins at a rotation
not escape from 1
of 1 per 24 hours. That is a rotation speed of th per hour.
the Earth based on 24
–1
the escape velocity Escape velocity of the Earth = 11.2 km s .
of the Earth?

SP 3.3.4 Menyelesaikan masalah yang melibatkan halaju lepas, v bagi roket dari permukaan Bumi, Bulan dan Marikh dan Matahari

Latihan / Exercises

1 Jejari Bumi ialah 6.37 × 106 m dan jisim Bumi ialah 5.97 × 1024 kg. Berapakah halaju lepas Bumi?
The radius of the Earth is 6.37 × 106 m, and the mass of the Earth is 5.97 × 1024 kg. What is the escape velocity from
the Earth?


2GM
vescape =
R


= 2(6.67 × 10–11 N m2 kg–2)(5.97 × 1024 kg)
6.37 × 106 m
U = √ 1.25 × 108 N m2 kg–1
N
I
T = 11 181 m s–1
3

2 Untuk meninggalkan Bulan, angkasawan Apollo perlu keluar dari modul lunar dan mencapai halaju lepas
Bulan. Jejari Bulan ialah 1.74 × 106 m dan jisim Bulan ialah 7.35 × 1022 kg. Berapakah halaju lepas yang
perlu dicapai oleh Neil Amstrong dan Buzz Aldrin dalam modul lunar untuk meninggalkan Bulan? Halaju
lepas dari Bumi ialah 11 184 m s–1 iaitu kira-kira 11.2 km s–1.
To leave the Moon, the Apollo astronauts had to take off in the lunar module, and reach the escape velocity of the Moon.
The radius of the Moon is 1.74 × 106 m, and the mass of the Moon is 7.35 × 1022 kg. What was the escape velocity that
Neil Armstrong and Buzz Aldrin needed to have in the lunar module to leave the Moon? The escape velocity from the
Earth is 11 184 m s–1, or approximately 11.2 km s–1.


2GM
vescape =
R


= 2(6.67 × 10–11 N m2 kg–2)(7.35 × 1022 kg)
1.74 × 106m
= √ 5.635 × 106 N m2 kg–1

= 2 374 m s–1

© Nilam Publication Sdn. Bhd. 76


MODUL • Fizik TINGKATAN 4

3 Hitung halaju lepas bagi dari permukaan Marikh. Halaju lepas Bulan ialah 2 374 m s–1 atau kira-kira
2.37 km s–1.
Calculate the escape velocity from the surface of Mars. The escape velocity from the Moon is 2 374 m s–1, or approximately
2.37 km s–1.
MMars = 6.42 × 1023 kg
RMars = 3.4 × 106 m


vescape = 2GM
R


vescape = 2(6.67 × 10–11 N m2 kg–2)(6.42 × 1023 kg)
3.4 × 106 m
vescape = 5.02 × 103 m s–1
= 5.02 km s–1

4 Berapakah jejari Schwarzschild bagi objek yang mempunyai jisim yang sama dengan jisim Matahari (iaitu,
jika anda boleh mengecilkan Matahari, pada titik apakah ia akan menjadi lubang hitam?
Andaikan bahawa kita mengekalkan jisim Matahari adalah sama tetapi kurangkan jejari Matahari supaya
halaju lepas menjadi lebih besar daripada kelajuan cahaya (300 000 km s–1 = 3.0 × 108 m s–1). Selesaikan
persamaan halaju lepas untuk R.
What would be the Schwarzschild radius for an object with the same mass as the Sun (i.e., if you could shrink the Sun,
at what point would it become a black hole)?
Assume that we keep the Sun's mass the same but reduce the Sun's radius such that the escape velocity becomes greater U
U
than the speed of light (300 000 km s–1 = 3.0 × 108 m s–1). We solve the escape velocity equation for R: N
N
II
T
T
vescape = (
2GM 12
R ) 13
2GM
v2escape =
R
2GM
R = 2
v escape
2(6.67 × 10–11 N m2 kg–2)(1.99 × 1030 kg)
R =
(3.0 × 108 m s–1)2
R
= 2.95 km

77 © Nilam Publication Sdn. Bhd.


MODUL • Fizik TINGKATAN 4

Praktis SPM / SPM Practice


Soalan Objektif / Objective Questions

1 Hitungkan pecutan graviti di permukaan Bumi yang 3 Hitung daya memusat yang dikenakan ke atas
mana jisim Bumi ialah 5.97 × 1024 kg dan berjejari 900 kg kereta yang melalui satu lengkungan yang
6.37 × 106 m. [G = 6.67 × 10–11 N m2 kg–2] rata berjejari 500 m pada halaju 25 m s–1. / Calculate
Calculate the gravitational acceleration on the surface the centripetal force exerted to the car of 900 kg through
of the Earth where the mass of the Earth is 5.97 × 1024 kg a flat curve with radius of 500 m at velocity of 25 m s–1.
and the radius is 6.37 × 106 m.
[G = 6.67 × 10–11 N m2 kg–2]
Rumus yang boleh diguna:
Formula that can be used:
r = 500 m mv2
R Rumus yang boleh diguna: F=
r
Formula that can be used: 25 m/s
GM
g=
R2
Rajah 3 / Diagram 3
A 90 N
Bumi / Earth
Rajah 1 / Diagram 1 B 1.125 × 103 N
A 8.9 m s–2 C 1.125 × 102 N
B 8.8 m s–2 D 1.25 × 103 N
C 9.8 m s–2
D 7.9 m s–2 4 Seorang atlit balingan tukul besi dalam sukan
SUKMA sedang memutarkan tukul besi yang
2 Sebuah satelit komunikasi mengorbit mengelilingi berjisim 2 kg dalam satu bulatan ufuk dengan halaju
Bumi pada ketinggian 35 000 km. Hitung pecutan 5 m s–1. Hitung daya memusat yang bertindak ke
U
graviti pada kedudukan satelit itu. atas tukul besi tersebut. / A hammer throw athlete is
N swinging the iron ball of mass 2 kg in a horizontal circle
I [G = 6.67 × 10–11 N m2 kg–2, M = 5.97 × 1024 kg,
T with a speed of 5 m s–1 in the SUKMA games. Calculate
R = 6.37 × 106 m]
3 A communication satellites orbits around the Earth at
the centripetal force that acts on the iron ball.

the height of 350 000 km. Calculate the gravitational 2m Rumus yang boleh diguna:
acceleration at the position of the satellite. Formula that can be used:
[G = 6.67 × 10–11 N m2 kg–2, M = 5.97 × 1024 kg, mv2
F=
R = 6.37 × 106 m] r
Satelit
Satellite
Rajah 4 / Diagram 4
A 40 N C 60 N
h = 35 000 km B 10 N D 100 N
r
Rumus yang boleh diguna:
R
Formula that can be used: 5 Planet Bumi dan Zuhrah yang mengorbit
GM mengelilingi Matahari. Cari tempoh orbit Zuhrah.
g=
(R + h)2 [Jejari orbit Bumi, r = 1.50 × 1011 m, tempoh
Bumi / Earth orbit Bumi, TB = 365 hari, jejari orbit Zuhrah,
rZ = 1.08 × 1011 m]
The Earth and the Venus orbits around the Sun. Find
Rajah 2 / Diagram 2 the orbital period of Venus. [Radius of orbits of Earth,
A 0.233 m s–2 r = 1.50 × 1011 m, period of orbit of the Earth, TB = 365
B 2.327 m s–2 days, radius of orbit of Venus, rZ = 1.08 × 1011 m]
C 0.223 m s–2 A 223 hari / days Rumus yang boleh diguna:
D 23.27 m s–2 B 213 hari / days Formula that can be used:
C 322 hari / days TZ2 r3
= Z
D 323 hari / days TB 2
rB3

© Nilam Publication Sdn. Bhd. 78


MODUL • Fizik TINGKATAN 4

6 Satelit astronomi merupakan sebuah teleskop 8 Satelit astronomi merupakan sebuah teleskop
mengelilingi Bumi yang digunakan untuk mengelilingi Bumi yang digunakan untuk
memetakan permukaan planet yang berbeza. memetakan permukaan planet yang berbeza.
Ketinggiannya dari Bumi dalam lingkungan 600 km Ketinggiannya dari Bumi dalam lingkungan
dengan laju linear ialah 7.62 × 103 m s–1. Berapakah 600 km. Berapakah laju linear satelit?
pecutan memusat? [R = 6.37 × 106 m] [Jejari Bumi, R = 6.37 × 106 m,
Astronomical satellite is a telescope orbits around the G = 6.67 × 10–11 N m2 kg–2, M = 5.97 × 1024 kg]
Earth that is used to capture the clear images of the Astronomical satellite is a telescope orbits around the
surface of different planets. Its height from the Earth is Earth that is used to capture the clear images of the
within 600 km with the linear speed of 7.62 × 103 m s–1. surface of different planets. Its height from the Earth is
What is the centripetal acceleration? [R = 6.37 × 106 m] within 600 km. What is the linear speed of the satellite?
[The radius of the Earth, R = 6.37 × 106 m,
Satelit astronomi
Astronomical satellite G = 6.67 × 10–11 N m2 kg–2, M = 5.97 × 1024 kg]
Rumus yang boleh diguna:
Formula that can be used: Satelit astronomi
v2 Astronomical satellite
a= Rumus yang boleh diguna:
R+h Formula that can be used:
Jejari orbit satelit, GM
a=
Radius of satellite's orbit, r2
r=R+h Jejari orbit,
Orbit satelit astronomi Radius of orbit,
Orbit of astronomical
satellite r = (R + h)
Orbit satelit astronomi
Rajah 5 / Diagram 5 Orbit of astronomical
satellite
A 8.0 m s–2
B 7.14 m s–2 Rajah 7 / Diagram 7
C 8.33 m s–2 A 8.63 × 103 m s–1
D 7.84 m s–2 B 8.06 × 103 m s–1
C 9.47 × 103 m s–1
7 Rajah 6 menunjukkan Bumi dan Marikh yang D 9.80 × 103 m s–1
U
U
N
N
mengorbit mengelilingi Matahari. Cari tempoh II
T
orbit Marikh. T
[Jejari orbit Bumi, r = 1.50 × 1011 m, tempoh orbit 13
Bumi, TB = 365 hari, jejari orbit Marikh,
rM = 2.28 × 1011 m]
Diagram 6 shows the Earth and the Mars that orbits
around the Sun. Find the orbital period of the Mars.
[Radius of orbit of the Earth, r = 1.50 × 1011 m, orbital
period of the Earth, TB = 365 days, radius of the Mars,
rM = 2.28 × 1011 m]

Matahari
Sun Bumi / Earth

Marikh / Mars

Rajah 6 / Diagram 6
A 400 hari / days
B 648 hari / days Rumus yang boleh diguna:
Formula that can be used:
C 548 hari / days TM2 r 3
D 684 hari / days = M
2
TB rB3

79 © Nilam Publication Sdn. Bhd.


MODUL • Fizik TINGKATAN 4

Soalan Struktur / Structure Question

1 Bulan dan Marikh ialah dua jasad dalam sistem Suria. Jadual 1 menunjukkan jisim dan jejari Bulan dan Marikh.
[G = 6.67 × 10–11 N m2 kg–2]
The Moon and the Mars are two bodies in the Solar System. Table 1 shows the mass and the radius of the Moon and the
Mars. [G = 6.67 × 10–11 N m2 kg–2]

Planet / Planet Jisim, m/kg / Mass, m/kg Jejari, R/m / Radius, R/m
Marikh / Mars 6.39 × 1023 3.39 × 106
Bulan / Moon 7.35 × 1022 1.74 × 106

Jadual 1 / Table 1
(a) Cari pecutan graviti di Bulan dan di Marikh.
Find the gravitational acceleration in the Moon and the Mars.
Bulan / Moon
–11 2 –2 Rumus yang boleh diguna:
gB = (6.67 × 10 N m kg 6)(7.35 × 1022 kg) Formula that can be used:
(1.74 × 10 ) m
2
GM
g=
–2
gB = 1.62 m s R2

Marikh / Mars
–11 2 –2
gM = (6.67 × 10 N m kg 6)(6.39 × 1023 kg)
(3.39 × 10 )2 m
–2
gM = 3.71 m s

U (b) Cari halaju lepas dari Bulan dan dari Marikh.


N
I Find the escape velocity from the Moon and from the Mars.
T
Bulan / Moon
3 Rumus yang boleh diguna:


vB = 2(6.67 × 10–11 N m2 kg–2)(7.35 × 1022 kg) Formula that can be used:


1.74 × 106 m v = 2GM
3 –1
vB = 2.37 × 10 m s R

Marikh / Mars


vM = 2(6.67 × 10–11 N m2 kg–2)(6.39 × 1023 kg)
3.39 × 106 m
vM = 5.01 × 103 m s–1

(c) Bandingkan pecutan graviti dan halaju lepas di Bulan dan di Marikh.
Compare the gravitational acceleration and the escape velocity of the Moon and the Mars.

Pecutan graviti dan halaju lepas di Bulan lebih kecil berbanding pecutan dan halaju lepas di Marikh

kerana jisim Bulan lebih kecil daripada jisim Marikh.

The gravitational acceleration and the escape velocity of the Moon is smaller than the gravitational acceleration

and the escape velocity of the Mars because the mass of the Moon is smaller than the mass of the Mars.

© Nilam Publication Sdn. Bhd. 80


MODUL • Fizik TINGKATAN 4

Unit
HABA
4 HEAT

4.1 KESEIMBANGAN TERMA / THERMAL EQUILIBRIUM SK 4.1

SP 4.1.1 Menjelaskan melalui contoh keseimbangan terma dalam kehidupan harian

1 Ayat di bawah menerangkan tentang haba, Q. Haba membolehkan sesuatu


The sentences below explain about heat, Q. objek menukar suhu atau
(a) Haba adalah suatu bentuk tenaga yang mengalir dari kawasan panas ke mengubah keadaan fizikal:
Heat makes an object to
kawasan sejuk. change its temperature or
change its physical state:
Heat is a form of energy flows from a hot region to a cool region. (a) Jika keadaan fizikal tidak
(b) Haba boleh memanaskan atau menyejukkan objek. berubah, suhunya akan
berubah
Heat can heat up or cool down an object. If the physical state is
(c) Untuk mengetahui suatu objek panas atau sejuk, ukur suhu objek tersebut. unchanged, the temperature
changes
To know whether an object is hot or cool, measure the temperature of the object. (b) Jika suhu tetap, keadaan
(d) Unit S.I. bagi haba: Joule (J). / S.I. unit of heat: Joule (J). fizikal akan berubah
If the temperature is
constant, the physical state
2 Lengkapkan ayat di bawah yang menerangkan tentang suhu, T. changes
Complete the sentences below that explain about temperature, T. Pertukaran Unit Suhu:

(a) Suhu ditakrifkan sebagai darjah kepanasan objek . Change Unit of Temperature:
θ ºC = (θ + 273) K

Temperature is defined as degree of hotness of an object. Contoh: / Example:


(i) 0 ºC = (0 + 273) K
(b) Unit S.I. bagi suhu: Kelvin (K) / S.I. unit for temperature: Kelvin (K) = 273 K
(ii) 27 ºC = (27 + 273) K
= 300 K
(iii) 100 ºC = (100 + 273) K
= 373 K
Aktiviti : Mengkaji keseimbangan terma di antara dua jasad yang
bersentuhan secara terma 100 ºC 373 ºK
Activity : To study thermal equilibrium between two bodies in thermal contact

U
U
Kaki retort 27 ºC 300 ºK N
Retort stand N
II
T
T
14
Termometer A Termometer B 0 ºC 273 ºK
Thermometer A Thermometer B

Kertas tisu / Tissue paper


Air paip / Tap water Air panas / Hot water
Bikar A / Beaker A Bikar B / Beaker B

Prosedur / Procedure
1 Bikar A dibalut dengan tisu dan diisi dengan 150 ml air paip.
Beaker A is wrapped with tissue and filled with 150 ml of tap water.
2 Bikar B diisi 40 ml air panas pada suhu 50 oC. Bikar B dimasukkan ke dalam bikar A.
Beaker B is filled with 40 ml of hot water at temperature 50 °C. Beaker B is inserted into beaker A.
3 Dua termometer berlabel A dan B dimasukkan ke dalam setiap bikar A dan bikar B. Bacaan termometer A
dan termometer B direkod dalam sela masa 30 s sehingga bacaan kedua-dua termometer menjadi sama.
Two thermometers labelled as A and B were inserted into beaker A and beaker B respectively. The readings of thermometer
A and B were recorded for every 30 s until the readings of both temperatures are the same.

81 © Nilam Publication Sdn. Bhd.

B4FizikF4(81-114)csy5p.indd 81 09/01/2020 9:57 AM


MODUL • Fizik TINGKATAN 4

Keputusan / Results

Masa, t (s) Suhu termometer A (°C) Suhu termometer B (°C)


Time, t (s) Temperature of thermometer A (°C) Temperature of thermometer B (°C)
0
30.0
60.0

Perbincangan / Discussions:
1 Bikar A dibalut dengan kertas tisu bagi menghalang haba dibebaskan ke persekitaran.
Beaker A is wrapped with tissue paper to avoid heat being released to the surrounding.

2 Bacaan suhu termometer B semakin menurun manakala bacaan suhu termometer A semakin
meningkat sehingga kedua-dua bacaan termometer menjadi sama .
The reading of temperature of thermometer B is decreasing while reading of temperature of thermometer A is
increasing until both temperatures become the same .

3 Haba dari air dalam bikar B dipindah ke dalam bikar A sehingga pengaliran haba menjadi
sifar .
Heat from water in beaker B is being transferred to beaker A until the net heat transfer is zero .

Definisi keseimbangan terma


Definition of thermal equilibrium
Dalam keseimbangan terma
At thermal equilibrium

A B A B

U Panas Sejuk Haba yang dibebaskan = Haba yang diserap (Kadar bersih pemindahan haba adalah SIFAR)
N Heat released = Heat absorbed (Rate of net heat transferred is ZERO)
I Hot Cold
T

4 • Dua objek berlainan suhu dalam sentuhan terma akan mencapai keseimbangan terma apabila:
Two objects with different temperature in thermal contact will reach thermal equilibrium when:
(a) mencapai suhu akhir yang sama, dan
reach same final temperature, and
(b) pemindahan bersih haba ialah sifar.
net heat transfer becomes zero.
• Pada keseimbangan terma, haba yang diserap oleh objek yang sejuk adalah sama dengan haba
yang dibebaskan oleh objek yang panas.
At thermal equilibrium, the heat absorbed by the cool object is equal to the heat released by the hot object.
• Pada keseimbangan terma,
Kadar pemindahan tenaga dari A ke B = Kadar pemindahan tenaga dari B ke A
At thermal equilibrium,

Rate of transfer of energy from A to B = Rate of transfer of energy from B to A

© Nilam Publication Sdn. Bhd. 82

B4FizikF4(81-114)csy5p.indd 82 09/01/2020 9:57 AM


MODUL • Fizik TINGKATAN 4

Aplikasi keseimbangan terma dalam kehidupan harian


Application of thermal equilibrium in daily life

Memanaskan objek Menyejukkan objek


Heating object Cooling object
(a) Bebuli termometer bersentuhan secara terma (a) Botol yang berisi susu panas bersentuhan secara
dengan badan pesakit. terma dengan air sejuk.
Bulb of thermometer is in thermal contact with the
patient’s body. Bottle filled with hot milk is in thermal contact with
cold water.
(b) Haba dari badan pesakit mengalir ke termometer
hingga suhu kedua-dua menjadi sama. (b) Haba dari botol berisi susu panas mengalir ke
Heat from the patient’s body flows to the thermometer air sejuk .
until both temperatures become the same. Heat from bottle filled with hot milk flows to the cold
(c) Suhu badan pesakit dapat ditentukan apabila water .
pengaliran bersih haba menjadi sifar.
The temperature of patient’s body can be determined (c) Suhu botol susu panas menurun , suhu air sejuk
when the net heat flow becomes zero. meningkat .
Temperature of bottle with hot milk decreases ,
temperature of cold water increases .

(d) Pengaliran bersih haba menjadi sifar dan


keseimbangan terma dicapai.
Termometer
Thermometer Net heat flow becomes zero and thermal equilibrium is
achieved.

Botol susu
Milk bottle Susu panas
Hot milk
Air sejuk (70 ºC)
Cold water
(20 ºC)

(a) Udara panas di dalam ketuhar bersentuhan secara (a) Makanan yang disimpan di dalam peti sejuk
terma dengan makanan. bersentuhan secara terma dengan udara di dalam
Hot air in the oven is in thermal contact with the food. peti sejuk. / Food that is kept in the refrigerator is in
thermal contact with air inside refrigerator.
(b) Haba dari udara panas mengalir ke makanan. U
U
mengalir N
N
hot air (b) Haba dari makanan ke udara di II
Heat from the flows to the food. T
dalam peti sejuk. / Heat from the food flows to T

(c) Suhu makanan meningkat sehingga ia the air inside the refrigerator. 14
masak.
(c) Suhu makanan menurun dan kekal
The temperature of the food increases until it
is cooked. segar untuk disimpan dalam tempoh yang
lebih lama.
The temperature of food decreases and remains
fresh for a longer period.

83 © Nilam Publication Sdn. Bhd.

B4FizikF4(81-114)csy5p.indd 83 09/01/2020 9:57 AM


MODUL • Fizik TINGKATAN 4

• Alat untuk mengukur suhu ialah termometer . Ciri-ciri fizikal (sifat termometri) termometer yang
berubah dengan haba / suhu
The measuring instrument to measure temperature is thermometer . Physical characteristics (Thermometric properties)
of thermometer that change with heat/
temperature:
• Jenis-jenis termometer / Types of thermometers: (a) Termometer berjenis cecair-dalam-kaca –
panjang / isi padu turus merkuri berubah
(a) Termometer cecair-dalam-kaca dengan haba
Liquid-in-glass thermometer Liquid-in-glass thermometer – the length /
volume of mercury column changes with heat
(i) Termometer merkuri / Mercury thermometer (b) Termometer termogandingan – d.g.e berubah
(ii) Termometer alkohol / Alcohol thermometer terhadap haba Thermocouple thermometer –
the e.m.f/current changes with heat
(b) Termometer termogandingan (c) Termometer rintangan – rintangan berubah
terhadap haba.
Thermocouple thermometer Resistance thermometer – the resistance
(c) Termometer rintangan changes with heat
(d) Termometer gas – tekanan gas berubah
Resistance thermometer terhadap haba.
(d) Termometer gas Gas thermometer – the gas pressure
changes with heat.
Gas thermometer

SP 4.1.2 Menentu ukur sebuah termometer cecair dalam kaca menggunakan dua takat tetap

Aktiviti: Penentukuran Termometer / Activity: Thermometer Calibration

Merkuri digunakan dalam


Termometer Merkuri / Mercury Thermometer: termometer kerana:
Mercury is used in the
Termometer merkuri thermometer because:
Mercury thermometer Ciri-ciri Penerangan
Characteristics Explanation
Legap Senang untuk
Takat ais Opaque mengambil bacaan
Ice point Takat stim Easy to take a
reading
Ais Steam point
Ice Mengembang Skala seragam
seragam Uniform scale
dengan haba
Kaki retort Stim Expands
Retort stand Steam uniformly with
heat
Air dari ais
yang melebur Kasa dawai Daya lekitan Tidak membasah
Water from Wire gauze yang tinggi tiub dan tidak
Xxxxxxxxxxxxxxxxxx High cohesive melekat pada
melting ice force dinding kaca
Penunu Bunsen Does not wet the
tube and does not
U Bunsen burner stick to the glass
N wall
I Takat didih Boleh mengukur
T 100
yang tinggi suhu yang tinggi

4 0 0 °c
High boiling
point
Can measure high
temperature

100 °c Kepekaan termometer boleh


θ ditingkatkan dengan
menggunakan:
The sensitivity of the
thermometer can be increased
Perubahan dalam panjang turus merkuri, Δl = l100 – l0 by using:
The change in length of mercury column, Δl = l100 – l0 (a) tiub kapilari yang kecil
narrow capillary tube
(b) dinding bebuli kaca yang
Perubahan suhu, ΔT1 = 100 ºC – 0 ºC = 100 ºC nipis
The change in temperature, ΔT1 = 100 ºC – 0 ºC = 100 ºC thin glass wall bulb
(c) saiz bebuli yang kecil
small bulb
Perubahan dalam panjang pada suhu θ = lθ – l0
The change in length at temperature θ = lθ – l0
lθ – l0
Oleh itu, suhu / Therefore, the temperature, θ = × 100 ºC
l100 – l0

© Nilam Publication Sdn. Bhd. 84

B4FizikF4(81-114)csy5p.indd 84 09/01/2020 9:57 AM


MODUL • Fizik TINGKATAN 4

Latihan / Exercises
1 Panjang turus merkuri pada takat ais dan pada takat 3 Jarak pada turus merkuri antara 0 ºC dan 100 ºC
stim masing-masing ialah 5.0 cm dan 40.0 cm. adalah 28.0 cm. Apabila termometer itu dimasukkan ke
Apabila termometer itu direndam dalam cecair P, dalam bikar yang berisi air, panjang turus merkuri
panjang turus merkuri adalah 23.0 cm. Berapakah adalah 24.5 cm di atas titik ais. Berapakah suhu air
suhu cecair P? tersebut?
The lengths of the mercury column at ice point and The distance between 0 ºC and 100 ºC on the mercury
steam point are 5.0 cm and 40.0 cm respectively. When column is 28.0 cm. When the thermometer is put into
the thermometer is immersed in liquid P, the length of a beaker of water, the length of the mercury column is
the mercury column is 23.0 cm. What is the temperature 24.5 cm above the ice point. What is the temperature of
of liquid P? the water?
lθ – l0 24.5 cm
SuhuP/TemperatureP = × 100 ºC θ = × 100 ºC
l100 – l0 28.0 cm
(23.0 – 5.0) cm = 87.5 ºC
= × 100 ºC
(40.0 – 5.0) cm
18.0 cm
= × 100 ºC
35.0 cm

= 51.43 °C

4 Jarak pada turus merkuri antara 0 ºC dan 100 ºC


adalah 25 cm. Apabila termometer diletakkan
dalam bikar yang berisi air, panjang turus merkuri
adalah 16 cm di atas titik ais.
2 Panjang turus merkuri pada takat stim dan pada The distance between 0 ºC and 100 °C on the mercury
takat ais masing-masing ialah 65.0 cm dan 5.0 cm. coloumn is 25 cm. When the thermometer is put into a
Apabila termometer itu direndam dalam cecair Q, beaker of water, the length of mercury column is 16 cm
panjang turus merkuri adalah 27.0 cm. Berapakah above the ice point.
suhu cecair Q? (a) Berapakah suhu air tersebut?
The lengths of the mercury column at the steam point What is the temperature of the water?
and ice point are 65.0 cm and 5.0 cm respectively. When (b) Berapakah panjang turus merkuri dari bebuli
the thermometer is immersed in liquid Q, the length of kaca pada suhu 30 ºC?
the mercury column is 27.0 cm. What is the temperature What is the length of the mercury column from the U
U
N
of liquid Q? bulb at 30 ºC? N
II
T
T
(27.0 – 5.0) cm 16 cm
θ =
(65.0 – 5.0) cm
× 100 ºC (a) θ =
25 cm
× 100 ºC 14
22 cm = 64 ºC
= × 100 ºC
60 cm lθ – l0
(b) 30 °C = × 100 ºC
= 36.67 ºC 25 cm
30 °C × 25 cm
lθ – l0 =
100 °C
= 7.5 cm

85 © Nilam Publication Sdn. Bhd.

B4FizikF4(81-114)csy5p.indd 85 09/01/2020 9:57 AM


MODUL • Fizik TINGKATAN 4

4.2 MUATAN HABA TENTU / SPECIFIC HEAT CAPACITY SK 4.2

SP 4.2.1 Menerangkan muatan haba, C

(a) Nyatakan definisi • Ia adalah kuantiti haba yang diperlukan untuk meningkatkan suhu
muatan haba, C. 1 °C
State the definition of sesuatu objek sebanyak .
heat capacity, C. It is the quantity of heat required to increase the temperature of an object
(b) Apakah unit S.I. 1 °C
by .
bagi muatan haba?
What ist the S.I. unit • Unit S.I. bagi muatan haba ialah joules per kelvin (J K–1).
for heat capacity? joules per kelvin
The S.I. unit for heat capacity is (J K–1).

Berikan contoh situasi • Objek yang mempunyai jisim yang lebih besar mempunyai muatan haba
untuk memahami yang lebih tinggi.
muatan haba. Object with a greater mass has a higher heat capacity.
Give example of situation • Contohnya, dua buah cerek yang sama saiz diisi dengan kuantiti air yang
to understand heat berbeza dididihkan pada masa yang sama. Selepas beberapa ketika, cerek
capacity. yang berisi kuantiti air yang sedikit mendidih dahulu berbanding cerek yang
berisi kuantiti air yang lebih banyak.
For example, two equal size of kettles filled with different quantity of water were
boiled at the same time. After several minutes, kettle filled with less quantity of
water boils first compared to kettle filled with more quantity of water.

Q
SP 4.2.2 Mendefinisi muatan haba tentu bahan, c = ———
m(Δθ)

Nyatakan definisi bagi Muatan haba tentu ialah kuantiti haba (Q) yang diperlukan oleh sesuatu bahan
muatan haba tentu, c. berjisim 1 kg untuk meningkatkan suhunya sebanyak 1 ºC.
State the definition of
specific heat capacity, c. The specific heat capacity is the quantity of heat (Q) required by a substance of mass
U 1 1
N kg to increase its temperature by ºC.
I
T
Persamaan bagi Kuantiti haba yang dibekalkan (Q)
4 muatan haba tentu, c.
Muatan haba tentu, c =
Jisim (m) × Perubahan suhu (Δθ)
Equation for specific heat
capacity, c. Quantity of heat supplied (Q)
Specific heat capacity, c =
Mass (m) × Change of temperature (Δθ)
Q

c= ⇒ Q = mcΔθ
mΔθ
Unit c / Unit for c = J kg–1 °C–1 atau / or J kg–1 K–1
Q = kuantiti haba diserap atau dibebaskan (J)
quantity of heat absorbed or released (J)
m = jisim / mass (kg)
Δθ = perubahan suhu / change of temperature (°C atau / or K)

© Nilam Publication Sdn. Bhd. 86

B4FizikF4(81-114)csy5p.indd 86 09/01/2020 9:57 AM


MODUL • Fizik TINGKATAN 4

Jadual di bawah menunjukkan muatan haba tentu bagi beberapa bahan. 1 Muatan haba tentu air ialah
The table below shows the specific heat capatity for certain substances. 4 200 J kg-1 ºC-1.
Specific heat capacity of water is
4 200 J kg-1 ºC-1.
Muatan haba tentu, c 2 Ini bermakna untuk 1 kg air meningkat
Bahan suhu sebanyak 1 ºC, air memerlukan
Specific heat capacity, c 4 200 J haba.
Substance
(J kg–1 °C–1) This means that for 1 kg of water, to
increase its temperature by 1 ºC, the water
needs 4 200 J of heat.
Air / Water 4 200 3 Ini menjadikan air sebagai agen penyejuk
yang baik.
This makes water as a good cooling agent.
Minyak masak / Cooking oil 1 850 Penerangan menggunakan teori kinetik jirim:
Explanation using kinetic theory of matter:
• Apabila bahan menyerap haba, getaran
Aluminium / Aluminium 900 atom / molekul sangat kuat.
When a substance absorbs heat, the
vibrations of atoms / molecules are strong.
Kuprum / Copper 390 • Atom/molekul bergerak dengan laju yang
lebih tinggi.
Atoms / molecules move with a higher
Gas metana / Methane gas 2 200 speed.
• Tenaga kinetik atom / molekul meningkat.
The kinetic energy of the atoms / molecules
Pasir / Sand 800 increases.
• Suhu bahan tersebut meningkat.
The temperature of the substance
Udara / Air 1 000 increases.
• Keadaan fizikal bahan tidak berubah.
The physical state of matter is unchanged.

Mengeksperimen untuk menentukan:


SP 4.2.3
(i)  Muatan haba tentu air (ii)  Muatan haba tentu aluminium

Eksperimen Untuk menentukan muatan haba tentu pepejal dan cecair


Experiment To determine the specific heat capacity of a solid and a liquid

Untuk menentukan nilai muatan haba tentu Untuk menentukan nilai muatan haba tentu
Tujuan air. aluminium. / To determine the specific heat
Aim
To determine the specific heat capacity of water. capacity of aluminium.

Senarai Cawan polistirena, air, pemanas rendam, Blok aluminium, kertas tisu, minyak,
radas dan kertas tisu, termometer, bekalan kuasa, pemanas rendam, termometer, bekalan kuasa, U
U
bahan pengacau, neraca elektronik dan jam randik. neraca elektronik dan jam randik N
N
II
List of Polystyrene cup, water, immersion heater, Aluminium block, tissue paper, oil, immersion T
T
apparatus and tissue paper, thermometer, power supply, stirrer, heater, thermometer, power supply, electronic
materials electronic balance and stopwatch. balance and stopwatch. 14
Pemanas rendam
Immersion heater
Termometer
Termometer Thermometer
Susunan Bekalan kuasa Thermometer
radas Power supply
Jam randik
Bekalan kuasa
Power supply Blok aluminium
Arrangement Stopwatch Aluminium block
Kertas tisu
of the Tissue paper 12
Lubang diisi dengan
minyak untuk
apparatus Air 9 3
meningkatkan konduksi
Water 6 Pemanas rendam Hole filled with
Blok kayu Immersion heater oil to increase
Wooden block thermal conductivity

87 © Nilam Publication Sdn. Bhd.

B4FizikF4(81-114)csy5p.indd 87 09/01/2020 9:57 AM


MODUL • Fizik TINGKATAN 4

1. Cawan diisikan dengan air, berjisim m. 1. Jisim blok aluminium, m, ditentukan


(contoh, m = 0.15 kg). / The cup is filled dengan menggunakan neraca elektronik.
with water with mass, m (example, m = 0.15 kg). The mass of the aluminium block, m is
2. Suhu awal air, θ1, direkodkan. / The initial determined using the electronic balance.
temperature of water, θ1, is recorded. 2. Suhu awal blok aluminium, θ1
3. Pemanas rendam dihidupkan dan pada masa direkodkan. / The initial temperature of the
yang sama jam randik dimulakan. aluminium block, θ1, is recorded.
Prosedur
The immersion heater is switched on and the 3. Pemanas dihidupkan dan pada masa yang
Procedure
stopwatch is started simultaneously. sama jam randik dimulakan.
4. Air dikacau berterusan. The heater is switched on and the stopwatch
The water is stirred continuously. is started simultaneously.
5. Pemanas ditutup selepas 10 minit. 4. Pemanas ditutup selepas 10 minit.
The heater is switched off after 10 minutes. The heater is switched off after 10 minutes.
6. Suhu tertinggi, θ2, direkodkan. 5. Suhu tertinggi, θ2, direkodkan.
The highest temperature, θ2, is recorded. The highest temperature, θ2, is recorded.
Kuasa pemanas rendam, P Kuasa pemanas rendam, P
Power of the immersion heater, P Power of the immersion heater, P
= W = W
Jisim air, m Jisim blok aluminium, m
Mass of water, m Mass of aluminium block, m
Penjadualan
data = kg = kg
Tabulate the Suhu awal air, θ1 Suhu awal aluminium, θ1
data Initial temperature of water, θ1 Initial temperature of aluminium, θ1
°
= C = °
C
Suhu akhir air, θ2 Suhu akhir aluminium, θ2
Final temperature of water, θ2 Final temperature of aluminium, θ2
°
= C = °
C
Pengiraan muatan haba tentu air, c: Pengiraan muatan haba tentu aluminium, c:
Calculation of specific heat capacity of water, c: Calculation of specific heat capacity of aluminium, c:
Analisis data Q = Pt = mcΔθ Q = Pt = mcΔθ
Analysis of the Pt = mc(θ2 – θ1) Pt = mc(θ2 – θ1)
data
Pt Pt
∴ c = ∴ c =
m(θ2 – θ1) m(θ2 – θ1)

U SP 4.24 Berkomunikasi untuk menerangkan aplikasi muatan haba tentu dalam kehidupan harian, kejuruteraan bahan dan fenomena alam
N
I
T
1 Aplikasi muatan haba tentu dalam kehidupan harian. / Application of specific heat capacity in daily life.
(a) Periuk tanah liat / Clay pot
4
• Tanah liat mempunyai muatan haba tentu lebih tinggi berbanding logam.
Clay has a higherspecific heat capacity than metal.
• Semasa memasak, haba dialirkan perlahan dari api kepada makanan di dalam
periuk. / During cooking, heat is flow slowly from the fire to the food inside the pot.
• Masa memasak makanan yang diperlukan lebih lama .
A longer cooking time is needed to cook the food.
• Selepas api dipadam, periuk tanah liat mempunyai suhu yang lebih tinggi berbanding makanan di
dalamnya. / After the flame is switched off, the clay pot has a higher temperature than the food inside it.
• Sejumlah besar haba masih terus dialirkan kepada makanan itu.
A considerable amount of heat continues to be transferred to the food.
• Selepas periuk tanah liat itu dialihkan daripada api, makanan di dalam periuk kekal panas dalam tempoh
masa yang lama.
The food inside the clay pot stay hot for a long time after the clay pot has been removed from the source of fire.

© Nilam Publication Sdn. Bhd. 88

B4FizikF4(81-114)csy5p.indd 88 09/01/2020 9:57 AM


MODUL • Fizik TINGKATAN 4

Air sebagai penyejuk dalam enjin kereta (sistem radiator kereta)


(b)
Water as coolant in a car engine (car radiator system)

Air dipam melalui


pam air
Water is circulated by Haba yang dihasilkan dari pembakaran
the water pump dalam silinder diserap oleh air
Heat generated from the combustion in the
cylinders is absorbed by the water
Udara masuk melalui
kipas radiator
Air drawn in by the
radiator fan Kipas
radiator
Haba hilang dari sirip Radiator fan
penyejuk ke persekitaran
Heat is lost from the cooling Aliran air memasuki enjin
fins to the surroundings Water enters the engine

• Air mempunyai muatan haba tentu yang tinggi .


Water has a high specific heat capacity.
• Air yang mengalir ke dalam blok enjin akan menyerap haba yang terhasil daripada pembakaran bahan
api di dalam enjin.
Water flows inside the engine block will absorb heat produced from the burning of fuel inside the engine.
• Suhu air yang meningkat akan mengalir ke kipas radiator. Udara sejuk akan disedut oleh kipas radiator
bagi menyerap haba dari air panas melalui sirip penyejuk.
The temperature of water rises and flows to the fan radiator. Cold air drawn in by the fan radiator will absorb
the heat from hot water through cooling fins.

2 Aplikasi muatan haba tentu dalam kejuruteraan bahan.


Application of specific heat capacity in material engineering.
Peralatan memasak / Cooking utensils

Bekas / Body Pemegang / Handle


• Diperbuat daripada logam. • Pemegang diperbuat daripada
Made of metal. bahan sintetik . / Handle is U
U
• Muatan haba tentu rendah , synthetic material
N
N
II
maka ia cepat menjadi panas. made of . T
T
Low specific heat capacity, so it • Muatan haba tentu tinggi , 14
becomes hot quickly. ia tidak akan menjadi sangat
• Ketumpatan rendah, maka ianya panas apabila haba mengalir.
sangat ringan. High specific heat capacity.
Low density, so it is very light. It will not become too hot when
• Makanan di dalam kuali dipanaskan heat is flow.
pada suhu yang tinggi dalam • Konduktor haba lemah, sangat
tempoh masa yang singkat. sedikit haba dipindahkan
Food in the wok can be heated at high kepada tangan seseorang yang
temperature in a short time. menyentuh pemegang periuk
itu.
Poor conductor of heat, very little
heat from the pot is transferred to
the hand of the person holding the
handle of the pot.

89 © Nilam Publication Sdn. Bhd.

B4FizikF4(81-114)csy5p.indd 89 09/01/2020 9:57 AM


MODUL • Fizik TINGKATAN 4

3 Aplikasi muatan haba tentu dalam fenomena alam


Application of specific heat capacity in natural phenomena
(a) Bayu laut / Sea breeze

• Suhu udara di permukaan laut adalah rendah. • Suhu udara di permukaan darat adalah
The temperature of air on sea surface is lower. lebih tinggi.
• Ketumpatan udara di permukaan laut adalah Temperature of air on land is higher.
• Ketumpatan udara di permukaan darat
lebih tumpat.
adalah kurang tumpat.
Density of air on sea surface is more dense. Density of air on land surface is less dense.
• Kawasan bertekanan lebih tinggi. • Kawasan bertekanan lebih rendah.
Area with higher pressure. Area with lower pressure.

• Muatan haba tentu Udara panas


• Muatan haba tentu
laut adalah lebih Hot air darat (pepejal)
tinggi . adalah lebih
Specific heat capacity
Udara Bayu laut rendah .
sejuk Sea
of sea is higher . Cold air Specific heat capacity
breeze
• Suhu laut menjadi of land (solid) is
Darat / Land lower .
lebih rendah . Laut / Sea
Temperature of sea • Suhu darat menjadi
lebih tinggi .
becomes lower .
Temperature of
Arah pergerakan udara. land becomes
Direction of air movement.
higher .

Bayu darat / Land breeze


(b)

• Suhu udara di permukaan laut adalah lebih • Suhu udara di permukaan darat adalah lebih
tinggi. rendah.
Temperature of air on sea surface is higher. Temperature of air on land is lower.
• Ketumpatan udara di permukaan laut adalah • Ketumpatan udara di permukaan darat adalah
U kurang tumpat. lebih tumpat.
N
I Density of air on sea surface is less dense. Density of air on land surface is more
T
• Kawasan bertekanan lebih rendah. dense.
4 Area with lower pressure. • Kawasan bertekanan lebih tinggi.
Area with higher pressure.

• Muatan haba tentu Udara sejuk • Muatan haba tentu


laut adalah lebih Cold air darat adalah lebih
tinggi . Udara rendah .
panas Bayu darat
Specific heat capacity Hot air Land breeze Specific heat capacity
of sea is higher . of land is lower .
• Suhu laut menjadi • Suhu darat menjadi
lebih tinggi . lebih rendah
Temperature of sea Temperature of land
becomes higher . Arah pergerakan udara. becomes lower .
Direction of air movement.

© Nilam Publication Sdn. Bhd. 90

B4FizikF4(81-114)csy5p.indd 90 09/01/2020 9:57 AM


MODUL • Fizik TINGKATAN 4

SP 4.25 Menyelesaikan masalah yang melibatkan muatan haba tentu menggunakan rumus, Q = mc∆θ

Latihan / Exercises
1 Sebuah blok logam berjisim 2 kg. Hitungkan 4 Sebuah 2 kW, 240 V pemanas elektrik digunakan
jumlah haba yang mesti dipindahkan kepada logam untuk memanaskan 3 kg air. Jika kuasa dibekalkan
untuk meningkatkan suhu dari 30 ºC kepada 70 ºC. selama 8 minit, berapakah peningkatan suhu air
(Muatan haba tentu logam = 500 J kg–1 ºC–1) tersebut?
A metal block has a mass of 2 kg. Calculate the amount (Muatan haba tentu air adalah 4 200 J kg–1 ºC–1)
of heat that must be transferred to the metal to raise its
A 2 kW, 240 V electric heater is used to heat up 3 kg
temperature from 30 ºC to 70 ºC.
(Specific heat capacity of the metal = 500 J kg–1 ºC–1) of water. If the power is supplied for 8 minutes, what
is the increase in temperature of the water?
Penyelesaian / Solution: (Specific heat capacity of water is 4 200 J kg–1 ºC–1)
Q = mcΔθ
= (2 kg) × (500 J kg–1 °C–1) × (70 – 30) °C Penyelesaian / Solution:
= (2 × 500 × 40) J Pt = mcΔθ
= 40 000 J (2 000 W)(8 × 60 s) = 3 kg × 4 200 J kg–1°C–1 × Δθ
(2 × 103 × 8 × 60) J
Δθ =
3 kg × 4 200 J °C–1
= 76.2 ºC
2 8.4 × 105 J tenaga haba meningkatkan suhu 4 kg
air dari 40 ºC hingga 90 ºC. Berapakah muatan
haba tentu air tersebut?
8.4 × 105 J of heat energy raises the temperature of
4 kg of water from 40 ºC to 90 ºC. What is the specific
heat capacity of the water?
Penyelesaian / Solution:
Q 5 Sebuah pemanas rendam 1.2 kW digunakan untuk
c =
mΔθ meningkatkan 0.2 kg air dalam bekas kuprum
(8.4 × 105) J berjisim 0.05 kg. Kirakan masa yang diambil agar
=
4 kg × (90 – 40) °C suhu air dan bekas dinaikkan sebanyak 20 °C.
(Muatan haba tentu air, ca = 4 200 J kg–1 °C–1).
= 4 200 J kg–1 ºC–1
(Muatan haba tentu kuprum, ck = 400 J kg–1 °C–1)
A 1.2 kW immersion heater is used to raise the
temperature of 0.2 kg water in a copper container
of mass 0.05 kg. Calculate the time taken so that the
3 0.2 kg air pada suhu 100 ºC dicampur dengan temperature of the water and the container is increased U
U
0.25 kg air pada suhu 10 ºC. Berapakah suhu by 20 °C. N
N
tertinggi yang dicapai oleh campuran tersebut? (Specific heat capacity of water, ca = 4 200 J kg–1 °C–1) II
T
T
0.2 kg of water at 100 ºC is mixed with 0.25 kg of water (Specific heat capacity of copper, ck = 400 J kg–1 °C–1)
at 10 ºC. What is the maximum temperature reached by
Penyelesaian / Solution:
14
the mixture?
Pt = mcΔθ
Penyelesaian / Solution:
(1.2 × 103 W) × t = macaΔθ + mkckΔθ
Haba yang dibebaskan = Haba yang diserap
(1 200 W) × t = Δθ (maca + mkck)
Heat released = Heat absorbed
= 20°C [(0.2 kg × 4 200 J kg–1 °C–1)
(0.2 kg)(ca)(100 – θ)°C = (0.25 kg)(ca)(θ – 10)°C
+ (0.05 kg × 400 J kg °C–1)]
di mana ca = muatan haba tentu air
17 200 J
where ca = specific heat capacity of water t =
20 – 0.2θ = 0.25θ – 2.5 1 200 J s–1
20 + 2.5 = 0.25θ + 0.2θ t = 14.33 s
22.5 = 0.45θ
22.5
θ = = 50 ºC
0.45

91 © Nilam Publication Sdn. Bhd.

B4FizikF4(81-114)csy5p.indd 91 09/01/2020 9:57 AM


MODUL • Fizik TINGKATAN 4

4.3 HABA PENDAM TENTU / SPECIFIC LATENT HEAT SK 4.3

SP 4.3.1 Menerangkan haba pendam

Nyatakan definisi haba Jumlah haba yang diserap atau yang dibebaskan pada suhu
pendam. tetap semasa perubahan keadaan jirim berlaku pada sesuatu bahan.
State the definition of latent The heat absorbed or released at a constant temperature
heat. during the changing state of substance occur.
Terangkan konsep bagi (a) Jirim wujud dalam 3 keadaan, iaitu pepejal , cecair
haba pendam tentu dengan dan gas .
perubahan keadaan jirim. solid liquid gases
Matter exists in 3 states, , and .
Explain the concept of latent
heat with the change of state (b) Perubahan fasa merujuk kepada perubahan keadaan jirim
of matter. suatu bahan.
Penerangan menggunakan Teori Phase change refers to the change in state of matter of a substance.
Kinetik jirim: / Explanation using
Kinetic Theory of matter: (c) Perubahan fasa melibatkan penyerapan atau pembebasan
• Apabila bahan menyerap haba,
getaran atom/molekul adalah kuat.
haba.
When a substance absorbs heat,
The phase change involves the absorption or release of heat.
the vibrations of atoms/molecules
are strong.
• Ikatan antara atom/molekul menjadi
(d) Haba yang terlibat semasa perubahan fasa dikenali sebagai
lemah. / The bonds between haba pendam dan digunakan terutamanya untuk melemahkan
atoms/molecules become weak.
• Atom/molekul dengan tenaga yang
dan mengatasi daya tarikan antara molekul. / Heat involved during
tinggi boleh memecahkan ikatan
dan mengubah kepada keadaan
the phase change is known as latent heat and used to weaken
fizikal yang baru.
Atoms/molecules with very high
and to overcome the intermolecular attraction force.
energy can break the bonds and
change to a new physical state.
(e) Tenaga kinetik molekul tidak berubah .
• Keadaan fizikal bahan berubah.
The physical state of matter
Kinetic energy of molecules is unchanged .
changes.
• Tenaga kinetik atom/molekul tidak
(f) Suhu semasa perubahan fasa adalah malar .
berubah.
The kinetic energy of the atoms/
Temperature during phase change is constant .
molecules is unchanged.
• Suhu tetap/tidak berubah.
The temperature is constant.

U • Nama proses / Name of process: Pendidihan / Boiling


N
I • Haba pendam diserap / Latent of heat is absorbed
T

4
• Nama proses / Name of process: Gas
Gas
Peleburan / Melting
• Haba pendam diserap
• Nama proses / Name of process:
Latent heat is absorbed
Kondensasi / Condensation
• Haba pendam dibebaskan
Latent heat is released
Pepejal / Solid

• Nama proses / Name of process:


Cecair / Liquid
Pembekuan / Freezing
• Haba pendam dibebaskan
Latent heat is absorbed

© Nilam Publication Sdn. Bhd. 92

B4FizikF4(81-114)csy5p.indd 92 09/01/2020 9:57 AM


MODUL • Fizik TINGKATAN 4

Mendefinisi
Q
(i) haba pendam tentu, l = —
SP 4.3.2 m
(ii) haba pendam tentu pelakuran, lf
(iii) haba pendam tentu pengewapan, lv

Nyatakan definisi bagi Kuantiti haba, Q yang diserap atau dibebaskan semasa perubahan fasa bagi
haba pendam tentu, l. 1 kg bahan tanpa perubahan suhu.
State the definition of specific The quantity of heat, Q that is absorbed or released during the change of phase of
latent heat, l. 1 kg of substance without the change of temperature.

Nyatakan persamaan Persamaan / Equation:


dan unit S.I. bagi haba Haba (Q) Heat (Q)
Haba pendam tentu, l = / Specific latent heat, l =
pendam tentu, l. Jisim (m) Mass (m)
State the equation and S.I. Q
unit for specific latent heat, l. l = ⇒ Q = ml
m

Unit S.I. bagi l ialah J kg–1


S.I. unit of l is J kg–1

Berikan dua jenis haba


(i) Haba pendam tentu pelakuran (lf)
pendam tentu.
Give two types of specific Specific latent heat of fusion (lf )
latent heat.
(ii) Haba pendam tentu pengewapan (lv)
Specific latent heat of vaporisation (lv)

Nyatakan definisi bagi • Kuantiti haba (Q) yang diperlukan untuk mengubah bahan berjisim 1 kg
haba pendam tentu pada suhu yang tetap.
pelakuran, lf . Quantity of heat (Q) required to change 1 kg of substance at a constant temperature.
State the definition of specific • Proses yang terlibat adalah peleburan (haba diserap) dan pembekuan (haba
latent heat of fusion, lf . dibebaskan).
The processes involved are melting (heat is absorbed) and freezing (heat is
Pelakuran – proses di mana
pepejal berubah kepada cecair released).
apabila haba diserap.
Fusion – A process where solid
changes to liquid when heat is
absorbed.

U
U
Ais / Ice Air / Water N
N
II
T
T
Nyatakan definisi bagi • Kuantiti haba (Q) yang diperlukan untuk mengubah bahan berjisim 1 kg
haba pendam tentu pada suhu yang tetap. 14
pengewapan, lv. Quantity of heat (Q) required to change 1 kg of substance at a constant temperature.
State the definition of specific • Proses yang terlibat adalah pendidihan (haba diserap) dan kondensasi (haba
latent heat of vaporisation, lv. dibebaskan).
The processes involved are boiling (heat is absorbed) and condensation (heat is
Pengewapan – proses di mana
cecair berubah kepada wap released).
apabila haba diserap.
Vaporisation – A process where
liquid changes to gas when heat is
absorbed.

Air Stim / Wap


Water Steam

93 © Nilam Publication Sdn. Bhd.

B4FizikF4(81-114)csy5p.indd 93 09/01/2020 9:57 AM


MODUL • Fizik TINGKATAN 4

Lengkung Pemanasan / Heating Curve


Kaji lengkung pemanasan bagi suatu bahan di bawah. / Study the heating curve of a substance below.
Suhu / °C
Temperature / °C
F

D E

B C

Masa / s
Time / s
(a) Nyatakan keadaan fizikal bahan pada kawasan berikut:
State the physical state of the substance at the following region:

AB Pepejal / Solid

BC Pepejal dan cecair / Solid and liquid

CD Cecair / Liquid

DE Cecair dan gas / Liquid and gas

EF Gas / Gas

(b) Berdasarkan lengkung pemanasan tersebut, terangkan perubahan keadaan fizikal dan suhu bahan.
Based on the following heating curve, explain the changes in physical state and temperature of the substance.

AB / CD / EF
1 Haba diserap . / Heat is absorbed .
U
N 2 Suhu semakin meningkat . / The temperature increases .
I
T 3 Tenaga kinetik molekul bertambah . / Kinetic energy of molecule increases .

4
BC
1 Haba pendam diserap untuk melemahkan ikatan antara molekul.
Latent heat is absorbed to weaken the bond between molecules.
2 Molekul dibebaskan daripada kedudukan tetap dan bergerak antara satu sama lain.
Molecules are released from their fixed position and move throughout the liquid.
3 Tenaga kinetik molekul dan suhu tidak berubah.
Kinetic energy of molecules and temperature remain unchanged.

DE
1 Haba pendam diserap untuk memutuskan ikatan antara molekul.
Latent heat is absorbed to break the bond between molecules.
2 Molekul dipisahkan jauh antara satu sama lain.
Molecules are separated far from each other.
3 Tenaga kinetik molekul dan suhu tidak berubah.
Kinetic energy of molecules and temperature remain unchanged.

© Nilam Publication Sdn. Bhd. 94

B4FizikF4(81-114)csy5p.indd 94 09/01/2020 9:57 AM


MODUL • Fizik TINGKATAN 4

Lengkung Penyejukan / Cooling Curve


Kaji lengkung penyejukan bagi suatu bahan di bawah. / Study the cooling curve of a substance below.
Suhu / °C
Temperature / °C

P
O

Q R

S
Masa / s
Time / s
(a) Nyatakan keadaan fizikal bahan pada kawasan berikut:
State the physical state of the substance at the following region:

NO Gas / Gas

OP Gas dan cecair / Gas and liquid

PQ Cecair / Liquid

QR Cecair dan pepejal / Liquid and solid

RS Pepejal / Solid

(b) Berdasarkan lengkung penyejukan tersebut, terangkan perubahan keadaan fizikal dan suhu bahan.
Based on the following cooling curve, explain the changes in physical state and temperature of the substance.

NO / PQ / RS
1 Haba dibebaskan . / Heat is released .
decreases U
2 Suhu semakin menurun . / The temperature . U
N
N
II
3 Tenaga kinetik molekul berkurang. / Kinetic energy of molecules decreases. T
T
14
OP
1 Haba pendam dibebaskan bagi molekul membentuk semula ikatan.
Latent heat is released to form the molecular bond.
2 Molekul bergerak antara satu sama lain.
Molecules move with each other.
3 Tenaga kinetik molekul dan suhu tidak berubah.
Kinetic energy of molecules and temperature remain unchanged.

QR
1 Haba pendam dibebaskan untuk menguatkan ikatan antara molekul.
Latent heat is released to strengthen the bond between molecules.
2 Molekul bergetar sekitar kedudukan yang tetap.
Molecules vibrate around the fixed position.
3 Tenaga kinetik molekul dan suhu tidak berubah.
Kinetic energy of molecules and temperature remain unchanged.

95 © Nilam Publication Sdn. Bhd.

B4FizikF4(81-114)csy5p.indd 95 09/01/2020 9:57 AM


MODUL • Fizik TINGKATAN 4

Mengeksperimen untuk menentukan,


SP 4.3.3
(i) haba pendam tentu pelakuran ais, lf (ii) haba pendam tentu pengewapan air, lv

Untuk menentukan haba pendam tentu pelakuran ais, lf dan haba pendam
Eksperimen tentu pengewapan air, lv / To determine the specific latent heat of fusion of ice,
Experiment
lf and specific latent heat of vaporisation of water, lv

Untuk menentukan haba pendam tentu Untuk menentukan haba pendam tentu
Tujuan
pelakuran ais, lf / To determine the specific pengewapan air, lv / To determine the specific
Aim
latent heat of fusion of ice, lf latent heat of vaporisation of water, lv

Ais tulen, pemanas rendam elektrik, corong Air tulen, kertas tisu, bikar, pemanas rendam
Radas dan penuras, bikar, jam randik, neraca elektronik, elektrik, nearaca elektronik, jam randik,
bahan bekalan kuasa, kaki retort, dan pengapit bekalan kuasa, kaki retort, dan pengapit
Apparatus and Pure ice, electric immersion heater, filter funnel, Pure water, tissue paper, beaker, electric
materials beaker, stopwatch, electronic balance, power immersion heater, electronic balance, stopwatch,
supply, retort stand, and clamp power supply, retort stand, and clamp

Ke bekalan
Pemanas rendam kuasa
Immersion heater To power Ke bekalan kuasa
supply To power supply
Susunan
Ais / Ice
radas
Arrangement Pemanas rendam
Bikar Immersion heater
of the Beaker Air Bikar / Beaker
apparatus Air / Water Water
Kertas tisu
Tissue paper
Set A Neraca elektronik Set B Neraca elektronik
Electronic balance Electronic balance

1. Radas disusun seperti yang ditunjukkan. 1. Radas disusun seperti yang ditunjukkan.
2. Jisim bagi setiap bikar kosong, A dan B 2. Satu bikar diletakkan di atas neraca
ditimbang menggunakan neraca. elektronik.

U
3. Setiap satu daripada dua corong penapis 3. Pemanas rendam elektronik direndam
N
I dipenuhi dengan kiub ais. sepenuhnya di dalam air dan diapit
T
4. Pemanas rendam dalam Set A ialah menggunakan kaki retort.
4
Prosedur eksperimen kawalan (tidak disambungkan 4. Pemanas elektrik dihidupkan untuk
Procedure kepada bekalan kuasa). Ini adalah untuk memanaskan air tersebut pada takat didih.
menentukan jisim ais dicairkan oleh haba 5. Apabila air mula mendidih pada kadar
sekitar. Pemanas di Set B dihidupkan. yang tetap, jam randik dimulakan dan
5. Apabila air mula menitis dari corong bacaan pada neraca elektronik, m1
penapis pada kadar yang tetap, jam randik direkodkan.
dimulakan dan bikar kosong, A dan B 6. Air dibiarkan mendidih selama tempoh
diletakkan di bawah corong penuras. t saat.

© Nilam Publication Sdn. Bhd. 96

B4FizikF4(81-114)csy5p.indd 96 09/01/2020 9:57 AM


MODUL • Fizik TINGKATAN 4

6. Selepas tempoh t saat, pemanas B 7. Pada akhir tempoh t saat, bacaan pada
dimatikan. Jisim kedua-dua bikar berisi neraca elektronik, m2, direkodkan.
air, A dan B ditentukan menggunakan 8. Kuasa pemanas rendam, P, direkodkan.
neraca elektronik. Kuasa pemanas, P 1. The apparatus was set up as shown.
direkodkan. 2. A beaker is placed on a platform of an
1. The apparatus was set up as shown. electronic balance.
2. The mass of each of the two empty beakers, A 3. An electronic immersion heater is fully
and B is determined using the electronic immersed in the water and held in this
balance. position by a retort stand.
3. Each of the two filter funnels is filled with ice 4. The electric heater is switched on to heat the
cubes. water to its boiling point.
4. The immersion heater in Set A is the control 5. When the water starts to boil at a steady rate,
experiment (not connected to the power the stopwatch is started and the reading on
supply). This is to determine the mass of ice the electronic balance, m1 is recorded.
melted by the surrounding heat. The heater in 6. The water is allowed to boil for a period of
Set B is switched on. t seconds.
5. When water starts to drip from the filter 7. At the end of the period of t seconds, the
funnels at a steady rate, the stopwatch is reading on the electronic balance, m2, is
started and the empty beakers, A and B are recorded.
placed on beneath the filter funnels. 8. The power of the immersion heater, P, is
6. After a period of t seconds, the heater B recorded.
is switched off. The masses of both beakers
of water, A and B are determined using the
electronic balance. Power of heater, P is
recorded.

Jisim air bagi set A, m1 (kg) Bacaan awal neraca elektronik, m1 (kg)
U
U
Mass of water in set A, m1 (kg) Initial reading of electronic balance, m1 (kg) N
N
Jisim air bagi set B, m2 (kg) Bacaan akhir neraca elektronik, m2 (kg) II
T
T
Keputusan Mass of water in set B, m2 (kg) Final reading of electronic balance, m2 (kg)
Results Kuasa pemanasan, P (W) Masa, t (s)
14
Power of heater, P (W) Time, t (s)
Masa pemanasan, t (s) Kuasa pemanasan, P (W)
Time of heating, t (s) Power of heating, P (W)

Pengiraan bagi haba pendam tentu Andaikan tiada kehilangan haba ke


perlakuran ais, lf. / Calculation of specific persekitaran dan semua haba yang
latent heat of fusion of ice, lf. dibekalkan oleh pemanas diserap oleh air.
E = Pt Assuming that the heat loss to the surroundings
Analisis data Q = mlf is negligible and all heat supplied by the heater
Analysis of the Andaikan tiada kehilangan haba ke is absorbed by water.
data persekitaran. Q = E
Assume no heat loss to the surroundings. mlv = Pt
Pt = mlf
Pt
Pt lv = m – m
lf = m – m 1 2
2 1

97 © Nilam Publication Sdn. Bhd.

B4FizikF4(81-114)csy5p.indd 97 09/01/2020 9:57 AM


MODUL • Fizik TINGKATAN 4

SP 4.3.4 Berkomunikasi untuk menerangkan aplikasi haba pendam tentu dalam kehidupan harian

Aplikasi haba pendam tentu dalam kehidupan harian


Application of specific latent heat in daily life

1 Penyejukan dalam peti ais / Refrigeration

1 Pemampat memampatkan agen penyejuk (gas) Injap pengembang


Penyejat / Evaporator
untuk menambah tekanan dan suhu. Expansion valve
Compressor compresses the cooling agent (gas) to
3
increase the pressure and temperature. 4
2 Apabila sampai pada kondenser, agen
penyejuk (gas) akan membebaskan haba Haba
diserap
pendam ketika proses kondensasi. Kondenser 5 Heat is
When reach the condenser, the cooling agent Condenser absorbed
2
(gas) will release latent heat during condensation
process. Haba
dibebaskan
3 Agen penyejuk (cecair) mengalir ke injap Heat is
pengembang. released
Pemampat
The cooling agent (liquid) flows to the expansion Compressor
valve.
4 Dalam penyejat, agen penyejuk (cecair) akan 1
menyerap haba pendam daripada udara di
dalam peti sejuk.
In the evaporator, the cooling agent (liquid) will absorb latent heat from air inside the refrigerator.
5 Agen penyejuk (gas) mengalir keluar ke pemampat.
The cooling agent (gas) flows out to the compressor.

2 Penyejatan peluh / Evaporation of sweat

• Air peluh tersejat . / Sweat evaporates .


• Air peluh menyerap haba pendam pengewapan daripada permukaan Tersejat / Evaporates
kulit badan. Peluh / Sweat
Sweat absorbs the latent heat of vaporisation from the skin surface. Kulit / Skin
• Permukaan kulit kehilangan haba menyebabkan suhu permukaan
kulit menurun .
U Kelenjar peluh
N Sweat gland
I Skin surface loses heat cause the temperature of skin surface
T
drops
4 .

3 Mengukus makanan menggunakan pengukus / Steam food using a steamer

• Air mempunyai haba pendam tentu pengewapan Haba pendam Penutup


tinggi . pengewapan Lid
yang dibebaskan
Latent heat of Stim / Steam
Water has a high specific latent heat of vaporisation
vaporisation. released Air mendidih
• Stim terkondensasi di permukaan makanan. Kuali Boiled water
Steam condenses on the surface of food. Wok

• Haba pendam pengewapan yang tinggi dibebaskan dan diserap oleh makanan.
High latent heat of vaporisation released and absorbed by the food.
• Makanan akan masak secara menyeluruh dan sempurna.
Food will be cooked thoroughly and perfectly.

© Nilam Publication Sdn. Bhd. 98

B4FizikF4(81-114)csy5p.indd 98 09/01/2020 9:57 AM


MODUL • Fizik TINGKATAN 4

SP 4.3.5 Menyelesaikan masalah yang melibatkan haba pendam

Latihan / Exercises
1 Berapakah tenaga harus dialihkan dari 4.0 kg air Penyelesaian / Solution:
pada 20 ºC untuk menghasilkan ais pada 0 ºC? Ais / Ice miciΔθi Ais / Ice mlf Air / Water
How much energy has to be removed from 4.0 kg of –15 °C 0 °C 0 °C
water at 20 ºC to produce a block of ice at 0 ºC?
[Muatan haba tentu air = 4.2 × 103 J kg–1 ºC–1, haba mwcwΔθw Air / Water mlv Stim / Steam
pendam tentu pelakuran ais = 3.34 × 105 J kg–1] 100 °C 100 °C
[Specific heat capacity of water = 4.2 × 103 J kg–1 ºC–1,
specific latent heat of fusion of ice = 3.34 × 105 J kg–1] Jumlah tenaga haba
= miciΔθi + mlf + mwcwΔθw + mlv
Total heat energy
Penyelesaian / Solution:
miciΔθi = (4 kg) × (2.1 × 103 J kg–1 °C–1) × (15 °C)
Air / Water mcΔθ Air / Water mlf Ais / Ice = 1.26 × 105 J
20 °C 0 °C 0 °C mlf = (4 kg) × (3.34 × 105 J kg–1)
= 1.336 × 106 J
Jumlah tenaga haba / Total heat energy mwcwΔθw = (4 kg) × (4.2 × 103 J kg–1 °C–1) × (100 °C)
= mcΔθ + mlf = 1.68 × 106 J
= [(4.0 kg) × (4.2 × 103 J kg–1 °C–1) × (20 – 0)°C] mlv = (4 kg) × (2.26 × 106 J kg–1)
+ [(4 kg) × (3.34 × 105 J kg–1)] = 9.04 × 106 J
= (3.36 × 105 J) + (13.36 × 105 J) ∴ Jumlah tenaga haba / Total heat energy
= 16.72 × 105 J = (1.26 × 105 J) + (1.336 × 106 J) + (1.68 × 106 J)
= 1.672 × 106 J + (9.04 × 106 J)
= 1.218 × 107 J
2 Sebuah pemanas elektrik 800 W digunakan untuk
memanaskan air. Berapakah masa yang diperlukan 4 Berapakah kuantiti haba yang diperlukan untuk
untuk mengurangkan air sebanyak 4 kg selepas air mencairkan 2.0 kg ais pada 0 ºC? (Haba pendam
mencapai takat didih? tentu pelakuran ais = 3.34 × 105 J kg–1)
[Haba pendam tentu pengewapan air What is the quantity of heat required to melt 2.0 kg
= 2.26 × 106 J kg–1] ice at 0 ºC? (The specific latent heat of fusion of ice
A 800 W electric heater is used to boil water. What is = 3.34 × 105 J kg–1)
the time required to reduce the mass of water by 4 kg Penyelesaian / Solution:
after the water has reached its boiling point? Q = mlf
[Specific latent heat of vaporisation of water = (2.0 kg) × (3.34 × 105 J kg–1)
= 2.26 × 106 J kg–1] = 6.68 × 105 J
Penyelesaian / Solution:
Tenaga elektrik yang Tenaga haba yang 5 Sebuah blok pepejal 0.5 kg dipanaskan oleh pemanas
dibekalkan
= diterima elektrik 100 W. Graf menunjukkan bagaimana suhu
Electric energy provided Heat absorbed berubah dengan masa. U
U
Pt = mlv 0.5 kg of a solid block is heated by a 100 W heater. The N
N
II
(800 W) t = 4 kg × 2.26 × 106 J kg–1 graph shows how the temperature varies with time. T
T

t =
(4 × 2.26 × 106) J
100
14
800 J s–1
Suhu / Temperature (°C)

= 1.13 × 104 s 80
60
3 Kirakan haba yang diperlukan untuk menukar 4 kg
40
ais pada –15 ºC kepada stim pada 100 ºC.
20
Calculate the heat required to convert 4 kg of ice at
–15 ºC to steam at 100 ºC. 0 Masa / Time (s)
0 100 200 300 400 500 600 700 800 900 10001100
[Muatan haba tentu ais / Specific heat capacity of ice
= 2.1 × 103 J kg–1 ºC–1, Hitungkan haba pendam tentu pelakuran pepejal
Muatan haba tentu air / Specific heat capacity of water itu. / Calculate the specific latent heat of fusion of the solid.
= 4.2 × 103 J kg–1 ºC–1, Penyelesaian / Solution:
Haba pendam tentu pelakuran ais Pt = mlf
Specific latent heat of fusion of ice ( 100 W) × (1 050 – 300) s = 0.5 kg × lf
= 3.34 × 105 J kg–1 dan / and 75 000 J
Haba pendam tentu pengewapan air lf =
Specific latent heat of vaporisation of water 0.5 kg
= 2.26 × 106 J kg–1] = 1.5 × 105 J kg–1

99 © Nilam Publication Sdn. Bhd.

B4FizikF4(81-114)csy5p.indd 99 09/01/2020 9:57 AM


MODUL • Fizik TINGKATAN 4

4.4 HUKUM GAS / GAS LAWS SK 4.4

SP 4.4.1 Menerangkan tekanan, suhu dan isi padu gas dari segi kelakuan molekul gas berdasarkan Teori Kinetik Gas

1 Terangkan kuantiti yang terlibat dalam kelakuan molekul gas berdasarkan Teori Kinetik Gas.
Explain the quantities involved in the gas molecule behaviour based on the Kinetic Theory of Gas.

Bekas tertutup Molekul gas / Gas molecule


Closed container

Ciri gas Penerangan


Characteristic of gas Explanation
Kadar perlanggaran antara molekul-molekul gas dengan dinding bekas. Apabila
kadar perlanggaran antara molekul gas dengan dinding bekas bertambah, tekanan
Tekanan (P) gas juga bertambah.
Pressure (P) Rate of collision between the gas molecules and the walls of the container. When the rate of
collisions between gas molecules and the walls of the container increases, the gas pressure
also increases.
Apabila molekul-molekul gas bergerak dengan halaju tinggi, tenaga kinetik
Suhu (T)
bertambah, maka suhu gas juga bertambah. / When the gas molecules moves at a
Temperature (T)
higher speed, the kinetic energy increases, so the gas temperature also increases.
Isi padu (V) Isi padu gas adalah sama dengan isi padu bekas.
Volume (V) Volume of gas is equal to the volume of container.

2 Berikan tiga jenis hukum gas yang ada dan definisinya.


Give three types of gas laws and their definitions.

(a) Hukum Boyle


Boyle’s Law

U
N
I
T

4 (b) Hukum Charles 3 Hukum Gas (c) Hukum Tekanan


Charles’ Law 3 Gas Laws Pressure Law

(a) Hukum Boyle: tekanan gas berkadar songsang dengan isi padu gas pada suhu yang malar.
Boyle’s Law: pressure is inversely proportional to the volume of gas at constant temperature.
(b) Hukum Charles: Isi padu gas berkadar terus dengan suhu gas pada tekanan yang malar.
Charles’ law: Volume of gas is directly proportional to the temperature of the gas at constant pressure .
(c) Hukum tekanan (Gay-Lussac): Tekanan gas berkadar terus dengan suhu pada isi padu yang
malar.
Pressure law (Gay-Lussac): Pressure of the gas is directly proportional to the temperature at constant
volume of gas.

© Nilam Publication Sdn. Bhd. 100

B4FizikF4(81-114)csy5p.indd 100 09/01/2020 9:57 AM


MODUL • Fizik TINGKATAN 4

SP 4.4.2 Mengeksperimen untuk menentukan hubungan antara tekanan dan isi padu bagi suatu gas berjisim tetap pada suhu malar

Eksperimen Hubungan antara isi padu gas dan tekanan gas


Experiment Relationship between volume of gas and pressure of gas

Inferens Isi padu gas pada suhu yang tetap mempengaruhi tekanan gas.
Inference Volume of the the gas at constant temperature affects the pressure of gas.
Hipotesis Apabila isi padu gas menurun, tekanan gas meningkat.
Hypothesis When the volume of the gas decreases, pressure increases.
Untuk menyiasat hubungan antara isi padu dan tekanan untuk jisim gas yang tetap
Tujuan pada suhu yang malar.
Aim To investigate the relationship between the volume and pressure for a fixed mass of gas at
constant temperature.
Pemboleh ubah dimanipulasikan: / Manipulated variable:
Isi padu, V / Volume, V

Pemboleh ubah Pemboleh ubah bergerak balas: / Responding variable:


Variables Tekanan, P / Pressure, P
Pemboleh ubah dimalarkan: / Constant variable:
Jisim dan suhu udara / Mass and temperature of air

Senarai radas Pam basikal, tiub kapilari, tiub getah dan tolok Bourdon.
dan bahan Bicycle pump, capillary tube, rubber tube and Bourdon gauge.
List of apparatus
and materials
Tiub kapilari
cm3
Capillary tube
Skala isi padu
Volume scale Tolok
Bourdon
Udara Bourdon
Air Pam
Susunan radas gauge Pump
Arrangement of the Minyak
apparatus Oil

U
U
Tangki minyak N
N
II
Oil reservoir T
T

1. Radas disediakan seperti dalam rajah di atas. 14


The apparatus is set up as shown in the diagram above.
2. Pam ditekan supaya aras minyak pada tiub kapilari adalah 20 cm3. Isi padu direkodkan.
The pump is pushed so that the level of oil in the capillary tube is 20 cm3. The volume is recorded.

Prosedur 3. Bacaan tekanan udara pada tolok Bourdon dicatatkan.


Procedure The reading of pressure of air on Bourdon gauge is recorded.
4. Eksperimen diulangi dengan isi padu, V = 25 cm3, 30 cm3, 35 cm3 dan 40 cm3.
The experiment is repeated with volume, V = 25 cm3, 30 cm3, 35 cm3 and 40 cm3.
1
5. Nilai-nilai isi padu, V, tekanan, P, dan V dijadualkan.
1
The values of volume, V, pressure, P, and V are tabulated.

101 © Nilam Publication Sdn. Bhd.

B4FizikF4(81-114)csy5p.indd 101 09/01/2020 9:57 AM


MODUL • Fizik TINGKATAN 4

Isi padu, V (cm3) Tekanan, P (kPa) 1 (cm–3)


Keputusan —
Volume, V (cm3) Pressure, P (kPa) V
Results
100
120
140
160
180

1 diplot dengan menggunakan


1 Graf tekanan, P melawan isi padu, V dan graf P melawan —
kertas graf. V
1 are plotted by using a graph
Graph of pressure, P against volume, V and graph of P against —
Analisis data V
paper.
Analysis of data
2 Adakah hipotesis diterima? / Is the hypothesis accepted?
Ya / Yes

Kesimpulan Apabila isi padu gas menurun, tekanan gas meningkat.


Conclusion When the volume of the gas decreases, the pressure of the gas increases.

SP 4.4.3 Mengeksperimen untuk menentukan hubungan antara isi padu dan suhu bagi suatu gas berjisim tetap pada tekanan malar

Eksperimen Hubungan antara suhu gas dan isi padu gas


Experiment Relationship between temperature of gas and volume of gas

Inferens Suhu gas mempengaruhi isi padu gas.


Inference Temperature of the gas affects the volume of gas.
Hipotesis Apabila suhu gas meningkat, isi padu gas juga meningkat.
Hypothesis When the temperature of the gas increases, volume of gas also increases.
U
N Untuk menyiasat hubungan antara suhu dan isi padu untuk jisim gas yang tetap pada
Tujuan
I tekanan malar. / To investigate the relationship between the temperature and volume for a fixed
T Aim
mass of gas at constant pressure.
4
Pemboleh ubah dimanipulasikan: / Manipulated variable:
Suhu, θ / Temperature, θ

Pemboleh ubah Pemboleh ubah bergerak balas: / Responding variable:


Variables Isi padu, V / Volume, V
Pemboleh ubah dimalarkan: / Constant variable:
Jisim dan tekanan udara / Mass and pressure of air

Tiub kapilari yang mengandungi udara terperangkap oleh satu turus asid sulfurik pekat,
Senarai radas pembaris, kaki retort, bikar, tungku kaki tiga, kasa dawai, pengacau, termometer, penunu
dan bahan Bunsen, air, ais dan gelang getah.
List of apparatus Capillary tube containing air trapped by a column of concentrated sulphuric acid, metre rule,
and materials retort stand, beaker, tripod stand, wire gauze, stirrer, thermometer, Bunsen burner, water, ice and
rubber band.

© Nilam Publication Sdn. Bhd. 102

B4FizikF4(81-114)csy5p.indd 102 09/01/2020 9:57 AM


MODUL • Fizik TINGKATAN 4

Pembaris / Metre rule


Tiub kapilari / Capillary tube
Termometer
Thermometer Pengacau / Stirrer
Ais / Ice
Susunan radas
l Bikar / Beaker
Arrangement of the
Air Asid sulfurik pekat / Concentrated sulphuric acid
apparatus Water Gas terperangkap
Trapped air

1. Radas disediakan seperti rajah di atas.


The apparatus is set up as shown in the diagram above.
2. Air di dalam bikar dipanaskan perlahan-lahan dan dikacau dengan seragam dan
perlahan. Apabila bacaan termometer itu adalah 30 °C, panjang turus udara yang
terperangkap, l cm, direkodkan.
Prosedur
The water in the beaker is heated slowly and stirred unifromly and gently. When the reading
Procedure
of the thermometer is 30 °C, the length of the trapped air, l cm is recorded.
3. Eksperimen diulangi dengan suhu, θ = 40 °C, 50 °C, 60 °C dan 70 °C.
The experiment is repeated with θ = 40 °C, 50° C, 60 °C and 70 °C.
4. Nilai-nilai panjang turus udara, l, dijadualkan.
The values of length of column of air l, are tabulated.

Suhu, θ (oC) Panjang turus udara, l (cm)


Temperature, θ (oC) Length of column of air, l (cm)
30
Keputusan 40
Results
50
U
U
60 N
N
II
T
70 T
14
1 Graf panjang turus udara, l melawan suhu, θ diplot dengan menggunakan kertas graf.
Graph of length of column of air, l against temperature, θ is plotted by using a graph paper.
Analisis data
Analysis of data 2 Adakah hipotesis diterima?
Is the hypothesis accepted?
Ya / Yes

Kesimpulan Apabila suhu gas meningkat, isi padu gas meningkat.


Conclusion When the temperature of the gas increases, the volume of the gas increases.

103 © Nilam Publication Sdn. Bhd.

B4FizikF4(81-114)csy5p.indd 103 09/01/2020 9:57 AM


MODUL • Fizik TINGKATAN 4

SP 4.4.4 Mengeksperimen untuk menentukan hubungan antara tekanan dan suhu bagi suatu gas berjisim tetap pada isi padu malar

Eksperimen Hubungan antara suhu gas dan tekanan gas


Experiment Relationship between temperature of gas and pressure of gas

Inferens Suhu gas mempengaruhi tekanan gas.


Inference The temperature of gas affects pressure of the gas.

Hipotesis Apabila suhu gas meningkat, tekanan gas meningkat.


Hypothesis When the temperature of the gas increases, pressure of gas increases.

Untuk menyiasat hubungan antara suhu dengan tekanan untuk jisim gas yang
Tujuan ditetapkan pada isi padu malar.
Aim To investigate the relationship between the temperature and pressure for a fixed mass of gas at
constant volume.

Pemboleh ubah dimanipulasikan:


Manipulated variable:
Suhu, θ / Temperature, θ

Pemboleh ubah Pemboleh ubah bergerak balas:


Responding variable:
Variables
Tekanan, P / Pressure, P

Pemboleh ubah dimalarkan:


Constant variable:
Jisim dan isi padu udara / Mass and volume of air

Senarai radas dan Kelalang dasar bulat, bikar, termometer, kaki retort, tolok Bourdon, pengacau, air, ais,
bahan tiub getah, tungku kaki tiga, kasa dawai dan penunu Bunsen
List of apparatus Round-bottomed flask, beaker, thermometer, retort stand, Bourdon gauge, stirrer, water, ice,
and materials rubber tube, tripod stand, wire gauze and Bunsen burner.

U
N
I Termometer Pengacau Tiub getah
T Thermometer Stirrer Rubber tube
4
Kaki retort
Retort stand
Tolok Bourdon
Susunan radas Ais / Ice Bourdon gauge
Arrangement of the Bikar / Beaker Air
Water
apparatus Udara / Air
Xxxxxxxxxxxxxxxxxx Kasa
Kelalang dasar bulat dawai
Round-bottomed flask Wire
gauze

© Nilam Publication Sdn. Bhd. 104

B4FizikF4(81-114)csy5p.indd 104 09/01/2020 9:57 AM


MODUL • Fizik TINGKATAN 4

1. Radas disediakan seperti yang ditunjukkan dalam rajah di atas.


The apparatus is set up as shown in the diagram above.
2. Campuran air dan ais dikacau secara berterusan supaya udara di dalam kelalang
mempunyai suhu yang sama seperti air.
The mixture of water and ice is stirred continuously so that the air in the flask has the same
temperature as the water.
4. Apabila bacaan termometer adalah 30 °C, bacaan tekanan, P, pada tolok Bourdon
Prosedur direkodkan.
Procedure When the reading of the thermometer is 30 °C, the reading of the pressure, P on Bourdon
gauge is recorded.
5. Eksperimen diulang dengan suhu air yang berlainan, iaitu θ = 40 °C, 50 °C, 60 °C dan
70 °C.
The experiment is repeated with different temperatures of water, that is, θ = 40 °C, 50°C, 60 °C
and 70 °C.
6. Nilai-nilai tekanan udara, P dijadualkan.
The values of pressure of the air, P is tabulated.

Suhu, θ (°C) Tekanan udara, P (kPa)


Temperature, θ (°C) Air pressure, P (kPa)

30
Penjadualan data 40
Tabulate the data
50
60
70
U
U
N
N
II
T
T
1 Graf tekanan, P melawan suhu, θ diplot dengan menggunakan kertas graf.
Graph of pressure, P against temperature, θ is plotted by using a graph paper. 14
Analisis data
Analysis of data 2 Adakah hipotesis diterima?
Is the hypothesis accepted?
Ya / Yes

Kesimpulan Apabila suhu gas meningkat, tekanan gas meningkat.


Conclusion When the temperature of the gas increases, pressure of gas increases.

105 © Nilam Publication Sdn. Bhd.

B4FizikF4(81-114)csy5p.indd 105 09/01/2020 9:57 AM


I
T
N
U

4
Rumusan bagi 3 hukum gas / Summary of 3 gas laws

Hukum Boyle / Boyle’s Law: Hukum Charles / Charles’ Law: Hukum Tekanan: / Pressure Law:
1
(m & T – pemalar / constant: P ∝ ) (m & P – pemalar / constant: V ∝ T) (m & V – pemalar / constant : P ∝ T)
V

B4FizikF4(81-114)csy5p.indd 106
Omboh Haba / Heat
Haba / Heat
Piston
MODUL • Fizik TINGKATAN 4

P1, V1 P2, V2
V1, T1 V2, T2 P1, T1 P2, T2

© Nilam Publication Sdn. Bhd.


(i) Penerangan dengan menggunakan Teori Kinetik (i) Penerangan menggunakan Teori Kinetik Jirim: (i) Penerangan menggunakan Teori Kinetik Jirim:
Jirim: Explanation using Kinetic Theory of Matter: Explanation using Kinetic Theory of Matter:
Explanation using Kinetic Theory of Matter: • Apabila gas dipanaskan, suhu akan •
Apabila gas itu dipanaskan, suhu
• Apabila isi padu bekas berkurang, isi padu meningkat . (T > T ) meningkat . (T > T )
2 1
2 1
gas juga berkurang . (V < V )
When the gas is heated, its temperature
2 1 When the gas is heated, temperature increases
When the volume of the container decreases, the (T2 > T1) increases . (T > T )
2 1
volume of the gas also decreases . (V < V )
2 1 • Isi padu gas juga bertambah menyebabkan Isi padu bekas dikekalkan malar, jadi isi

• Kadar perlanggaran antara molekul gas kadar perlanggaran molekul gas dengan padu gas adalah tetap .
dengan dinding bekas bertambah . dinding bekas tidak berubah . The volume of the container is kept constant, so
The rate of collision between the gas molecules The volume of the gas also increases to keep the the volume of the gas is constant .

106
rate of collision between the gas molecules and
and the walls of the container increases . Kadar perlanggaran di antara molekul gas

the walls of the container is constant .
• Tekanan gas turut bertambah . meningkat
dengan dinding bekas itu .
increases • Tekanan gas adalah tetap . The rate of collision between the gas molecules and
The gas pressure .
• Tenaga kinetik gas tetap, maka suhu gas The gas pressure is constant . the walls of the container increases .
juga tetap
. T2 > T1 dan / and V2 > V1 meningkat
• Tekanan gas .
The kinetic energy of the gas is constant, so the
(ii) Hukum Charles menyatakan bahawa untuk increases
constant The gas pressure .
temperature of the gas is . jisim gas yang tetap, isi padu gas berkadar
T2 > T1 dan / and P2 > P1
V2 < V1 dan / and P2 > P1 terus dengan suhu mutlaknya jika
(ii) Hukum Tekanan menyatakan bahawa bagi suatu
(ii) Hukum Boyle menyatakan bahawa untuk tekanan gas itu adalah tetap.
Charles' Law states that for a fixed mass of gas, the jisim gas yang tetap, tekanan gas adalah berkadar
jisim gas yang tetap, tekanan gas berkadar terus
songsang volume of the gas is directly proportional to the dengan suhu mutlak gas jika isi
dengan isi padu gas jika suhu padu gas adalah tetap.
adalah malar. absolute temperature of the gas if the pressure is kept
constant. Pressure Law states that for a fixed mass of gas, the
Boyle's Law states that for a fixed mass of gas, the directly
inversely pressure of gas is proportional to
gas pressure is proportional to the the absolute temperature of the gas if the volume of the
volume of the gas if the temperature is kept constant. gas is kept constant.

09/01/2020 9:57 AM
(iii) Hubungan / Relationship: (iii) Hubungan / Relationship: (iii) Hubungan / Relationship:
Tekanan gas (P) berkadar songsang Isi padu gas (V) adalah berkadar terus Tekanan gas (P) berkadar terus dengan
dengan isi padu gas (V) dengan suhu mutlak gas (T) suhu mutlak gas (T).
inversely directly directly

B4FizikF4(81-114)csy5p.indd 107
Gas pressure (P) is proportional to Volume of gas (V) is proportional to the Pressure of gas (P) is proportional to
the gas volume (V) absolute temperature of gas (T) the absolute temperature of gas (T).
1 V∝T P∝T
P∝
V
(iv) Persamaan / Equation: (iv) Persamaan / Equation: (iv) Persamaan / Equation:
1 V = k (T) ; k adalah malar / is a constant P = k (T) ; k adalah malar / is a constant
P = k ( ) ; k adalah malar / is a constant
V
V P
k = PV = malar / constant k = = pemalar / constant k = = pemalar / constant
T T
∴ P1V1 = P2V2 V V P P
∴ 1 = 2 ∴ 1 = 2
T1 T2 T1 T2
(v) Graf / Graph:
(v) Graf / Graph: (v) Graf / Graph:
(a) P
P berkadar (a) V / cm3 (a) P / Pa P berkadar
songsang V berkadar terus
dengan V terus dengan T
P is inversely dengan T P is directly
proportional to V V is directly proportional to T

107
V proportional to T T/K
0
T/K 0
0
(b) P (b)
V / cm3 P / Pa
P berkadar (b)
P berubah secara
terus V berubah secara linear dengan θ
1
dengan __ linear dengan θ P varies linearly
V V varies linearly with θ
P is directly with θ
1
__ 1 θ / °C
0 V proportional to __ θ / °C –273 0
V –273 0
(c) PV V (c) P
(c) __
T T
__
P adalah tetap
PV adalah tetap __
V adalah tetap T
PV is constant T __
P is constant
__
V is constant T
T
V
0 0 T
T
0

© Nilam Publication Sdn. Bhd.


MODUL • Fizik TINGKATAN 4

II
T
T
N
U
N
U

14

09/01/2020 9:57 AM
MODUL • Fizik TINGKATAN 4

Menyelesaikan masalah melibatkan tekanan, suhu dan isi padu suatu gas yang berjisim tetap dengan menggunakan rumus dari Hukum-
SP 4.4.5
hukum Gas

Latihan / Exercises
1 Isi padu gelembung udara pada dasar laut sedalam 3 Satu campuran udara dan wap petrol disuntik
50 m ialah 250 cm3. Jika tekanan atmosfera ialah ke dalam enjin silinder kereta apabila isi padu
10 m air, cari isi padu gelembung udara apabila ia silinder itu ialah 100 cm3. Tekanan adalah 1.0 atm.
tiba ke permukaan laut. / The volume of an air bubble Injap ditutup dan campuran dimampatkan kepada
at a 50 m deep seabed is 250 cm3. If the atmospheric 20 cm3. Cari tekanan sekarang.
pressure is 10 m of water, find the volume of the air A mixture of air and petrol vapour is injected into
the cylinder of a car engine when the volume of the
bubble when it reaches the surface of the sea.
cylinder is 100 cm3. Its pressure is 1.0 atm. The valve
Penyelesaian / Solution: is closed and the mixture is compressed to 20 cm3.
P2 = 10 m air / water Find the current pressure.
Penyelesaian / Solution:
P1 = (50 m + 10 m) air / water Menggunakan Hukum Boyle, / Using Boyle's Law,
= 60 m air / water P1V1 = P2V2
1 atm × 100 cm3 = P2 × 20 cm3
V1 = 250 cm3 100 cm3
P2 = × 1 atm = 5 atm
Menggunakan Hukum Boyle, / Using Boyle's Law, 20 cm3
P1V1 = P2V2
(60 m) (250 cm3) = (10 m) × V2 4 Isi padu gas 20 m3 pada suhu 37 °C dipanaskan
(60 m × (250 cm3) sehingga menjadi 87 °C pada tekanan malar.
V2 = Berapakah peningkatan isi padunya?
10 m
A gas of volume 20 m3 at 37 °C is heated until its
= 1 500 cm3 temperature becomes 87 °C at constant pressure. What
is the increase in volume?
2 Rajah di bawah menunjukkan tiub kaca yang Penyelesaian / Solution:
mengandungi udara yang terperangkap di dalamnya. Menggunakan Hukum Charles, / Using Charles' Law,
Pada 17 °C, turus udara yang terperangkap ialah V1 V2
=
29 cm. T1 T2
The diagram below shows a glass tube containing some 20 m3 V2
=
trapped air inside it. At 17 °C, the vertical column of (37 + 273) K (87 + 273) K
trapped air is 29 cm. 20 m3 × 360 K
V2 = = 23.23 m3
310 K
Merkuri 5 cm Peningkatan isi padu / Increase in volume
U Mercury
N = (23.23 – 20) m3 = 3.23 m3
I Udara
T 29 cm pada/at 17 °C
Air 5 Tekanan udara di dalam bekas pada 33 °C adalah
4 1.4 × 105 N m–2. Bekas itu dipanaskan sehingga
Berapakah panjang turus udara yang terperangkap suhu 55 °C. Berapakah tekanan udara akhir jika
pada suhu 57 °C? / What is the vertical column of isi padu bekas ditetapkan? / The air pressure in a
trapped air at a temperature of 57 °C? container at 33 °C is 1.4 × 105 N m–2. The container is
heated until the temperature is 55 °C. What is the final
Penyelesaian / Solution:
air pressure if the volume of the container is fixed?
Menggunakan Hukum Charles, / Using Charles' Law,
V1 V2 Penyelesaian / Solution:
= di mana A = luas keratan Menggunakan Hukum Tekanan, / Using Pressure Law,
T1 T2 rentas tiub
P1 P2
A (L1) A (L2) where A = cross-sectional
=
= , area of the tube T1 T2
T1 T2
29 cm L2 1.4 × 105 N m–2 P2
= =
(17 + 273) K (57 + 273) K (33 + 273) K (55 + 273) K
1.4 × 105 N m–2 × 328 K
29 cm × 330 K P2 =
L2 = 306 K
290 K
= 1.5 × 10 N m–2 5
= 33 cm

© Nilam Publication Sdn. Bhd. 108

B4FizikF4(81-114)csy5p.indd 108 09/01/2020 9:57 AM


MODUL • Fizik TINGKATAN 4

Praktis SPM / SPM Practice


Soalan Objektif / Objective Questions
1 Rajah 1 menunjukkan satu sudu besi pada suhu 3 Rajah 3 menunjukkan susunan radas terdiri
bilik direndam dalam air panas pada 70 °C. daripada penitis dan botol dengan lembaran getah.
Diagram 1 shows an iron spoon at room temperature Diagram 3 shows the arrangement of apparatus made
being immersed in hot water at 70 ºC. up of a dropper and bottle with a rubber sheet.
Lembaran getah
Rubber sheet

Botol
Sudu besi Air Bottle
Iron spoon Water Penitis
Dropper Air
Water
Rajah 1 / Diagram 1
Keseimbangan terma tercapai apabila Rajah 3 / Diagram 3
The termal equilibrium is reached when Apabila lembaran getah ditekan,
A suhu air = suhu sudu. When the rubber sheet is pressed,
the temperature of the water = the temperature of A penitis ternaik.
the spoon. the dropper rises.
B jisim sudu = jisim air yang tersesar. B air memasuki tiub penitis.
the mass of the spoon = the mass of water displaced. water enters the dropper tube.
C isi padu sudu = isi padu air yang tersesar. C tekanan di dalam botol berkurang.
the volume of the spoon = the volume of water the pressure inside bottle decreases.
displaced. D daya tujah penitis berkurang.
D muatan haba tentu sudu = muatan haba tentu the upthrust on the dropper decreases.
air.
the specific heat capacity of the spoon = the specific
heat capacity of water. 4 Graf pada Rajah 4 menunjukkan perubahan suhu
ais apabila ia dipanaskan.
2 Rajah 2 menunjukkan blok pepejal M dan N yang The graph in Diagram 4 shows the change in
berjisim sama sedang dipanaskan. Suhu awal M temperature of ice when it is heated.
dan N adalah sama dan dipanaskan dengan jumlah Suhu / Temperature (°C)
tenaga yang sama. / Diagram 2 shows solid blocks
M and N of equal mass, are being heated. The initial
temperatures of M and N are the same and they are
heated by the same amount of energy. U
U
D N
N
C II
Pengadang haba B T
T
Heat shield A 14
M N Masa / Time (s)
Rajah 4 / Diagram 4
Dapur Antara titik A, B, C atau D, yang manakah air
Stove
wujud sebagai campuran cecair dan gas?
Rajah 2 / Diagram 2 At which points, A, B, C or D, does water exist as
Dapat diperhatikan bahawa M panas lebih cepat mixture of a liquid and a gas?
daripada N. Pemerhatian ini adalah disebabkan
oleh perbezaan dalam / It is observed that M is hot
faster than N. This observation is due to the difference in
A ketumpatan. / density.
B takat lebur. / melting point.
C haba pendam tentu pelakuran.
latent heat of fusion.
D muatan haba tentu. / specific heat capacity.

109 © Nilam Publication Sdn. Bhd.

B4FizikF4(81-114)csy5p.indd 109 09/01/2020 9:57 AM


MODUL • Fizik TINGKATAN 4

5 Suhu air mendidih ialah 100 °C. Jika air mendidih 7 Rajah 6 menunjukkan satu turus udara terperangkap
pada suhu lebih tinggi daripada 100 °C, apakah di dalam tiub kapilari oleh benang merkuri. Tiub
sebabnya? rerambut diletakkan di dalam tiga keadaan yang
The temperature at which water boils is 100 °C. If water berbeza K, L dan M.
boils at a temperature higher than 100 °C, what is the Diagram 6 shows an air column trapped inside a
reason? capillary tube by a thread of mercury. The capillary tube
A Air adalah tulen. is placed in three different situations K, L and M.
The water is pure.
B Isi padu air besar.
L
The volume of the water is greater.
C Air mengandungi bendasing.
The water contains impurities.
D Air mendidih di dalam bekas logam yang
merupakan pengalir haba yang baik. K M
The water is boiled in a metal container which is a Rajah 6 / Diagram 6
good conductor of heat.
Tekanan udara di K, L dan M masing-masing P1,
P2 dan P3. Antara perbandingan berikut, yang
manakah adalah betul?
6 Rajah 5 menunjukkan satu omboh kedap udara.
The air pressure in K, L and M are P1, P2 and P3
Diagram 5 shows an airtight piston.
respectively. Which of the following comparison is
Udara / Air correct?
A P1 = P2 = P3 C P1 < P2 < P3
B P1 > P2 > P3 D P1 = P2 > P3

10 cm
8 Rajah 7 menunjukkan pembentukan bayu darat.
Klip Omboh kedap udara Diagram 7 shows the formation of a land breeze.
Clip Airtight piston
Rajah 5 / Diagram 5 Waktu malam
Night time
Ia adalah sukar untuk menolak omboh ke hadapan
ke dalam picagari apabila klipnya ditutup. Ini adalah Udara panas
Udara sejuk Hot air
kerana Cool air
It is difficult to push the piston forward clip when its is
closed. This is because
A rintangan geseran antara silinder dan omboh. Darat / Land Laut / Sea
the frictional resistance between the cylinder and
the piston Rajah 7 / Diagram 7
U
N B molekul udara dalam silinder berlanggar Antara pernyataan berikut, yang manakah betul?
I Which of the following statements is correct?
T dengan omboh pada kadar yang lebih tinggi.
4 the air molecules in the cylinder collide with the
piston at higher rate.
A Muatan haba air laut lebih tinggi daripada
muatan haba darat.
C daya tolakan antara molekul-molekul udara. The heat capacity of the sea water is higher than
of the forces of repulsion between the air molecules. the heat capacity of the land.
D bilangan molekul-molekul udara di dalam B Tekanan laut adalah lebih tinggi daripada di
silinder bertambah. atas darat.
the number of air molecules in the cylinder increases. The pressure of the sea is higher than that on the
land.
C Suhu darat menjadi lebih tinggi daripada suhu
air laut.
The temperature of the land becomes higher than
the temperature of the sea water.
D Ketumpatan darat kurang daripada air laut.
The density of the land is less than that of sea water.

© Nilam Publication Sdn. Bhd. 110

B4FizikF4(81-114)csy5p.indd 110 09/01/2020 9:57 AM


MODUL • Fizik TINGKATAN 4

9 Berapakah haba yang diperlukan untuk menukarkan 12 Haba pendam tentu pengewapan air ialah haba
0.2 kg air mendidih kepada wap? / How much heat is yang diserap oleh
needed to change 0.2 kg of boiling water to vapour? The specific latent heat of vaporisation of water is the
(Haba pendam tentu pengewapan air ialah heat absorbed by
2.26 × 106 J kg–1) A 1 kg air untuk menaikkan suhu sebanyak 1 °C.
(Specific latent heat of vaporisation of steam is 1 kg of water to increase the temperature by 1 °C.
2.26 × 106 J kg–1) B 1 m3 air untuk menaikkan suhu sebanyak 1 °C.
A 452 kJ C 113 kJ 1 m3 of water to increase the temperature by 1 °C.
B 500 kJ D 1.33 kJ C 1 kg air semasa mendidih untuk menukarkannya
kepada stim.
10 Sebuah pemanas berkuasa 1 200 W sedang 1 kg of water during boiling to change it to steam.
memanaskan suatu cecair pada takat didihnya. Jika D 1 m3 air semasa mendidih untuk menukarkannya
haba pendam tentu pengewapan cecair itu ialah kepada stim.
2.0 × 106 J kg–1, berapakah jisim cecair itu yang telah 1 m3 of water during boiling to change it to steam.
ditukarkan kepada wap setelah dipanaskan selama
10 minit? / An electric heater with a power of 1 200 W is
heating a liquid at its boiling point. If the specific latent 13 Antara graf berikut, yang manakah menunjukkan
heat of vaporisation of the liquid is 2.0 × 106 J kg–1, what is hubungan antara isi padu, V, dengan suhu, T, bagi
the mass of the liquid that has changed into vapour after suatu gas yang berjisim tetap pada tekanan yang
10 minutes? malar?
A 0.18 kg C 3.0 kg Which of the following graphs shows the relationship
B 0.36 kg D 5.6 kg between the volume, V, of a fixed mass of gas with its
temperature, T, at constant pressure?
11 Rajah 8(a) dan Rajah 8(b) menunjukkan suatu A V/m3 V/m3
eksperimen untuk menentukan haba pendam
tentu pelakuran ais. Rajah 8(b) adalah eksperimen
kawalan.
Diagram 8(a) and Diagram 8(b) show an experiment
being used to determine the specific latent heat of fusion T/K T/K
0 0
of ice. Diagram 8(b) is a control experiment.
Pemanas V/m3 B V/m
V/m33 V/m33
V/m
rendam
Immersion
heater
Ais Ais
Ice Ice
T/K T/K
T/K T/K
T/K
0 0 0 U
U
N
N
V/m
V/m 33
C V/m
V/m 33
V/m3 II
T
Air Air T

(a)
Water
(b)
Water 14

Rajah 8 / Diagram 8
Eksperimen kawalan adalah untuk T/K T/K
T/K T/K T/K
The control experiment is to 0 0 0
A mengawal kadar peleburan ais. V/m3
control the rate of melting of ice. D V/m3

B memastikan air yang digunakan adalah tulen.


ensure that the water used is pure.
C menentukan nilai purata bagi haba pendam
tentu pelakuran ais.
T/K T/K
determine the average value of the 0specific latent 0
heat of fusion of ice.
D menentukan jisim ais yang melebur tanpa
pemanasan daripada pemanas rendam itu.
determine the mass of ice which melts without
heating it with the heater.

111 © Nilam Publication Sdn. Bhd.

B4FizikF4(81-114)csy5p.indd 111 09/01/2020 9:57 AM


MODUL • Fizik TINGKATAN 4

Soalan Struktur / Structure Questions


1 Rajah 1.1 dan Rajah 1.2 menunjukkan gas, X, yang berjisim tetap terperangkap dalam dua kelalang serupa yang
dipanaskan dengan kuantiti haba yang berbeza.
Diagram 1.1 and Diagram 1.2 show that a fixed mass of trapped gas, X, in two identical flasks heated with different
quantities of heat.

Tolok Tolok
Bourdon Bourdon
Termometer Termometer
Bourdon Bourdon
Thermometer Thermometer
Kelalang gauge Kelalang gauge
Pengacau Pengacau
Flask Flask
Stirrer Stirrer
Air dan ais Blok Air dan ais Blok
Water and ice Udara kayu Water and ice Udara kayu
Air Wooden Air Wooden
block block
Dapur Dapur
Stove Stove
Rajah 1.1 / Diagram 1.1 Rajah 1.2 / Diagram 1.2
(a) Nyatakan kuantiti fizik dan unit S.I.nya yang diukur oleh tolok Bourdon itu.
State the physical quantity and its S.I. unit measured by the Bourdon gauge.

Tekanan udara. Pascal atau N m–2 / Air pressure. Pascal or N m–2

(b) Kedua-dua dapur ditutup selepas 8 minit. Berdasarkan Rajah 1.1 dan Rajah 1.2,
Both stoves are turned off after 8 minutes. Based on Diagram 1.1 and Diagram 1.2,
(i) bandingkan bacaan kedua-dua tolok Bourdon itu. / compare the readings of both Bourdon gauges.
Bacaan tolok Bourdon dalam Rajah 1.2 lebih tinggi daripada bacaan tolok Bourdon dalam Rajah 1.1.
The reading of Bourdon gauge in Diagram 1.2 is higher than that of Diagram 1.1.
(ii) bandingkan bacaan kedua-dua termometer. / compare the readings of both thermometers.
Bacaan termometer dalam Rajah 1.2 lebih tinggi daripada bacaan termometer dalam Rajah 1.1.
The reading of the thermometer in Diagram 1.2 is higher than that of Diagram 1.1.
(iii) nyatakan hubungan antara tekanan gas, X dalam kelalang dengan suhu.
U state the relationship between the pressure of gas, X in the flask and the temperature.
N
I
T Apabila suhu menaik, tekanan gas, X dalam kelalang menaik.
4 When the temperature increases, the pressure of gas, X in the ask increases.

(c) Berdasarkan teori kinetik, terangkan sebab bagi jawapan di 1(b)(iii).


Based on kinetic theory, explain the reason for the answer in 1(b)(iii).

Apabila molekul-molekul gas dalam kelalang menerima haba, halaju molekul-molekul gas bertambah.

Ini menyebabkan tenaga kinetik molekul-molekul gas itu bertambah. Apabila tenaga kinetik molekul-

molekul gas bertambah, frekuensi perlanggaran di antara molekul-molekul gas dengan dinding kelalang

bertambah, maka tekanan gas dalam kelalang bertambah.

When the gas molecules in the flask receive heat, the velocity of these molecules increases. This causes the kinetic

energy of the gas molecules to increase.When the kinetic energy of the gas molecules increases, the frequency of
collision between the gas molecules and the walls of the flask increases, thus the gas pressure in the flask increases.

© Nilam Publication Sdn. Bhd. 112

B4FizikF4(81-114)csy5p.indd 112 09/01/2020 9:57 AM


MODUL • Fizik TINGKATAN 4

(d) Namakan hukum yang terlibat apabila isi padu gas, X dimalarkan.
Name the law involved when the volume of gas, X is constant.

Hukum Tekanan / Pressure Law

2 Gelembung udara yang terhasil di dasar akuarium naik dari kedudukan X ke kedudukan Y. Didapati saiz
gelembung udara tersebut menjadi lebih besar apabila ia semakin menghampiri permukaan air.
The air bubbles produces at the base of aquarium rising up from position X to position Y. It is noticed that the size of the
air bubbles becomes bigger as it approaches the surface of water.

Bekalan oksigen
Oxygen supply

Y Gelembung udara
Air bubbles

Akuarium ikan / Fish aquarium

(a) Banding: / Compare:


(i) Kedalaman gelembung udara di kedudukan X dan kedudukan Y.
The depth of air bubble at position X and position Y.
Kedalaman gelembung udara di kedudukan X > Kedalaman gelembung udara di kedudukan Y
The depth of air bubble at position X > The depth of air bubble at position Y

(ii) Tekanan yang dialami oleh gelembung udara di kedudukan X dan kedudukan Y.
The pressure acting on the air bubble at positon X and position Y.
Tekanan yang dialami oleh gelembung udara di kedudukan X > Tekanan yang dialami oleh
gelembung udara di kedudukan Y
The pressure acting on the air bubble at position X > The pressure acting on the air bubble at position Y

(iii) Isi padu gelembung udara di kedudukan X dan kedudukan Y. U


U
N
The volume of the air bubble at position X and position Y. N
II
T
T
Isi padu gelembung udara di kedudukan X < Isi padu gelembung udara di kedudukan Y
14
The volume of the air bubble at position X < The volume of the air bubble at position Y

(iv) Suhu udara gelembung udara di kedudukan X dan kedudukan Y.


The temperature of the air bubble at position X and position Y.
Suhu udara gelembung udara di kedudukan X = Suhu udara gelembung udara di kedudukan Y
The temperature of the air bubble at position X = The temperature of the air bubble at position Y

(b) Hubung kaitkan: / Relate:


(i) kedalaman gelembung udara dengan tekanan udara di dalam gelembung.
the depth of the air bubble and air pressure in the air bubble.
Semakin bertambah kedalaman gelembung udara, semakin bertambah tekanan udara di dalam
gelembung udara / As the depth of the air bubble increases, the pressure inside the air bubble increases

113 © Nilam Publication Sdn. Bhd.

B4FizikF4(81-114)csy5p.indd 113 09/01/2020 9:58 AM


MODUL • Fizik TINGKATAN 4

(ii) tekanan udara di dalam gelembung dengan isi padu gelembung udara.
the air pressure inside the air bubble and the volume of air bubble.
Semakin bertambah tekanan udara di dalam gelembung, semakin berkurang isi padu gelembung
udara ATAU sebaliknya
As the pressure inside the air bubble increases, the volume of air bubble decreases OR vice versa

(c) Berdasarkan jawapan di (b)(ii), deduksikan hukum fizik.


Based on your answer in (b)(ii) , deduce the physics law.
Hukum Boyle / Boyle's law

KEFAHAMAN TEORI: / UNDERSTANDING THE THEORY:


PV = pemalar / contant
pemalar / contant
V =
P
• Untuk suhu gas malar, isi padu gas bergantung pada tekanan gas
As the temperature of gas is constant, the volume of gas depends on pressure
• Semakin bertambah tekanan gas, semakin berkurang isi padu gas ATAU sebaliknya
As the gas pressure increases, the volume of gas decreases OR vice versa
1

V berkadar songsang dengan P (V α )
P
1
V is inversely proportional to P (V α )
P
• Kedalaman, h, bertambah, tekanan ke atas gelembung udara, P, bertambah
As the depth, h increases, pressure on the air bubble, P increases

P berkadar langsung dengan h (P α h)
P is directly proportional to h (P α h)
Pemboleh ubah: / Variables:
Dimanipulasikan / Manipulated : Isi padu gas / Volume of gas
Bergerak balas / Responding : Tekanan gas / Gas pressure
Dimalarkan / Constant : Suhu gas / Temperature of gas
• Dari X ke Y, terangkan mengapa saiz gelembung udara tersebut menjadi lebih besar apabila ia semakin menghampiri
permukaan air)
From X to Y, explain why the size of air bubble becomes bigger as it approaches the surface of water
U
• Kedalaman air semakin berkurang
N The depth of water decreases
I • Pαh
T
1
4 • Vα
P
• Hukum Boyle / Boyle’s law

Maklumat tambahan:
Additional information:

© Nilam Publication Sdn. Bhd. 114

B4FizikF4(81-114)csy5p.indd 114 09/01/2020 9:58 AM


MODUL • Fizik TINGKATAN 4

Unit
GELOMBANG
5 WAVES

5.1 ASAS GELOMBANG / FUNDAMENTAL OF WAVES SK 5.1

SP 5.1.1 Memerihalkan gelombang

• Gangguan yang memindahkan tenaga antara dua titik melalui getaran zarah.
Nyatakan definisi
Disturbance that transmits energy between two points through particle vibration.
gelombang.
• Tiada pemindahan jirim apabila gelombang berlaku.
State the definition of waves.
No transferring of matter when waves happen.

Aktiviti : Mengkaji bahawa gelombang memindahkan tenaga tanpa memindahkan jirim.


Activity : To study on waves transfer energy without transferring matter.

Radas dan Bahan / Apparatus and Materials


Spring slinki dan reben / Slinky spring and ribbon
Reben
Ribbon

A B

Prosedur / Procedure
1 Reben diikat pada spring slinki dan dipegang oleh dua orang murid pada kedua-dua hujung spring.
A ribbon is tied to the slinky spring and hold by 2 pupils at the each end of the spring.
2 Hujung spring A digerakkan ke atas dan ke bawah manakala hujung spring B berada dalam keadaan pegun.
End A is moved up and down while end B is at the rest state.
3 Gerakan gelombang yang terbentuk sepanjang spring dan gerakan reben diperhatikan.
The movement of the waves formed along the spring and movement of the ribbon is observed.

Perbincangan / Discussion
1 Apabila spring digerakkan ke atas dan ke bawah, gelombang terhasil apabila sesuatu medium
digetarkan pada tempat tertentu. U
U
N
N
When the spring is moved up and down, waves are produced when a medium vibrates at fixed position. II
T
T

2 Murid pada hujung spring B merasai tenaga yang terhasil daripada gelombang tersebut.
15
The pupil at end B feels an energy is produced from the waves.

3 Reben yang mewakili jirim hanya bergerak ke atas dan ke bawah apabila gelombang pada spring terhasil.
Perambatan gelombang memindahkan tenaga dari satu tempat ke tempat lain tanpa memindahkan jirim .
Ribbon that represent matter only moves up and down when the waves on the spring are produced. Propagation of waves
transfers energy from one place to another without transferring matter .

115 © Nilam Publication Sdn. Bhd.


MODUL • Fizik TINGKATAN 4

SP 5.1.2 Menyatakan jenis gelombang

Nyatakan dua jenis gelombang. • Gelombang progresif / Progressive waves


State two types of waves. • Gelombang pegun / Stationary waves

• Gelombang progresif ialah gelombang di mana profil


gelombang merambat dengan masa.
Nyatakan definisi gelombang progresif
Progressive waves are waves where the wave profile propagate
dan jenis gelombang yang ada bagi
with time.
gelombang progresif.
• Terdapat dua jenis gelombang progresif:
State the definition of progressive waves and
the type of waves occur for progressive waves.
There are two types of progressive wave:
i) Gelombang melintang / Transverse waves
ii) Gelombang membujur / Longitudinal waves

• Gelombang pegun ialah gelombang di mana profil


gelombang tidak merambat dengan masa.
Contoh: gelombang yang terhasil daripada alat-alat muzik.
Stationary waves are waves where the wave profile do not
propagate with time.
Nyatakan definisi gelombang pegun dan
Example: waves that produced from musical instruments.
jenis gelombang yang ada bagi
• Terdapat dua jenis gelombang pegun:
gelombang pegun.
There are two types of stationary wave:
State the definition of stationary waves and
the type of waves occur for stationary waves.
i) Gelombang mekanik – gelombang air dan gelombang
bunyi.
Mechanical waves – water waves and sound waves.
ii) Gelombang elektromagnet – gelombang cahaya dan
gelombang radio.
Electromagnetic waves – light waves and radio waves.

Dua contoh sistem yang menghasilkan gelombang pegun:


Two examples of systems that produce stationary (standing) waves:

Gelombang pegun dalam tali yang terikat pada suatu penghujung


Standing waves in a string fixed at one end

Apabila suatu gelombang terusan dihantar di sepanjang Gelombang menuju / Incident wave
seutas tali yang terikat pada suatu penghujung, maka
kedua-dua gelombang menuju dan gelombang terpantul
akan terhasil pada tali itu. Bentuk gelombang yang
terhasil dipanggil gelombang pegun kerana gelombang
itu nampaknya pegun. Pada keseluruhannya, gelombang Gelombang terpantul / Reflected wave
U itu tidak progresif.
N When a continuous wave is sent along a string to a fixed
I
T position at the other end, then both incident and reflected
5 waves will be formed in the string. Each wave is travelling
in the opposite direction. The resultant waveform is called a
stationary or standing wave because it seems to be standing
Gelombang pegun yang terhasil
still. The wave as a whole is not progressive. Resultant standing wave

© Nilam Publication Sdn. Bhd. 116


MODUL • Fizik TINGKATAN 4

Gelombang pegun dalam sebuah paip tertutup / Standing waves in a closed pipe
Gelombang pegun juga boleh terbentuk dalam paip. Kedua-dua gelombang
menuju dan gelombang terpantul wujud dalam paip ini. Bentuk gelombang
yang terhasil dipanggil gelombang pegun kerana gelombang itu nampaknya
seperti pegun. Pada keseluruhannya, gelombang itu tidak progresif. Banyak
alat muzik bergantung kepada gelombang pegun di sepanjang tulus udara
dalam paip itu. Rajah di sebelah menunjukkan contoh gelombang pegun
dalam kes sebuah paip organ tertutup (contoh ini mempunyai suatu
penghujung yang tertutup).
Standing waves can also be set up in pipes. Both incident waves and reflected waves are present in the pipe. The resultant
waveform is called a stationary or standing wave because it seems to be standing still. The wave as a whole is not progressing.
Many musical instruments depend on standing waves along the air columns inside them. The diagram shows one example
of standing wave in a closed organ pipe (this example has one closed end).

SP 5.1.3 Membandingkan gelombang melintang dan gelombang membujur

Perbandingan antara gelombang melintang dan gelombang membujur


Comparison between transverse wave and longitudinal wave

Gelombang melintang Gelombang membujur


Transverse waves Longitudinal waves
Gelombang membujur ialah gelombang yang
Gelombang melintang ialah gelombang di mana
mana zarah-zarah medium bergetar pada
zarah-zarah medium bergetar pada arah yang
arah yang selari dengan arah
berserenjang dengan arah perambatan
Definisi perambatan gelombang.
Definition gelombang.
Longitudinal waves are waves in which the
Transverse waves are waves in which the particles of
particles of the medium vibrate in the direction
the medium in the direction perpendicular to parallel to the direction of propagation
the direction of propagation of wave.
of wave.

Arah getaran C Arah Arah perambatan gelombang


zarah perambatan Direction of wave propagation
The direction The direction
of vibration of
the particles of wave
T T propagation
C = Puncak / Crest C R C R C
T = Lembangan / Trough
Arah getaran zarah C = Mampatan / Compression
Rajah Direction of vibration R = Renggangan / Rarefaction
Dsiplacement of oscillation

Diagram of the particles


Panjang gelombang, λ
Wavelength, λ U
U
N
N
Arah getaran spring
Sesaran ayunan

II
Amplitud, a / Amplitude, a Direction of the vibration of the spring T
T
Jarak dari sumber
Distance from source 15
Titik dalam fasa yang sama
λ
Points in phase

Gelombang air, gelombang cahaya Gelombang bunyi


Contoh
Example Water waves, light waves Sound waves

117 © Nilam Publication Sdn. Bhd.


MODUL • Fizik TINGKATAN 4

Definisi istilah yang berkaitan dengan gelombang:


The definition of terms related to waves:

SP 5.1.4 Menerangkan ciri-ciri gelombang iaitu amplitud (A), tempoh (T), frekuensi (f), panjang gelombang (λ) dan laju gelombang (v)

Sesaran maksimum dari kedudukan


keseimbangan. a kedudukan
Amplitud, a keseimbangan
Amplitude, a
Maximum displacement from the equilibrium the equilibrium
position. a position
(i) Unit S.I. : / S.I. unit : m

• Masa yang diambil untuk membuat satu ayunan lengkap.


Tempoh ayunan, T
Time taken to make one complete oscillation.
Period, T
• Unit S.I. : / S.I. unit : s

• Bilangan ayunan lengkap dalam masa satu saat.


Number of complete oscillation in 1 second.
Frekuensi, f • Unit S.I. / S.I. unit : Hz (s–1) 1
Frequency, f • Formula yang menghubungkaitkan T dan f, T = f
1
Formula which relates T and f, T = f

• Bagi gelombang melintang; jarak di Puncak


Crest
antara dua puncak atau dua lembangan Lembangan
berturutan dalam suatu gelombang. λ Trough
For transverse wave, distance between two
consecutive crest or two consecutive troughs
of waves. λ
Gelombang melintang
• Bagi gelombang membujur; panjang Transverse wave
Panjang gelombang, λ
gelombang ialah jarak di antara titik
Wavelength, λ Renggangan / Rarefaction
tengah dua renggangan yang berturutan λ
atau jarak di antara dua titik tengah dua
mampatan yang berturutan.
For longitudinal waves; the wavelength λ Mampatan
is distance between the centre of two Compression
consecutive rarefaction or distance between Gelombang membujur
two consecutive compression. Longitudinal waves

• Jarak perambatan gelombang dalam satu saat.


The distance propagated by a wave in one second.
Halaju gelombang, v
• Halaju gelombang bergantung pada medium di mana gelombang merambat
Wavespeed, v
menerusinya.
U The wavespeed depends on the medium the waves are travelling through.
N
I
T
Puncak Titik pada gelombang dengan sesaran positif yang maksimum.
5 Crest A point on a wave with the maximum positive displacement.

Lembangan Titik pada gelombang dengan sesaran negatif yang maksimum.


Trough maximum negative
A point on a wave with the displacement.

© Nilam Publication Sdn. Bhd. 118


MODUL • Fizik TINGKATAN 4

Daripada rumus laju: / From the equation of speed:


jarak yang dilalui / distance travelled by a wave profile
=
masa yang diambil / time taken
l
Laju gelombung / Speed of wave, v =
T
=
l
T
l( )
l
Frekuensi gelombang / Frequency of wave, f =
T
Maka / Thus, v = f λ

SP 5.1.5 Melakar dan mentafsir graf gelombang: i) sesaran melawan masa ii) sesaran melawan jarak

Definisi ayunan Gerakan berkala yang berulang-alik pada kedudukan keseimbangan.


Definition of oscillation A periodic motion performs a repeated to-and-fro motion about an equilibrium position.

Kedudukan
Kedudukan objek di mana tiada daya paduan yang bertindak ke atasnya.
keseimbangan
The position of the object where there is no resultant force acting on it.
Equilibrium position

Kedudukan
keseimbangan
Equilibrium Satu ayunan lengkap
position One oscillation
Satu ayunan lengkap
One complete Satu ayunan lengkap
oscillation One oscillation

Pergerakan dari satu kedudukan tertentu ke kedudukan lain dan balik ke


kedudukan yang asal . / The movement from a certain position to the other
position and back to the original position .

Graf sesaran-masa (s – t) Graf sesaran-jarak (s – d)


Displacement-time graph (s – t) Displacement-distance graph (s – d)

Sesaran, s (cm) Sesaran, s (cm)


Displacement, s (cm) Displacement, s (cm)

λ λ

a a
O Masa, t (s) O Jarak, d (cm) U
U
Graf Time, t (s) Distance, d (cm) N
N
a a II
Graph T
T
15
λ λ
O = Kedudukan keseimbangan
The equillibrium position

a = Amplitud / Amplitude a = Amplitud / Amplitude


T= Tempoh / Period λ = Panjang gelombang
1 Wavelength
f =
T

119 © Nilam Publication Sdn. Bhd.


MODUL • Fizik TINGKATAN 4

SP 5 .1.6 Menentukan panjang gelombang, λ, frekuensi, f dan laju gelombang, v

Eksperimen Panjang gelombang air


Experiment Wavelength of water waves

Tujuan Menentukan panjang gelombang, frekuensi dan laju gelombang.


Aim To determine the wavelength, frequency and wave speed.

Tangki riak, stroboskop mekanikal, pembaris meter, motor penggetar, kertas putih,
Radas dan bahan bekalan kuasa, lampu dan bar kayu.
Apparatus and
materials Ripple tank, mechanical stroboscope, metre rule, vibrator motor, white paper, power supply, lamp
and wooden block.

Lampu
Motor Lamp
Motor
Air
Water Stroboskop
mekanikal
Mechanical
stroboscope
Bar getaran
Vibrating bar

Satah gelombang air


Kertas putih (skrin) Plane water waves
White paper (screen)

Prosedur 1 Tangki riak disediakan seperti dalam rajah dan sehelai kertas putih diletakkan di
Procedure
bawah tangki riak.
A ripple tank is set up as shown and a sheet of white paper is placed under the ripple tank.
2 Hidupkan suis motor penggetar pada frekuensi 10 Hz.
Switch on the vibrator motor at a frequency of 10 Hz.
3 Perhatikan gelombang air dengan menggunakan stroboskop, ukur panjang gelombang
dan rekodkan.
Observe the water waves by using the stroboscope, measure the wavelength and record it.
4 Ulangi eksperimen sekurang-kurangnya empat kali pada frekuensi yang berbeza:
Repeat the experiment at least four times at different frequencies of vibrator motor.

U
N f, Hz λ / cm
I
T 10
Penjadualan data
5 Tabulation of the
20
data 30
40
50

1 Graf λ melawan f diplot. / Graph of λ against f is plotted.


Analisis data
2 Daripada graf, nilai laju gelombang, v dapat ditentukan.
Analysis of data
From the graph, the value of wave speed, v can be determined.

© Nilam Publication Sdn. Bhd. 120


MODUL • Fizik TINGKATAN 4

Latihan / Exercises
1 Seorang pelajar menggetarkan suatu sistem getaran 4 Rajah di bawah menunjukkan gelombang bunyi
spring sebanyak 4 kali per saat. Panjang gelombang yang dihasilkan daripada satu tala bunyi.
bagi spring tersebut ialah 0.6 m. Berapakah laju The diagram below shows the sound waves produced
gelombang menerusi spring tersebut? / A student by a tuning fork.
makes a spring vibrating system vibrate at 4 times per 4 cm
second. The wavelength of the wave on the spring is
0.6 m. What is the wave speed moving along the spring?
Penyelesaian / Solution:
Diberi / Given
150 cm
f = 4 Hz, λ = 0.6 m
v = f λ
= 4 Hz × 0.6 m (a) Berdasarkan rajah, tentukan
= 2.4 m s–1 Based on the diagram, determine
(i) amplitud / the amplitude
2 Gelombang radio merambat dengan halaju (ii) panjang gelombang / the wavelength
3 × 108 m s–1. Berapakah panjang gelombang bagi (b) Berapakah frekuensi gelombang bunyi
gelombang radio FM yang diterima pada 200 MHz sekiranya laju gelombang adalah 330 m s–1?
bagi mendapatkan siaran radio tersebut? What is the frequency of the sound waves if the
Radio waves travel at a speed of 3 × 108 m s–1. What is speed wave is 330 m s–1?
the wavelength of FM radio waves received at 200 MHz Penyelesaian / Solution:
on your radio dial?
Penyelesaian / Solution: 4 cm
(a) (i) a = 2
(b) v = f λ
Diberi / Given 330 m s–1 = f × 0.5 m
= 2 cm
v = 3 × 108 m s–1, f = 200 MHz f = 660 Hz
v = fλ
(ii) 3λ = 150 cm
(3 × 108 m s–1) = (200 × 106 Hz) × λ
λ = 50 cm
λ = 1.5 m
3 Seutas tali direntangkan di atas lantai. Satu 5 Rajah di bawah menunjukkan graf sesaran-masa.
penghujung tali tersebut digoyangkan dengan The diagram below shows the displacement-time graph.
frekuensi 8 Hz. Graf di bawah menunjukkan Sesaran / m
Displacement / m
sebahagian pergerakan tali dalam suatu masa.
A long rope is stretched out on the floor. One end of 10
the rope is then shaken at frequency of 8 Hz. The graph
below shows the rope at a particular moment in time. 0 Masa / s
0.2 0.4 0.6 0.8 Time / s
Sesaran / m / Displacement / m –10
0.5
0
0.4 0.8 1.2 1.6 Jarak / m Berdasarkan graf di atas, tentukan
Distance / m
–0.5 Based on the graph above, determine the
(a) amplitud / amplitude
Tentukan / Determine: (b) tempoh / period U
U
N
N
(a) amplitud / the amplitude (c) frekuensi / frequency II
T
(b) panjang gelombang / the wavelength Penyelesaian / Solution: T

(c) laju gelombang / the wave speed (a) a = 10 m 1 15


(c) f =
Penyelesaian / Solution: (b) T = 0.4 s T
1
(a) a = 0.5 m (c) v = f λ = 0.4 s
(b) λ = 0.8 m = 8 Hz × 0.8 m = 2.5 Hz
= 6.4 m s–1

121 © Nilam Publication Sdn. Bhd.


MODUL • Fizik TINGKATAN 4

5.2 PELEMBAPAN DAN RESONANS / DAMPING AND RESONANCE SK 5.2

5.2.1 Memerihalkan pelembapan dan resonans bagi satu sistem ayunan/getaran


SP
5.2.2 Mewajarkan kesan resonans terhadap kehidupan

Pelembapan dalam sistem ayunan / Damping in an oscillating system

Sistem ayunan / Oscillating system


• Sistem ayunan itu tidak berayun berterusan dengan amplitud yang sama kecuali apabila sistem itu berayun
dalam vakum. / The oscillating system does not continue with the same amplitude indefinitely except when the
system is oscillating in a vacuum.

Mengalami / Experiences

Pelembapan / Damping Resonans / Resonance


• Pelembapan ialah pengurangan amplitud bagi satu • Resonans berlaku apabila suatu sistem
sistem ayunan di mana tenaga dilesapkan dalam bentuk dipaksa bergetar pada frekuensi yang sama
tenaga haba. / Damping is the decrease in amplitude of an
dengan frekuensi aslinya yang disebabkan
oscillating system when energy is drained out as heat energy.
• Amplitud akan semakin berkurang dan menjadi sifar oleh daya luar.
Resonance occurs when a system is made to
apabila ayunan berhenti. / The amplitude will gradually
oscillate at a frequency equivalent to its natural
decrease and become zero when the oscillation stops.
frequency by an external force.
• Sistem yang mengalami resonans ini berayun
Graf pelembapan / A graph to show damping pada amplitud yang maksimum .
The resonating system oscillate at its
Amplitud / Amplitude Sesaran / Displacement maximum
d (cm) amplitude.
a (cm)

Beberapa kesan Some effects of


daripada resonans resonance
t (s) 0 t (s)
0 Masa / Time Masa / Time
• Penala di dalam radio dan televisyen
diputarkan untuk memilih stesen program.
Dua jenis pelembapan Two types of damping Penala dalam litar diubah sehingga resonans
tercapai, pada frekuensi yang dihantar
Pelembapan luaran Pelembapan dalaman daripada stesen tertentu yang dipilih. Maka,
External damping Internal damping
isyarat elektrik yang kuat terhasil.
The tuner in a radio or television enables us to
Berlaku disebabkan Berlaku kerana kehilangan select the programmes. The circuit in the tuner
kehilangan tenaga untuk tenaga kerana mampatan is adjusted until resonance is achieved at the
mengatasi daya geseran dan regangan zarah-zarah di frequency transmitted by a particular station
atau rintangan udara. dalam sistem. / Caused due selected. Hence, a strong electrical signal is
Caused due to loss of energy to loss of energy due to the produced.
to overcome frictional force or stretching and compression of • Kesan resonans juga membawa kemusnahan.
U air resistance. particles in the system. Contohnya, jambatan runtuh apabila
N amplitud getaran bertambah yang disebabkan
I
T
oleh resonans, seperti apa yang telah berlaku
Menyebabkan / Cause di Tacoma Narrows Bridge di USA pada
5 1949. Fenomena berlaku oleh tindakan
tenaga angin yang menyebabkan jambatan bergetar
• Amplitud dan berkurang dengan amplitud yang besar.
energy The effects of resonance can also cause damage.
Amplitude and decreases
For example, a bridge can collapse when the
• Frekuensi malar amplitude of its vibration increases as a result of
constant resonance, such as the Tacoma Narrows Bridge
• Frequency is in USA in 1949. The action of the wind caused
the bridge to vibrate at a large amplitude.

© Nilam Publication Sdn. Bhd. 122


MODUL • Fizik TINGKATAN 4

Istilah Definisi
Term Definition

Daya luar Daya yang dikenakan kepada sistem ayunan untuk membolehkan sistem berayun secara
External force berterusan. / Force that applied to an oscillation system to enable the system to oscillate continuously.

Ayunan paksa
Sistem ayunan yang dikenakan daya luar berkala.
Forced
An oscillating system that oscillates because of the periodic external force that applied on it.
oscillation

Frekuensi asli Frekuensi sesuatu sistem yang bergetar sendiri tanpa sebarang daya luar yang bertindak ke
Natural atasnya.
frequency The frequency of a system which oscillates freely without the action of an external force.

Maklumat tambahan:
Additional information:

Teknik Menjawab [Format Kertas 2 : Kefahaman / Esei Pendek]


Answering Technique [Paper 2 Format : Comprehension / Short Essay]

Soalan / Question: Jawapan / Answer:


Bandul B diayunkan. Dengan menggunakan
konsep fizik yang sesuai, terangkan apa akan • Bandul B berayun dan memindahkan tenaga kepada
berlaku kepada ayunan bandul-bandul yang bandul-bandul lain. / Pendulum B oscillates and transfers
lain selepas beberapa ketika. / Pendulum B is
energy to others pendulum.
oscillated. Using appropriate physics concept,
explain what will happen to the state of oscillation • Panjang bandul B = Panjang bandul D
of other pendulums after a while.
Length of pendulum B = Length of pendulum D
Kaki retort [4 markah / marks]
Retort stand • Frekuensi bandul B = Frekuensi bandul D
Benang
Thread Frequency of pendulum B = Frequency of pendulum D
energy to others pendulum.
A F
B D • Bandul D mengalami resonans
E
C Pendulum D undergoes resonance
• Bandul D berayun dengan amplitud maksimum U
U
Pemberat N
N
Loads Pendulum D oscillates with the maximum amplitude II
T
T
Nota / Note: Perhatian / Attention: 15
(i) Frekuensi ayunan bandul bergantung kepada Satu poin = satu markah
panjang bandul. / Frequency of pendulum One point = one mark
depends on length of the pendulum. Boleh diolah lebih daripada 4 poin
(ii) Dua bandul dengan panjang yang sama Can present more than 4 points
mempunyai frekuensi dan tempoh yang Ketertiban poin tidak penting
sama. / Two pendulum of the same length have The sequence of the points is not important
the same frequency and period. Poin perlu ringkas, padat, tepat dan bermakna
The points should be simple, compact, accurate and meaningful

123 © Nilam Publication Sdn. Bhd.


MODUL • Fizik TINGKATAN 4

5.3 PANTULAN GELOMBANG / REFLECTION OF WAVES SK 5.3

Menghuraikan pantulan gelombang dari aspek sudut tuju (i), sudut pantulan (r), panjang gelombang (λ), frekuensi (f), laju, (v) dan arah
5.3.1
SP perambatan gelombang
5.3.2 Melukis gambar rajah untuk menunjukkan pantulan gelombang air satah bagi pemantul satah

Garisan atau permukaan yang menyambungkan titik-titik pada fasa yang


sama.
The locus of points which vibrates in phase .
Definisi muka
Muka gelombang
gelombang Wavefront
Definition of C Titik dalam fasa yang sama
C Points in phase
wavefront
C = Puncak / Crest
T = Lembangan / Trough
T

Muka gelombang membulat Muka gelombang satah


Circular wavefront Plane wavefront

λ Arah perambatan gelombang


λ The direction of wave
Arah perambatan
Jenis muka gelombang propagation
gelombang The direction of
Types of wavefront wave propagation
Muka gelombang
Wavefront
Muka gelombang / Wavefront

Arah perambatan gelombang adalah berserenjang dengan muka gelombang.


The direction of the propagation of a wave is perpendicular to its wavefront.

Nyatakan • Berlaku apabila gelombang tuju terkena pemantul dan mengubah arah perambatannya.
definisi pantulan Occurs when an incident wave strikes a reflector and undergoes a change in direction of
gelombang. propagation.
State the definition • Berlaku kepada gelombang air, gelombang bunyi, gelombang cahaya dan gelombang
of reflection of elektromagnet.
waves. Occurs to water waves, sound waves, light waves, and electromagnetic waves.

•i=r
Nyatakan
• Gelombang tuju, gelombang pantulan dan garis normal terletak pada satah yang sama
hukum pantulan
State the Laws of
pada sudut tegak dengan permukaan pantulan pada titik tuju.
U The incident wave, reflected wave and normal lie in the same plane which is perpendicular to the
Reflection
N reflecting surface at the point of incidence.
I
T

5 Apakah ciri-
ciri pantulan
•i=r
• Panjang gelombang , frekuensi dan laju gelombang tidak berubah selepas pantulan.
gelombang? Wavelength, frequency and wave speed do not change after reflection.
What is the • Arah perambatan gelombang berubah apabila ia dipantulkan.
characteristics of Direction of propagation of wave changes when it is reflected.
laws of reflection?

© Nilam Publication Sdn. Bhd. 124


MODUL • Fizik TINGKATAN 4

Rajah yang menunjukkan pantulan gelombang / Diagram that shows reflection of waves

Gelombang tuju Gelombang tuju


Incident wave Incident wave
λ
λ

Pemantul cekung
λ

Concave reflector
Pemantul satah
Plane reflector
i r

Gelombang tuju Gelombang


Incident wave pantulan
Reflected
Pemantul satah wave λ λ λ
Plane reflector Gelombang pantulan Gelombang pantulan
Normal
Normal Reflected wave Reflected wave

i = sudut tuju (sudut di antara gelombang tuju dan normal)


angle of incidence (the angle between the direction of propagation of the incident wave and the normal)
r = sudut pantulan (sudut di antara gelombang pantulan dan normal)
angle of reflection (the angle between the direction of propagation of the reflected wave and the normal)
Gelombang tuju / Incident wave
Gelombang pantulam / Reflected wave

PANTULAN GELOMBANG (Mencirikan) / Reflection of waves (Characterisation)

Panjang
Mempunyai
gelombang tetap
Constant sudut tuju yang sama
wavelength dengan sudut pantulan
Has an angle of
(λ)
incidence that equal to
the angle of reflection

Gelombang tuju,
gelombang pantulan dan Ciri-ciri
garis normal berada pada Pantulan
satah yang sama Gelombang
Incident wave, reflected Characteristics of
wave and the normal lie in Reflection Frekuensi tetap
the same plane of Wave Constant
frequency
(f) U
U
N
N
II
T
T
Halaju tetap 15
Arah perambatan Constant
gelombang berubah velocity
Direction of wave (v)
propagation changed

125 © Nilam Publication Sdn. Bhd.


MODUL • Fizik TINGKATAN 4

5.3.3 Mewajarkan aplikasi pantulan gelombang dalam kehidupan harian


SP
5.3.4 Menyelesaikan masalah melibatkan pantulan gelombang

Aplikasi Pantulan Gelombang dalam Kehidupan Harian


Application of Reflection of Waves in Daily Lives
Transduser
1 Gelombang ultrasonik digunakan dalam bidang perubatan untuk memeriksa Transducer
struktur organ dalaman dalam badan manusia.
Ultrasonic waves is used in medical field to scan structures of internal organs in
human body. Rahim ibu
Mother's womb
2 Gelombang radio digunakan dalam bidang telekomunikasi dengan
menggunakan antena parabola untuk memantulkan gelombang yang diterima
oleh satelit komunikasi.
Radio waves is used in telecommunication field by using parabola antenna to reflect
waves received by the communication satellites.

3 Kapal selam dapat mengesan sesuatu kedalaman, halangan atau struktur Kapal selam
batuan di dalam laut dengan memantulkan gelombang yang terkena pada Submarine
sesuatu permukaan di dalam laut.
A submarine can detect the depth, obstruction or structure of rocks in the sea by Halangan
Obstruction
reflecting waves that are hit to a surface in the sea.
Kapal / Ship
4 Teknologi sonar digunakan oleh kebanyakan kapal bagi mengesan kawasan
yang mempunyai banyak ikan. Tranduser memancarkan gelombang ke dalam
air dan akan dipantulkan oleh ikan kembali ke tranduser. Pemancar
Penerima
Sonar technology is used by most ships to detect areas with large numbers of fish. The Receiver Emitter
transducer transmits the wave into the water and is reflected by the fish back to the Air laut / Sea water
transducer.
Dasar laut / Seabed

Latihan / Exercises
1 Rajah di bawah menunjukkan seorang pelajar tembakan selepas 8 saat. Jika halaju bunyi ialah
berdiri di antara dua dinding. Pelajar tersebut 320 ms–1, berapakah jarak pemburu itu dari tebing
menepuk tangan sekali yang menghasilkan gema. bukit? / A hunter was standing at a distance from a
Halaju bunyi dalam udara ialah 320 ms–1. high cliff. He fired one shot and then heard an echo
The diagram below shows a student was standing of the shot after 8 seconds. If velocity of the sound is
between two walls. The student clap his hands once which 320 ms–1, what is the distance of the hunter to the cliff?
produces echoes. Velocity of the sound in air is 320 ms–1. A 40 m C 760 m
B 80 m D 1280 m

3 Gelombang ultrasonik dihantar ke dasar sebuah


U lautan yang dalamnya 900 m. Berapakah selang
N masa antara penghantaran suatu denyutan ultrasonik
I
T dengan pengesanannya selepas dipantul dari dasar
Berapakah tempoh antara gema pertama dan gema
5 kedua? / What is the time interval between the first echo
laut jika halaju gelombang bunyi ialah 375 ms–1?
An ultrasound wave is sent to the bottom of an
and the second echo? ocean within 900 m. What is the interval between the
A 1 s C 3 s transmission of an ultrasound pulse and its detection
B 2 s D 4 s after being reflected off the sea if the velocity of the
sound wave is 375 ms–1?
2 Seorang pemburu berdiri pada suatu jarak dari A 0.6 s C 2.4 s
sebuah tebing bukit yang tinggi. Dia melepaskan B 1.2 s D 4.8 s
satu das tembakan dan kemudian terdengar gema

© Nilam Publication Sdn. Bhd. 126


MODUL • Fizik TINGKATAN 4

5.4 PEMBIASAN GELOMBANG / REFRACTION OF WAVES SK 5.4

Menghuraikan pembiasan gelombang dari aspek sudut tuju (i), sudut biasan (r), panjang gelombang, (λ), frekuensi (f), laju (v) dan arah
5.4.1
SP perambatan gelombang
5.4.2 Melukis gambar rajah untuk menunjukkan pembiasan gelombang bagi dua kedalaman yang berbeza

Pembiasan gelombang / Refraction of waves

Definisi / Definition Ciri-ciri gelombang yang terbias


Characteristics of wave after refraction
Pembiasan gelombang ialah perubahan arah
perambatan gelombang disebabkan oleh perubahan Gelombang mempunyai frekuensi yang sama tetapi
halaju apabila gelombang merambat dari satu berbeza pada laju gelombang, panjang gelombang
medium ke medium lain, yang mempunyai dan arah perambatan.
perbezaan ketumpatan optik atau kedalaman. The wave has the same frequency but different wave
Refraction of waves is the change in direction of speed, wavelength and direction of propagation.
propagation of the wave when its velocity changes as the
waves propagate from one medium to another, which
has different optical densities or different depths.

Berlaku kepada / Occur to


Gelombang air, gelombang bunyi, gelombang cahaya dan gelombang elektromagnet
Water waves, sound waves, light waves and electromagnetic waves

Ciri-ciri gelombang / Characteristics of waves


Keadaan Kawasan dalam Kawasan cetek
Situation In deep water In shallow water

Halaju Lebih laju Lebih perlahan


Speed Faster Slower

Panjang gelombang Lebih panjang Lebih pendek


Wavelength Longer Shorter

Frekuensi Tidak berubah Tidak berubah


Frequency Unchanged Unchanged
U
U
N
N
(a) Lukiskan gelombang air yang terbias. / Draw the refracted water waves. II
T
T
Dalam
Deep
Cetek
Shallow
Dalam
Deep
Dalam
Deep
Cetek
Shallow
15
Dalam
Deep

127 © Nilam Publication Sdn. Bhd.


MODUL • Fizik TINGKATAN 4

(b) Lukis dan labelkan gelombang cahaya yang terbias. / Draw and label the refracted light waves.
Sinar tuju
Incident ray i Sinar tuju
Incident ray
Kaca i
Sinar terbias Glass
Refracted ray
r Udara Sinar terbias
Udara Kaca Air r
Air Glass Refracted ray

Apabila sinar tuju merambat dari medium kurang Apabila sinar tuju merambat dari medium yang lebih
tumpat ke medium yang lebih tumpat, ia akan dibiaskan tumpat ke medium yang kurang tumpat, ia akan dibiaskan
mendekati garis normal. menjauhi garis normal.
When the light ray travels from an optically less dense When the light travels from an optically denser medium to an
medium to an optically denser medium, it bends towards the optically less dense medium, it bends away from the normal.
normal.

Kemahiran mengkonsepsi secara kuantitatif / Conceptualisation skills quantitatively


Pembiasan gelombang air / Refraction of water waves
v1 v2
λ1 λ2 (a) Gelombang air mengalami pembiasan apabila merambat
melalui dua kawasan yang berbeza kedalaman.
d2 Water waves undergoes refraction when propagates through
d1 two areas of different depth.
Blok kaca
Air / Water (b) Frekuensi gelombang adalah tetap.
Glass block
Frequency of waves is constant.

(c) v = fλ
(d) Laju gelombang, v, bergantung pada panjang gelombang, λ.
Speed of waves, v, depends on wavelength, λ.
(e) Bandingkan / Compare:
• d1 lebih besar daripada / is larger than d2
•λ lebih besar daripada / is larger than λ2
1

• v1 lebih besar daripada / is larger than v2

(f) Hubung kaitkan antara kedalaman air, panjang gelombang air dan laju gelombang air:
Relate the depth of water, wavelength and speed of water waves:
Semakin bertambah kedalaman air, semakin bertambah panjang gelombang dan laju gelombang.
As the depth of water increases, the wavelength and speed of wave increases .

(g) Formula yang menghubungkaitkan v dan λ untuk kawasan air dalam dan kawasan air cetek:
U Formula that relates v and λ for area of deep water and area of shallow water:
N
I Frekuensi di / Frequency on :
T v
Kawasan dalam / Deep area : f1 = 1 Maklumat tambahan:
5 l1
v
Additional information:
Kawasan cetek / Shallow area : f2 = 2
l2
f1 = f2
v1 v2
=
l1 l2

© Nilam Publication Sdn. Bhd. 128


MODUL • Fizik TINGKATAN 4

Teknik Menjawab [Format Kertas 2 : Kefahaman / Esei Pendek]


Answering Technique [Paper 2 Format : Comprehension / Short Essay]

Pembiasan gelombang bunyi pada waktu malam yang sejuk / Refraction of sound wave on cold night time

Soalan / Question: Perhatian / Attention:


Pada waktu malam yang sejuk, bunyi kereta api yang keluar Idea tentang suhu lapisan-lapisan udara
Idea about the temperature of air layers
dari terowong kedengaran lebih jelas. Terangkan. Idea tentang ketumpatan lapisan-lapisan udara
On a cold night, the sound of the train coming out of the tunnel Idea about the density of air layers
sounds more clearly. Explain. Idea tentang arah perambatan dan fenomena
[4 markah / marks] gelombang
Idea about the direction and phenomenon of waves
Idea tentang perubahan panjang gelombang / laju
gelombang
Idea about the change of wavelength / speed of waves
Idea tentang fenomena yang menyebabkan
Terowong perubahan arah gelombang yang menghala ke
Tunnel permukaan darat (pantulan dalam penuh)
Idea about the phenomenon that causes the change of
direction of waves toward the ground surface (total
internal reflection)

Kereta api / Train

Jawapan / Answer:

• Lapisan udara di lapisan bawah lebih sejuk berbanding lapisan atas.


Air layer near the ground is colder than the top layer.
• Ketumpatan lapisan udara sejuk lebih tinggi daripada lapisan udara panas.
The density of colder air layer is greater than hotter air layer.
• Gelombang bunyi terbias menjauhi normal.
Sound waves refracted away from normal.
• Panjang gelombang di lapisan udara sejuk lebih kecil daripada lapisan udara panas.
Wavelength of colder air layer is smaller than hotter air layer.
• Laju gelombang di lapisan udara sejuk lebih kecil daripada lapisan udara panas.
Wave speed of colder air layer is smaller than hotter air layer.
• Gelombang bunyi terbias dan mendekati permukaan darat disebabkan pantulan dalam penuh.
Sound waves is refracted and move towards the ground surface due to total internal reflection.
U
U
N
N
II
T
T
15

129 © Nilam Publication Sdn. Bhd.


MODUL • Fizik TINGKATAN 4

Eksperimen Pembiasan gelombang air


Experiment Refraction of water waves

Inferens / Inference
Panjang gelombang air bergantung pada kedalaman air. / The wavelength of water waves depends on the depth of water.

Hipotesis / Hypothesis
Apabila kedalaman air meningkat, maka panjang gelombang air meningkat.
When the depth of water increases, the wavelength of the water waves increases.

Tujuan / Aim
Mengkaji hubungan antara kedalaman air dan panjang gelombang air.
To investigate the relationship between the depth of water and the wavelength of water waves.

Pemboleh ubah / Variables


1 Pemboleh ubah dimanipulasikan: / Manipulated variable:
Kedalaman air / Depth of water

2 Pemboleh ubah bergerak balas: / Responding variable:


Panjang gelombang / Wavelength

3 Pemboleh ubah yang dimalarkan: / Constant variable:


Frekuensi / Frequency

Senarai radas dan bahan / List of apparatus and materials


Tangki riak, lampu, motor, bar kayu, bekalan kuasa, kertas putih, protraktor, satah pantulan, plat perspek,
pembaris 15 cm, stroboskop mekanikal, pembaris meter
Ripple tank, lamp, motor, wooden bar, power supply, white paper, protractor, plane reflector, perspex plate, 15 cm ruler,
mechanical stroboscope, metre rule

Susunan radas / Arrangement of the apparatus


Lampu
Lamp
Motor
Motor Air
Water Plat perspeks
Perspex plate

Stroboskop
U
N mekanikal
I Mechanical
T stroboscope
5 Bar getaran
Vibrating bar

Kertas putih (skrin)


White paper (screen)

© Nilam Publication Sdn. Bhd. 130


MODUL • Fizik TINGKATAN 4

Prosedur / Procedure
1 Kedalaman air, d, di atas permukaan perspek, diukur dengan menggunakan pembaris 15 cm.
The depth of water, d, over a perspex plate, is measured by using a 15 cm ruler.
2 Bekalan kuasa dihidupkan untuk menggetarkan motor supaya gelombang satah merambat merentasi plat
perspek. Gelombang itu dibekukan dengan menggunakan stroboskop mekanikal. Gelombang itu dilakar pada
skrin. / The power supply is switched on to produce plane waves which propagate across the perspex plate. The waves
are frozen by a mechanical stroboscope. The waves are sketched on the screen.
3 Jarak antara 11 jalur terang berturutan, x, diukur dengan menggunakan pembaris meter dan direkodkan.
The distance between 11 successive bright bands, x, is measured by the metre ruler and recorded.
x
4 Panjang gelombang, λ, dikira / The wavelength λ, is calculated, λ = cm
10
5 Eksperimen diulang 4 kali dengan menambahkan bilangan plat perspek untuk mengubah kedalaman air di
atas plat perspek. / The experiment is repeated 4 times by increasing the number of perspex plates to change the depth
of the water across the plate.

Menjadualkan data / Tabulation of the data


Kedalaman air / Depth of water, d / cm
Panjang gelombang / Wavelength, λ / cm

Analisis data / Analysis of data

λ / cm

0 d / cm

Latihan / Exercise
1 Rajah di bawah menunjukkan gelombang satah air bergerak dari kawasan P ke kawasan Q yang mempunyai
kedalaman berbeza. Halaju gelombang air di kawasan P ialah 18 cm s–1. Berapakah halaju gelombang air di
kawasan Q? / The diagram below shows a plane water wave moving from area P to area Q of different depths. The
speed of the water wave in area P is 18 cm s–1. What is the speed of the water wave in area Q?
Penyelesaian / Solution: P Q
Kedalaman P: / Deep region P:
4λp= 12 cm λp = 3 cm 18 cm s–1 = f (3 cm)
12 vp = f λp \ f = 6 Hz
λp = cm 12 cm 12 cm
U
4 U
N
N
II
P Q T
T
Frekuensi gelombang di kawasan Q adalah bersamaan dengan frekuensi
gelombang di kawasan P. / Frequency of wave in region Q is the same as the
15
frequency of wave in region P.
8λQ = 12 cm Oleh itu, / Hence, vQ = f λQ
4λ = 12 cm 8λ = 12 cm
12
λQ = cm vQ = (6 Hz) × 1.5 cm
8
= 9.0 cm s–1
= 1.5 cm

131 © Nilam Publication Sdn. Bhd.


MODUL • Fizik TINGKATAN 4

5.5 PEMBELAUAN GELOMBANG / DIFFRACTION OF WAVES SK 5.5

5.5.1 Menghuraikan pembelauan gelombang dari aspek panjang gelombang (λ), frekuensi (f), laju (v), amplitud (A) dan arah perambatan gelombang
SP 5.5.2 Menentukan faktor-faktor yang mempengaruhi pembelauan gelombang
5.5.3 Melukis gambar rajah untuk menunjukkan corak pembelauan gelombang air dan kesan pembelauan cahaya

Definisi Penyebaran gelombang apabila gelombang merambat melalui sesuatu celahan atau
pembelauan halangan.
gelombang The spreading of waves when the waves propagate through a slit or side of a barrier.
Definition of
diffraction of
waves
Kesan
pembelauan Dimalarkan / Constant Berubah / Change
ke atas ciri-ciri
gelombang Panjang gelombang, λ / Wavelength, λ Amplitud berkurang / Amplitude decreases
Effects of the Frekuensi, f / Frequency, f Arah perambatan / Direction of propagation
diffraction on
charactenistics of Laju, v / Speed, v Tenaga berkurang / Energy decreases
waves
Corak pembelauan bagi faktor-faktor berikut: / Pattern of diffraction for the following factors:
(a) Saiz celah sempit, panjang gelombang tetap (b) Saiz celah lebar, panjang gelombang tetap
Narrow slit size, fixed wavelength Wide slit size, fixed wavelength

(c) Panjang gelombang panjang, saiz celah tetap (d) Panjang gelombang pendek, saiz celah tetap
Long wavelength, fixed slit size Short wavelength, fixed slit size

Video

Pembelauan Gelombang
Faktor-faktor yang mempengaruhi pembelauan gelombang dan kesannya: Diffraction of Waves

Factors which affecting diffraction of waves and their effects:

Perubahan Faktor yang tidak


Faktor Kesan belauan
U ke atas faktor berubah / Factor
N Factor Effects of diffraction
Changes on factors that is fixed
I
T
Sempit / Narrow Lebih ketara / More significant
5 Saiz celah
Panjang
gelombang
Size of slit Kurang ketara / Less significant
Lebar / Wide Wavelength

Panjang Pendek / Short Kurang ketara / Less significant


Saiz celah
gelombang Size of slit Lebih ketara / More significant
Wavelength Panjang / Long

© Nilam Publication Sdn. Bhd. 132


MODUL • Fizik TINGKATAN 4

Eksperimen Pembelauan gelombang air


Experiment Diffraction of water waves

Inferens / Inference

Sudut bengkokan gelombang air bergantung kepada saiz celah.


The angle of bending of the waves depends on the size of the slit.

Hipotesis / Hypothesis

Sudut bengkokan gelombang air, θ meningkat apabila saiz celah, a berkurang.


The angle of bending of the waves, θ increases as the size of slit, a decreases.

Tujuan / Aim
Mengkaji hubungan antara sudut bengkokan gelombang air dan saiz celah.
To investigate the relationship between the angle of bending of the waves and the size of the slit.

Pemboleh ubah / Variables


1 Pemboleh ubah dimanipulasikan: / Manipulated variable:
Saiz celah, a, / Size of slit, a

2 Pemboleh ubah bergerak balas: / Responding variable:


Sudut bengkokan gelombang air, θ / Angle of bending of the water waves, θ

3 Pemboleh ubah yang dimalarkan: / Constant variable:


Frekuensi penggetar / Frequency of vibrator

Senarai radas dan bahan / List of apparatus and materials


Tangki riak, lampu, motor, bar kayu, bekalan kuasa, kertas putih, dua batang bar besi, pembaris meter, protraktor
dan stroboskop mekanikal
Ripple tank, lamp, motor, wooden bar, power supply, white paper, two pieces metal bar, metre rule, protractor and
mechanical stroboscope

Susunan radas / Arrangement of the apparatus


Lampu
Lamp
Motor
Motor Air
Water Bar logam
Metal bar
Celah
Slit U
U
N
N
Stroboskop II
mekanikal T
T
Mechanical
stroboscope 15
Bar getaran
Vibrating bar

Kertas putih (skrin)


White paper (screen)

133 © Nilam Publication Sdn. Bhd.


MODUL • Fizik TINGKATAN 4

Prosedur / Procedure
1 Susunan radas disusun seperti rajah di sebelah.
The apparatus is set up as shown in the diagram.
2 Dengan menggunakan pembaris meter, lebar celah diukur, a = 0.5 cm.
By using a metre rule, the width of the slit is measured, a = 0.5 cm.
3 Bekalan kuasa dihidupkan. Gelombang dibekukan dengan menggunakan stroboskop mekanikal.
The power supply is switched on. The waves are freeze by a mechanical stroboscope.
4 Bentuk muka gelombang yang terhasil selepas melalui celah dilukis.
The shape of the wavefront after passing through the slit is drawn.
5 Dengan menggunakan protraktor, sudut bengkokan, θ diukur.
By using a protractor, the angle of bent, θ is measured.
6 Eksperimen diulang dengan menggunakan lebar celah, a = 1.0 cm, 1.5 cm, 2.0 cm dan 2.5 cm.
The experiment is repeated for width of slit, a = 1.0 cm, 1.5 cm, 2.0 cm and 2.5 cm.
Keputusan / Results

Saiz celah, a / cm
0.5 1.0 1.5 2.0 2.5
Size of slit, a / cm
Sudut bengkokan gelombang air, θ / °
Angle of bending of water waves, θ / °

Perbincangan / Discussions
1 Graf sudut bengkokan gelombang air, θ melawan saiz celah, a diplot dengan menggunakan kertas graf.
A graph of angle of bending of water waves, θ against the size of slit, a is plotted by using a graph paper.
2 Bandingkan ciri-ciri muka gelombang sebelum dan selepas melalui celah di dalam jadual di bawah.
Compare the characteristics of the wavefront before and after passing through the slit in the table below.

Ciri-ciri Perbandingan sebelum dan selepas melalui celah
Characteristics Comparison before after passing through the slit
Panjang gelombang
Wavelength
Frekuensi
Frequency
Laju gelombang
Wave speed
Amplitud
Amplitude
U
N Arah perambatan
I Direction of propagation
T

5
3 Adakah hiposis diterima? / Is the hypothesis accepted?
Ya / Yes

Kesimpulan / Conclusion
Sudut bengkokan gelombang air, θ meningkat apabila saiz celah, a berkurang.
The angle of bending of the waves, θ increases as the size of slit, a decreases.

© Nilam Publication Sdn. Bhd. 134


MODUL • Fizik TINGKATAN 4

Eksperimen Pembelauan gelombang cahaya


Experiment Diffraction of light waves

Inferens / Inference

Lebar pinggir cerah di tengah bergantung kepada saiz celah.


The width of the middle bright fringe depends on the size of the slit.

Hipotesis / Hypothesis

Lebar pinggir cerah di tengah bertambah apabila saiz celah berkurang.


The width of the middle bright fringe increases as the size of the slit decreases.

Tujuan / Aim
Mengkaji hubungan lebar pinggir cerah di tengah dengan saiz celah.
To investigate the relationship between the width of the middle bright fringe and the size of the slit.

Pemboleh ubah / Variables


1 Pemboleh ubah dimanipulasikan: / Manipulated variable:
Saiz celah
Size of slit

2 Pemboleh ubah bergerak balas: / Responding variable:


Lebar pinggir cerah di tengah
The width of the middle bright fringe.

3 Pemboleh ubah yang dimalarkan: / Constant variable:


Cahaya monokromatik (cahaya dengan satu panjang gelombang sahaja)
Monochromatic light (light of one wavelength only)

Senarai radas dan bahan / List of apparatus and materials


Sumber cahaya monokromatik, celah tunggal, pembaris meter
Monochromatic light source, single slit, metre rule

Susunan radas / Arrangement of the apparatus

Sumber cahaya laser Celah tunggal


Laser light source Single slit
U
U
N
N
II
T
T
15

Skrin
Screen Pinggir tengah cerah
x Middle bright fringe

135 © Nilam Publication Sdn. Bhd.


MODUL • Fizik TINGKATAN 4

Prosedur / Procedure
1 Susunan radas disusun seperti rajah di sebelah.
The apparatus is set up as shown in the diagram.
2 Lebar celah direkodkan, a = 0.2 mm. Cahaya dari sumber ditujukan ke arah celah.
The width of the slit is recorded, a = 0.2 mm. The light beam from the source is directed towards the slit.
3 Bekalan kuasa dihidupkan. Gelombang dibekukan dengan menggunakan stroboskop mekanikal.
The power supply is switched on. The waves are freeze by a mechanical stroboscope.
4 Corak yang terbentuk pada skrin dilukis.
The pattern formed on the screen is drawn.
5 Dengan menggunakan pembaris meter, lebar pinggir cahaya di tengah, x, diukur dan direkodkan.
By using a metre rule, the width of the middle bright fringe, x is measured and recorded.
6 Eksperimen diulangi dengan menggunakan lebar celah, a = 0.4 mm, 0.6 mm, 0.8 mm dan 1.0 mm.
The experiment is repeated for widths of slit, a = 0.4 mm, 0.6 mm, 0.8 mm and 1.0 mm.
Keputusan / Results

Lebar celah / Width of slit, a / mm 0.2 0.4 0.6 0.8 1.0


Lebar pinggir tengah cerah / Width of middle bright fringe, x / cm

Perbincangan / Discussions
1 Graf lebar pinggir tengah cerah, x melawan lebar celah, a diplot dengan menggunakan kertas graf.
A graph of width of middle bright fringe, x against width of slit, a is plotted by using a graph paper.
2 Daripada lukisan corak yang terbentuk pada skrin, nyatakan perbezaan imej yang terbentuk bagi saiz celah
yang lebar dan saiz celah yang sempit. / From the drawing of pattern formed on the screen, state the difference in
the image formed for wide slit and narrow slit.
3 Jika celah digantikan dengan lubang jarum yang bersaiz kecil dan lubang jarum yang bersaiz besar, lakarkan
corak yang terbentuk pada skrin tersebut.
If the slit is replaced with a small pin hole and a large pin hole, sketch the pattern formed in the following screen.

SP 5.5.4 Menjelaskan aplikasi pembelauan gelombang dalam kehidupan harian

Pembelauan Gelombang dalam Kehidupan Harian


Diffraction of Waves in Daily Life

1 Kawasan air yang tenang seperti di pantai atau pinggir laut disebabkan oleh benteng
pertahanan yang menghalang ombak besar daripada menghampiri kawasan daratan.
U A region of calm water like beach or seafront is caused by an embankment barrier that prevent
N big waves from approaching the mainland.
I
T

5 2 Kad bank seperti kad debit mengandungi tanda keselamatan yang diperbuat daripada
hologram, terhasil daripada kesan pembelauan cahaya.
Bank cards like debit card contain safety features that made by holograms produced from
diffraction of lights.

3 Haiwan seperti ikan paus menghasilkan gelombang infrasonik bagi menghantar isyarat
pada jarak yang jauh.
Animals like whales produce infrasonic waves to transmit signals at long distance.

© Nilam Publication Sdn. Bhd. 136


MODUL • Fizik TINGKATAN 4

5.6 INTERFERENS GELOMBANG / INTERFERENCE OF WAVES SK 5.6

5.6.1 Menghuraikan prinsip superposisi gelombang


SP 5.6.2 Melukis corak gelombang interferens bagi air, bunyi dan cahaya
5.6.3 Menghubung kait λ, a, x dan D berdasarkan corak intereferens gelombang

Definisi Kesan superposisi dua atau lebih gelombang dari sumber gelombang
interferens yang koheren.
gelombang The effect of superposition of two or more waves from a coherent source of waves.
Definition of
interference of
waves
Maksud Dua sumber gelombang yang menghasilkan gelombang dengan frekuensi yang sama,
gelombang yang panjang gelombang yang sama dan beza fasa yang tetap (sefasa).
koheren Two sources of waves which produce waves with the same frequency, same wavelength and the
Meaning of phase difference is constant.
coherent waves
Definisi prinsip Apabila dua gelombang merambat serentak dan bertindih pada satu titik, sesaran paduan
superposisi ialah hasil tambah sesaran individu bagi kedua-dua gelombang.
Definition of When two waves move simultaneously and coincide at a point, the resultant displacement is the
the principle of sum of the individual displacements of the two waves.
superposition
Kesan daripada superposisi dua puncak gelombang atau dua lembangan gelombang dari
sumber yang koheren untuk menghasilkan amplitud yang maksimum .
The effect of the superposition of two wave crests or two waves troughs of coherent source to

Interferens generate the maximum amplitude.


membina Gelombang paduan = 2a
Constructive Gelombang paduan = 2a Resulting wave = 2a
Resulting wave = 2a
interference Puncak / Crest

a a
2a 2a
a a
Lembangan / Trough

Kesan daripada superposisi satu puncak gelombang dan satu lembangan gelombang dari
sumber yang koheren untuk menghasilkan amplitud sifar. / The effect of superposition of one
wave crest and one wave trough from a coherent source to produce a zero amplitude.
Interferens
memusnah Gelombang paduan = 0
Destructive Puncak / Crest a Resulting wave = 0
interference a U
U
Lembangan / Trough N
N
II
T
T
15
Antinod Titik di mana interferens membina terbentuk.
Antinode A point where constructive interference occurs.
Video
Nod Titik di mana interferens memusnah terbentuk.
Node A point where destructive interference occurs.

Interferens Gelombang
Interference of Waves

137 © Nilam Publication Sdn. Bhd.


MODUL • Fizik TINGKATAN 4

Eksperimen Interferens gelombang air


Experiment Interference of water waves

Inferens / Inference

Jarak antara dua garis nod yang berturutan bergantung kepada jarak antara dua sumber koheren.
The distance between two consecutive nodal lines depends on the distance between two coherent sources.

Hipotesis / Hypothesis

Jarak antara dua garis nod berturutan, x bertambah apabila jarak antara dua sumber koheren, a berkurang.
The distance between two consecutive nodal lines, x increases as the distance between two coherent sources, a decreases.

Tujuan / Aim
Mengkaji hubungan antara jarak antara dua sumber koheren, a, dan jarak antara dua garis nod yang berturutan, x.
To investigate the relationship between the distance of two coherent sources, a, and the distance of two consecutive nodal

lines, x.

Pemboleh ubah / Variables


1 Pemboleh ubah dimanipulasikan: / Manipulated variable:
Jarak antara dua sumber koheren, a / The distance between two coherent sources, a

2 Pemboleh ubah bergerak balas: / Responding variable:


Jarak antara dua garis nod yang berturutan, x / The distance between two consecutive nodal lines, x

3 Pemboleh ubah yang dimalarkan: / Constant variable:


Frekuensi penggetar, panjang gelombang, jarak antara sumber (pencelup) dan kedudukan di mana x diukur.
Frequency of vibrator, the wavelength, distance between sources and the position where x is measured.

Senarai radas dan bahan / List of apparatus and materials


Tangki riak, lampu, motor, bar kayu, bekalan kuasa, kertas putih, penggetar sfera, pembaris meter, air dan stroboskop
mekanikal.
Ripple tank, lamp, motor, wooden bar, power supply, white paper, spherical dippers, metre rule, water and mechanical
stroboscope

Susunan radas / Arrangement of the apparatus


Lampu / Lamp
Motor
Motor Air
Water Pencelup
U
N Dipper Tangki riak
I Ripple tank
T

5 Stroboskop mekanikal
Mechanical stroboscope
Penggetar sfera
Spherical dippers

Bayang corak interferens Kertas putih (skrin)


Shadow of interference patterns White paper (screen)

© Nilam Publication Sdn. Bhd. 138


MODUL • Fizik TINGKATAN 4

Prosedur / Procedure
1 Radas disusun seperti ditunjukkan pada rajah di sebelah.
The apparatus is set up as shown in the diagram.
2 Dengan menggunakan pembaris meter, jarak antara dua pencelup diukur, a = 2.0 cm.
By using a metre rule, the distance between two dippers is measured, a = 2.0 cm.
3 Bekalan kuasa dihidupkan untuk menghasilkan dua gelombang membulat dari pencelup. Gelombang dibekukan
dengan menggunakan stroboskop mekanikal. Gelombang dilakarkan pada skrin.
The power supply is switched on to produce two circular waves from the dippers. The waves are frozen by a mechanical
stroboscope. The waves are sketched on the screen.
4 Dengan menggunakan pembaris meter, jarak antara dua garis nod yang berturutan, x, diukur dan direkodkan.
By using a metre rule, the distance between two consecutive nodal lines, x, is measured and recorded.
5 Eksperimen diulangi pada jarak yang berbeza antara dua pencelup, a = 4.0 cm, 6.0 cm, 8.0 cm dan 10.0 cm.
The experiment is repeated with different values of the distance between two dippers a = 4.0 cm, 6.0 cm, 8.0 cm and
10.0 cm.
6 Semua bacaan direkodkan dalam di dalam jadual.
All readings are recorded in a table.
Keputusan / Results

a / cm 2.0 4.0 6.0 8.0 10.0


x / cm

Analisis data / Analysis of data


1 Graf jarak antara dua garis nod berturutan, x melawan jarak antara dua pencelup, a diplot dengan menggunakan
kertas graf.
A graph of distance between two consecutive nodal lines, x against distance between the two dippers, a is plotted by
using a graph paper.

2 Nyatakan sebab berlakunya kawasan cerah dan kawasan gelap pada corak inteferens tersebut.
State the causes of formation of bright and dark regions in the interference pattern.

Eksperimen Interferens gelombang cahaya


Experiment Interference of light waves

Inferens / Inference

Jarak antara dua pinggir cerah yang berturutan bergantung kepada panjang gelombang cahaya.
U
U
The distance between two consecutive bright fringes depends on the wavelength of the light waves. N
N
II
T
T
Hipotesis / Hypothesis
15
Jarak antara dua pinggir cerah yang berturutan, x bertambah apabila panjang gelombang cahaya, λ bertambah.
The distance between two consecutive bright fringes, x increases when the wavelength of the light waves, λ increases.

Tujuan / Aim
Mengkaji hubungan antara panjang gelombang cahaya, λ dan jarak antara dua pinggir cerah yang berturutan, x.
To investigate the relationship between the wavelength of light waves, λ and the distance between two consecutive bright
fringes, x.

139 © Nilam Publication Sdn. Bhd.


MODUL • Fizik TINGKATAN 4

Pemboleh ubah / Variables


1 Pemboleh ubah dimanipulasikan: / Manipulated variable:
Panjang gelombang cahaya laser, λ. / The wavelength of the laser light waves, λ

2 Pemboleh ubah bergerak balas: / Responding variable:


Jarak antara dua pinggir cerah berturutan, x. / The distance between two consecutive bright fringes, x.

3 Pemboleh ubah yang dimalarkan: / Constant variable:


Jarak pemisahan antara celah, a dan jarak antara dwicelah dan skrin, D.
Slit separation, a, and the distance between double slit and screen, D

Senarai radas dan bahan / List of apparatus and materials


Sumber cahaya laser, skrin, dwicelah, pembaris meter dan pembaris 15 cm
Laser light source, screen, double slit, metre ruler and 15 cm ruler.

Susunan radas / Arrangement of the apparatus

Skrin
Screen

L

Sumber cahaya laser a Kawasan


Laser light source
interferens
Slit dwicelah Interference Corak pinggir interferens
Double slit area Interference fringe pattern
D

Prosedur / Procedure
1 Radas disusun seperti ditunjukkan di dalam rajah.
The apparatus is set up as shown in the diagram.
2 Sumber cahaya laser dengan panjang gelombang, λ1, dihidupkan.
The laser light source with a wavelength, λ1, is switched on.

3 Corak interferens terbentuk di atas skrin diperhatikan dan dilakarkan.


The interference pattern formed on the screen is observed and drawn.

4 Dengan menggunakan pembaris 15 cm, jarak merentasi 5 pinggir cerah yang berturutan, L diukur dan
direkodkan. / By using a 15 cm ruler, the distance across 5 consecutive bright fringes, L is measured and recorded.
L cm
5 Jarak antara dua pinggir cerah yang berturutan dikira, x =
4
U The distance between two consecutive bright fringes, x = L cm is calculated.
N 4
I
T 6 Eksperimen diulangi dengan sumber laser yang berbeza supaya panjang gelombang = λ2, λ3, λ4, λ5.
The experiment is repeated with different laser light sources so that the wavelength = λ2, λ3, λ4 , λ5.
5
Keputusan / Results

λ / nm
x / cm

© Nilam Publication Sdn. Bhd. 140


MODUL • Fizik TINGKATAN 4

Analisis data / Analysis of data


1 Graf jarak antara dua pinggir cerah yang berturutan, x melawan panjang gelombang, λ diplot dengan
menggunakan kertas graf.
A graph of distance between two consecutive bright fringes, x against wavelength, λ is plotted by using a graph paper.
2 Terangkan kewujudan jalur gelap dalam corak interferens.
Explain the formation of dark fringes in the interference of patterns.

Eksperimen Interferens gelombang bunyi


Experiment Interference of sound waves

Inferens / Inference

Jarak antara dua kawasan bunyi kuat berturutan bergantung kepada jarak antara pendengar dan dua pembesar suara.
The distance between two successive loud regions depends on the distance between the listener and the two loudspeakers.

Hipotesis / Hypothesis

Jarak antara dua kawasan bunyi kuat berturutan, x bertambah apabila jarak antara pendengar dan dua pembesar
suara, D bertambah. / The distance between two successive loud regions, x increases as the distance between the listener and
the two loudspeakers, D increases.

Tujuan / Aim
Mengkaji hubungan antara jarak antara pendengar dari dua pembesar suara, D dengan jarak antara dua kawasan
bunyi kuat yang berturutan, x. / To investigate the relationship between the distance between the listener and the two
loudspeakers, D, and the distance between two successive loud regions, x.

Pemboleh ubah / Variables


1 Pemboleh ubah dimanipulasikan: / Manipulated variable:
Jarak antara pendengar dan dua pembesar suara, D. / The distance between the listener and two loudspeakers, D.

2 Pemboleh ubah bergerak balas: / Responding variable:


Jarak antara dua kawasan bunyi kuat yang berturutan, x. / The distance between two successive loud regions, x.

3 Pemboleh ubah yang dimalarkan: / Constant variable:


Panjang gelombang bagi gelombang bunyi / Jarak antara dua pembesar suara.
The wavelength of sound waves / Distance between the two loudspeakers.

Senarai radas dan bahan / List of apparatus and materials


Dua pembesar suara, penjana isyarat audio, dawai penyambung dan pembaris meter. U
U
Two loudspeakers, audio generator, connection wires and metre rule. N
N
II
T
T
Susunan radas / Arrangement of the apparatus
15
Penjana isyarat audio / Audio generator

Pembesar suara Pembesar suara


Loudspeaker Loudspeaker

a=1m

Pendengar D=2m
Listener

141 © Nilam Publication Sdn. Bhd.


MODUL • Fizik TINGKATAN 4

Prosedur / Procedure

1 Dua pembesar suara dilaraskan pada jarak 1 m antara kedua-duanya.


The two loudspeakers are adjusted at 1 m apart.
2 Dengan menggunakan pembaris meter, jarak antara pendengar dan pembesar suara, D = 2.0 m, diukur.
By using a metre rule, the distance between the listener and the loudspeaker, D = 2.0 m, is measured.
3 Penjana isyarat audio dihidupkan. Eksperimen ini dijalankan di gelanggang terbuka supaya tiada gema.
The audio generator is switched on. This experiment is conducted in an open court so that there is no echo.
4 Pendengar berjalan selari dengan sisi depan meja dan jarak antara dua kawasan bunyi kuat berturutan, x diukur
dan direkodkan. / The listener walks in a straight path parallel to the front edge of the table and the distance between
two successive loud regions, x, is measured and recorded.
5 Eksperimen diulangi pada jarak yang berbeza antara pendengar dengan pembesar suara, D = 4.0 m, 6.0 m,
8.0 m dan 10.0 m. / The experiment is repeated with different distances between the listener and the loudspeakers,
D = 4.0 m, 6.0 m, 8.0 m and 10.0 m.

Keputusan / Results

D/m 2.0 4.0 6.0 8.0 10.0


x/m

Analisis data / Analysis of data

1 Graf jarak antara dua kawasan kuat berturutan, x melawan jarak antara pendengar dan pembesar suara, D
diplot. / A graph of the distance between two successive loud regions, x against the distance between the listener and
loudspeaker, D is plotted.

Rajah corak gelombang interferens: Rumus Interferens Young: ax


Diagram of patterns of wave interference: Young’s Interference Formula:
λ=
D

Garis antinod Garis nod


(Interferens membina) (Interferens memusnah)
Antinodal line Nodal line
(Constructive interference) (Destructive interference) D

Puncak
S1 S2 Crest
a
Lembangan
Trough

a = Jarak antara dua sumber koheren


U Distance between two coherent sources
N
I = Panjang gelombang / Wavelength
S1 S2
T x = Jarak antara dua garis nod atau garis
5 Sumber gelombang
Waves sources antinod yang berturutan / Distance between
two consecutive nodal lines or antinodal lines
D = Jarak berserenjang dari dua sumber ke
Puncak / Crest
titik pengukuran x
Lembangan / Trough
Perpendicular distance from the two sources
Nod / Node
to the point of measurement of x
Antinod / Antinode

© Nilam Publication Sdn. Bhd. 142


MODUL • Fizik TINGKATAN 4

5.6.4 Menyelesaikan masalah yang melibatkan interferens gelombang


SP
5.6.5 Berkomunikasi untuk menerangkan aplikasi interferens gelombang dalam kehidupan harian

Aplikasi Inteferens Gelombang dalam Kehidupan Harian


Applications of Interference of Waves in Daily Life

1 Kapal yang besar mempunyai stuktrur luan bebuli untuk menjana gelombang air yang berinterferens secara
memusnah dengan haluan kapal. Ini dapat membuatkan air di sekitar kapal menjadi lebih tenang dan
mengurangkan seretan air.
Large ship has a structure of bulbous bow that generates water waves which interfere destructively with the water waves
around the hull. This causes the water around the ship to become calmer and reduce the water drag.
2 Kanta anti-pantulan mempunyai salutan pada permukaannya bagi memantulkan cahaya berinterferens secara
memusnah. Ini bagi mengelakkan pembentukan imej pada kanta cermin mata dan mendapatkan penglihatan
yang lebih jelas.
Anti-reflection lens has a coating on the surface to reflect light to interfere destructively. This is to prevent any image
formed on the lens and to get a clearer vision.
3 Fon kepala yang dipakai oleh juruterbang mempunyai sistem mikrofon dan pemancar Maklumat tambahan:
Additional information:
yang menghasilkan gelombang bunyi secara interferens memusnah dengan bunyi
sekeliling yang bising.
Head phones used by pilots have a microphone and transmitter system that produce sound waves
which interfere destructively with the surrounding noise.

Latihan / Exercises
1 Dalam satu eksperimen untuk mengkaji corak interferens gelombang air, jarak antara dua pencelup sfera ialah
2.5 cm dan jarak antara dua garis antinod berturutan pada titik pengukuran ialah 5.0 cm. Berapakah panjang
gelombang bagi air jika jarak dari dua pencelup ke titik pengukuran itu ialah 10.0 cm?
In an experiment to investigate the interference pattern of water waves, the distance between two spherical dippers is
2.5 cm and the distance between two consecutive antinodal lines is 5.0 cm. What is the wavelength of the water waves
if the distance from the two dippers to the point of measurement is 10.0 cm?
Penyelesaian / Solution:
Diberi / Given a = 2.5 cm, x = 5.0 cm, D = 10.0 cm
ax
λ =
D
= 2.5 cm × 5.0 cm
10.0 cm
= 1.25 cm

2 Isyarat sonar telah dihantar secara menegak ke dasar laut dari sebuah kapal dan dipantulkan dari dasar laut
U
dan dikesan oleh mikrofon 0.8 s selepas dipancarkan. Jika halaju bunyi dalam air ialah 1 500 m s–1, berapakah U
N
N
kedalaman dasar laut itu? II
T
T
A sonar signal sent vertically downwards from a ship is reflected from the ocean floor and detected by a microphone
on the keel 0.8 s after transmission. If the speed of sound in water is 1 500 m s–1, what is the depth of the ocean? 15
Penyelesaian / Solution:
2d = v × t
2d = (1 500 m s–1) × (0.8 s)
1 500 m s–1 × 0.8 s
d = = 600 m
2

143 © Nilam Publication Sdn. Bhd.


MODUL • Fizik TINGKATAN 4

3 Dalam eksperimen interferens cahaya yang menggunakan cahaya hijau dengan panjang gelombang
5 × 10–7 m, jarak antara dua pinggir cerah yang terbentuk pada skrin ialah 0.4 mm. Apabila eksperimen
diulangi dengan menggunakan cahaya monokromatik S, jarak antara dua pinggir cerah yang terbentuk ialah
0.48 mm. Berapakah panjang gelombang cahaya S? / In an experiment of light interference using green light of
wavelength 5 × 10–7 m, two consecutive bright fringes formed on the screen are 0.4 mm apart. When the experiment is
repeated using monochromatic light S, two consecutive bright fringes formed are 0.48 mm apart. What is the wavelength
of light S?
Penyelesaian / Solution:
Diberi λHijau / Given λGreen = 5 × 10–7 m,
xHijau / Green = 4 × 10–4 m, xs = 4.8 × 10–4 m
ax
Dengan menggunakan λ / By using λ = ,
D
a a
λHijau = ( )(xHijau ) ("a" dan D adalah malar di sini), / λGreen = ( D )(xGreen ) (here, "a" and D are constants),
D
λHijau / Green
\( a )= (i)
D xHijau / Green
a
Tetapi / But λS = ( )(xS)
D
λ
\(a)= S (ii)
D xS
Pers. (i) = Pers. (ii); / Equation (i) = Equation (ii);
λHijau / Green λS
\ =
xHijau / Green xS
λHijau / Green
\ λS = (
xHijau / Green )(xS)
–7
= (5 × 10–4 m) × (4.8 × 10–4 m)
(4 × 10 m)
= 6.0 × 10–7 m

4 Rajah di sebelah menunjukkan corak pinggir yang terbentuk dalam


eksperimen dwicelah apabila cahaya monokromatik digunakan. Jarak
antara celah ialah 0.5 mm dan skrin adalah sejauh 3.0 m dari dwicelah itu.
The diagram on the right shows the fringe pattern obtained in a double slit
experiment when a monochromatic light is used. The double slits are 0.5 mm
apart and the screen is 3.0 m away from the double slits.
1.4 cm
Berapakah panjang gelombang cahaya monokromatik itu?
What is the wavelength of the monochromatic light?
Penyelesaian / Solution:
Diberi / Given:
a = 0.5 mm = 0.5 × 10–3 m
D = 3.0 m
U
N x = 1.4 cm
I 5
T
= 0.28 cm
5 = 0.28 × 10–2 m
ax
λ =
D
(0.5 × 10–3 m) × (0.28 × 10–2 m)
=
3.0 m
= 4.67 × 10–7 m

© Nilam Publication Sdn. Bhd. 144


MODUL • Fizik TINGKATAN 4

5 Rajah di bawah menunjukkan dua pembesar suara disambungkan kepada satu penjana isyarat audio untuk
menghasilkan dua set gelombang bunyi yang koheren. Penjana isyarat audio dihidupkan. Seorang pelajar
yang berjalan di sepanjang garis lurus XY pada jarak 5.0 m dari pembesar suara mendengar satu siri bunyi
kuat dan lemah yang berselang-seli di titik P, Q, R, S dan T.
The diagram below shows the two speakers connected to an audio signal generator to generate two sets of sound waves
that are coherent. Audio signal generator is switched on. A student who walked along the straight line XY at a distance
of 5.0 m from the speaker hear a series of alternating loud and soft sounds at points P, Q, R, S and T.
Penjana isyarat audio
Audio signal generator
Pembesar suara Pembesar suara
Loudspeaker Loudspeaker

2.0 m

5.0 m

X Y
P Q R S T

(a) Dua pembesar suara itu dipisahkan dengan jarak 2.0 m. Gelombang bunyi yang dihasilkan mempunyai
frekuensi 1 200 Hz dan laju 300 m s–1.
Two speakers are separated by a distance of 2.0 m. The sound waves produced has a frequency of 1200 Hz and at
speed of 300 m s–1.
(i) Hitungkan panjang gelombang bagi gelombang bunyi.
Calculate the wavelength of the sound waves.
(ii) Hitungkan jarak di antara dua bunyi kuat yang berturutan.
Calculate the distance between two consecutive strong sounds.
(b) Menggunakan konsep interferens gelombang dan konsep fizik lain yang sesuai, terangkan bagaimana jarak
di antara dua bunyi kuat yang berturutan berkurang jika eksperimen ini dijalankan pada waktu malam.
Using the concept of wave interference and other appropriate physics concept, explain how the distance between two
consecutive loud noise is reduced if this experiment is carried out at night.
Penyelesaian / Solution:
(a) (i) v = fλ (b) • Waktu malam, suhu udara menurun. [M1]
300 = 1 200 x λ At night, temperatures drop.
• Ketumpatan udara bertambah. [M2]
λ = 0.25 m Density of air increases.
ax • Panjang gelombang bunyi berkurang. [M3]
(ii) λ=
D The wavelength of sound wave decreases.
(2.0)x
0.25 = • x berkadar langsung dengan λ. [M4]
5.0
x is directly proportional to λ.
\ x = 0.625 m

U
U
N
N
II
T
T
15

145 © Nilam Publication Sdn. Bhd.


MODUL • Fizik TINGKATAN 4

5.7 GELOMBANG ELEKTROMAGNET / ELECTROMAGNETIC WAVES SK 5.7

5.7.1 Mencirikan gelombang elektromagnet


SP 5.7.2 Menyatakan komponen-komponen spektrum elektromagnet mengikut urutan dari segi panjang gelombang dan frekuensi
5.7.3 Berkomunikasi untuk menerangkan aplikasi setiap komponen spektrum elektromagnet dalam kehidupan

1 Gelombang elektromagnet ialah gelombang melintang, yang terdiri daripada ayunan medan elektrik dan
medan magnet yang berserenjang dengan satu sama lain.
Electromagnetic waves are transverse waves, consisting of oscillating electric fields and magnetic fields which are
perpendicular to each other.

Medan elektrik, E / Electric field, E

Medan magnet, B
Magnetic field, B

Arah perambatan
Direction of propagation

2 Spektrum elektromagnet ialah siri gelombang-gelombang elektromagnet mengikut ketertiban panjang


gelombang secara menurun atau frekuensi secara menaik.
Electromagnetic spectrum is a series of electromagnetic waves in descending order of wavelength or ascending order
of frequency.

Panjang gelombang/ m
Panjang gelombang semakin besar / Increasing wavelength Wavelength / m
10–11 10–10 10–9 10–8 10–7 10–6 10–5 10–4 10–3 10–2 10–1 1 101 102 103 104

Gelombang
Sinar gama Sinar-X Ultraungu Infra merah Radio
mikro
Gamma ray X-rays Ultraviolet Infrared Radio
Microwave

1020 1019 1018 1017 1016 1015 1014 1013 1012 1011 1010 109 108 107 106 105 104 Frekuensi / Hz
Frequency / Hz
U Cahaya Frekuensi semakin tinggi / Increasing frequency
N
I nampak
T Visible light
5

© Nilam Publication Sdn. Bhd. 146


MODUL • Fizik TINGKATAN 4

Ciri-ciri Gelombang Elektromagnet


Charateristics of Electromagnet Waves

Komponen medan magnet dan medan


elektrik saling bergetar berserenjang
antara satu sama lain dan ke arah
perambatan gelombang.
The magnetic field and electric field
components of the wave oscillate at right
angles to each other and to the direction
of propagation of the wave.
Gelombang Boleh
melintang dikutubkan .
Transverse Can be
waves polarised .

Boleh merambat
Mematuhi persamaan Ciri-ciri
melalui vakum pada
gelombang, Gelombang
kelajuan cahaya,
c (halaju cahaya) = f λ Elektromagnet
Characteristics of c = 3 × 108 m s–1
Obey the wave equation, Can travel through a
c (speed of light) = f λ Electromagnetic
Waves vacuum at the speed of
light, c = 3 × 108 m s–1

Tenaga
dipindahkan oleh Neutral
gelombang. elektrik
Energy Electrically
neutral
is transferred by the
waves.

U
U
N
N
Tidak memerlukan Mempunyai fenomena yang sama seperti: II
T
T
medium Undergo the same phenomena such as:
perambatan.
untuk
Pantulan, pembiasan, pembelauan
15
Do not require a
dan interferens
medium
Reflection, refraction, diffraction
for propagation.
and interference

147 © Nilam Publication Sdn. Bhd.


MODUL • Fizik TINGKATAN 4

Aplikasi gelombang elektromagnet dan kesan buruknya


The applications of electromagnetic waves and its negative effects

Jenis gelombang Aplikasi Kesan buruk


Types of waves Application Negative effects
• Komunikasi radio jarak jauh
Long distance radio communication
Dos berlebihan boleh
• Penyiaran (TV dan radio) menyebabkan kanser
Gelombang radio Radio and TV broadcasting atau leukimia
Radio wave • Komunikasi tanpa wayar Large doses of radio wave
Wireless communication may cause cancer or
leukaemia
• Untuk mengimbas badan penumpang di lapangan terbang
To scan body of passengers at airport
• Satelit komunikasi / Communication satellites
• Radar pesawat / Plane radar Katarak, kesan pada
Gelombang
otak
mikro • Memasak menggunakan ketuhar gelombang mikro Cataracts, effects on the
Microwave Cooking by using a microwave oven brain
• Pemerangkap laju / Speed trap
• Memasak / Cooking
Sinaran • Alat kawalan jauh / Remote control Pemanasan melampau
inframerah
• Kamera keselamatan / Security camera Overheating
Infrared ray
• Rawatan otot / Treatment for muscle
Terlalu banyak cahaya
• Penglihatan / For vision menyebabkan kerosakan
Cahaya nampak
• Fotografi / Photography retina
Visible light
• Fotosintesis / Photosynthesis To much light can damage
the retina

• Mengesan wang kertas palsu Dos sinaran UV yang


Detecting authenticity of currency notes tinggi boleh merosakkan
retina dan menyebabkan
Sinar ultraungu • Penulenan air minuman
Ultraviolet ray kanser kulit
Purification of drinking water
High doses of UV ray can
• Pensterilan alat perubatan damage retina and causes
Sterilising medical instruments skin cancer
• Mengesan keretakan tulang atau organ dalaman
To detect fractures bones or internal organs Kerosakan sel yang
Sinar-X • Mengimbas barang di lapangan terbang menyebabkan kanser
X-rays To scan baggages at airport Cell damage that cause
U
N • Mengesan keaslian lukisan cancer
I
T To determine authenticity of paintings
5 • Rawatan kanser
Cancer treatment Kerosakan sel yang
menyebabkan kanser
Sinar gama • Pensterilan alat perubatan
Gamma ray dan mutasi
Sterilising medical instruments
Cell damage that cause
• Mengesan kebocoran paip cancer and mutations
To detect pipe leakage

© Nilam Publication Sdn. Bhd. 148


MODUL • Fizik TINGKATAN 4

Praktis SPM / SPM Practice


Soalan Objektif / Objective Questions
1 Graf yang manakah menunjukkan hubungan 3 Antara rajah berikut, yang manakah menunjukkan
antara frekuensi, f, dengan tempoh, T, bagi suatu contoh gelombang membujur? / Which diagram
gelombang? shows an example of a longitudinal wave?
Which graph shows the relationship between frequency, A Cahaya bergerak dari lampu ke skrin.
f and period, T, of a wave? Light traveling from a lamp to a screen.
A f B f

Lampu Kanta Skrin


Lamp Lens Screen

0 T 0 T
B Riak air disebabkan pencelup bergetar ke atas
f f dan ke bawah.
C D Water ripple caused by a dipper moving up and down.

Pencelup
Dipper Air
Water
0 T 0 T
C Spring ditolak ke hadapan dan ke belakang.
A spring is pushed forwards and backwards.
2 Rajah 1 menunjukkan gelombang bunyi dipantulkan
pada dinding konkrit.
Diagram 1 shows a sound wave reflected on a concrete D Spring ditolak ke atas dan ke bawah.
wall. A spring is pushed up and down.
Gelombang tuju Dinding konkrit
Incident wave Concrete wall

4 Rajah 2 menunjukkan penggunaan gelombang


Gelombang pantulan elektromagnet. / Diagram 2 shows the application of
Reflected wave
electromagnetic wave.
Rajah 1 / Diagram 1
Perbandingan manakah yang betul tentang pantulan
gelombang bunyi dan gelombang tuju?
Which comparison is correct about the reflected sound
wave and the incident sound wave?
A Panjang gelombang bagi gelombang tuju lebih Rajah 2 / Diagram 2
pendek daripada gelombang pantulan. Pasangan manakah menunjukkan tentang ciri-ciri U
U
The wavelength of the incident wave is shorter than gelombang yang digunakan? N
N
II
the reflected wave. Which pair shows the characteristics of the wave used? T
T
B Laju gelombang tuju dan gelombang pantulan
adalah sama. / The speed of the incident wave and Frekuensi Panjang gelombang 15
the reflected wave is the same. Frequency Wavelength
C Frekuensi gelombang tuju kurang daripada A Tinggi / High Pendek / Short
gelombang pantulan. / The frequency of the
incident wave is less than the reflected wave. B Tinggi / High Panjang / Long
D Sudut gelombang tuju lebih besar daripada sudut
gelombang pantulan. / The angle of incident wave C Rendah / Low Panjang / Long
is greater than the angle of reflected wave.
D Rendah / Low Pendek / Short

149 © Nilam Publication Sdn. Bhd.


MODUL • Fizik TINGKATAN 4

5 Rajah 3 menunjukkan perambatan gelombang air 7 Rajah 5 menunjukkan satu alat yang digunakan oleh
melalui blok perspek di dalam tangki riak. pengurup wang untuk mengesan wang kertas palsu.
Diagram 3 shows water wave propagating through a Diagram 5 shows a device used by a money-changer to
perspex block in a ripple tank. detect counterfeit bank notes.

Blok perspek
Perspex block

Rajah 3 / Diagram 3 Rajah 5 / Diagram 5


Corak gelombang manakah yang diperhatikan Jenis gelombang elektromagnet yang digunakan
apabila gelombang merentasi blok perspek? ialah / The type of electromagnetic wave used is
Which wave pattern is observed when the wave pass A gelombang mikro / microwave
through the perspex block? B inframerah / infrared
A C C sinar ultraungu / ultraviolet ray
D sinar-X / X-ray

8 Rajah 6 menunjukkan imej yang diperoleh seorang


B D guru yang mengendalikan suatu aktiviti dengan
cahaya laser serta radas yang lain. / Diagram 6
shows an image obtained by a teacher who conducted
an activity with a laser light and other apparatus.

6 Tsunami adalah satu siri gelombang air yang


disebabkan oleh sesaran isi padu air yang besar,
biasanya terjadi dalam lautan atau tasik yang
besar. Sesaran air ini selalunya menyumbang
Rajah 6 / Diagram 6
kepada kejadian gempa bumi, gelinciran tanah,
Rajah 6 menunjukkan fenomena
letupan gunung berapi dan runtuhan salji. Didapati
Diagram 6 shows the phenomenon of
bahawa panjang gelombang, λ tsunami semakin
A pantulan / reflection
pendek apabila menghampiri pantai seperti yang
B pembiasan / refraction
ditunjukkan dalam Rajah 4.
A tsunami is a series of water wave caused by the
C pembelauan / diffraction
displacement of a large volume of water, usually occurs D interferens / interference
in ocean or large lake. This displacement of water is
usually contributes to earthquakes, landslides, volcanic 9 Rajah 7 menunjukkan satu corak interferens.
eruptions and avalanche. It is found that the wavelength, Diagram 7 shows an interference pattern.
λ of the tsunami becomes smaller towards the beach as
Puncak
shown in the Diagram 4. Source 1 Crest
Sumber 1
a 2.5 cm
Source 2
Sumber 2 7.0 cm
U
N
I
T Rajah 7 / Diagram 7

5 Jika panjang gelombang ialah 1.0 cm, berapakah


nilai bagi jarak antara dua sumber, a?
Rajah 4 / Diagram 4
If the wavelength is 1.0 cm, then what is the value of a,
Ini disebabkan oleh / It is due to
the distance between the two sources?
A pantulan gelombang / reflection of waves
A 2.0 cm
B pembiasan gelombang / refraction of waves
B 2.4 cm
C pembelauan gelombang / diffraction of waves
C 2.8 cm
D interferens gelombang / interference of waves
D 3.2 cm

© Nilam Publication Sdn. Bhd. 150


MODUL • Fizik TINGKATAN 4

10 Rajah 8 menunjukkan dua denyutan gelombang 11 Rajah 9 menunjukkan pinggir-pinggir yang


dihasilkan pada P dan Q. P dan Q adalah sama diperoleh apabila cahaya hijau digunakan dalam
jarak dari X. eksperimen dwicelah Young.
Diagram 8 shows two wave pulses produced at P and Diagram 9 shows the fringes obtained when green light
Q. P and Q are at the same distance from X. is used in a Young’s double slit experiment.
P
a
a
X
Q
Rajah 9 / Diagram 9
Antara berikut, yang manakah merupakan pinggir-
Rajah 8 / Diagram 8 pinggir yang diperhatikan jika cahaya hijau
Bentuk gelombang yang manakah diperhatikan di digantikan dengan cahaya merah?
X? / Which waveform is observed at X? Which of the following fringes are observed when the
A green light is replaced by red light?
2a
A
B
2a
B
C

D a C

Soalan Struktur / Structure Questions


1 Rajah 1 menunjukkan spektrum bagi gelombang elektromagnet.
Diagram 1 shows the spectrum of electromagnetic waves.

Gelombang Gelombang Cahaya Sinar ultraungu Sinar-X Sinar gama


radio mikro Q nampak Ultraviolet X-ray Gamma ray
Radio waves Microwave Visible light

Rajah 1 / Diagram 1
(a) (i) Dengan merujuk kepada Rajah 1, namakan sinaran Q.
Based on Diagram 1, name the type of radiation Q.
Q ialah sinaran inframerah / Q is infrared ray

(ii) Namakan satu kegunaan sinaran Q.
Name one application of radiation Q.
U
Alat kawalan jauh. / Remote control. U
N
N
II
T
T
(b) Bandingkan panjang gelombang bagi gelombang mikro dengan panjang gelombang bagi sinar-X.
Compare the wavelength of microwave with wavelength of X-ray. 15
Panjang gelombang bagi gelombang mikro lebih panjang daripada panjang gelombang sinar-X.
The wavelength of microwave is longer than wavelength of X-ray.

(c) Namakan satu gelombang elektromagnet yang panjang gelombangnya lebih panjang daripada panjang
gelombang Q. / Name one electromagnetic wave which has longer wavelength than wavelength of Q.
Gelombang radio // gelombang mikro. / Radio wave // microwave.

151 © Nilam Publication Sdn. Bhd.


MODUL • Fizik TINGKATAN 4

(d) Namakan satu kuantiti fizik yang malar bagi semua gelombang elektromagnet.
Name one constant physical quantity in all electromagnetic waves.
Semua gelombang elektromagnet mempunyai laju yang sama dalam vakum.
All electromagnetic waves have the same speed in vacuum.

(e) Apakah bezanya di antara gelombang elektromagnet dengan gelombang bunyi?
What is the difference between electromagnetic wave and sound wave?
Gelombang elektromagnet ialah gelombang melintang manakala gelombang bunyi ialah gelombang
membujur. // Gelombang elektromagnet boleh merambat dalam vakum manakala gelombang bunyi tidak
boleh. / Electromagnetic wave is a transverse wave whereas sound wave is a longitudinal wave. // Electromagnetic
waves can travel in vacuum whereas sound waves cannot.

2 Rajah 2 menunjukkan seorang pemain gitar memetik tali gitar dan bunyi dihasilkan.
Diagram 2 shows a guitarist plucks the strings of a guitar and the sound is produced.

Rajah 2 / Diagram 2
(a) Dengan menggunakan konsep fizik yang sesuai, terangkan bagaimana seorang pendengar dapat
mendengar bunyi yang dihasilkan oleh gitar.
By using the appropriate physics concept, explain how a listener can hear the sound produced by the guitar.
• Tali gitar dipetik dan bergetar / Strings of guitar are plucked and strings vibrates.
• Getaran tali gitar menggetarkan molekul-molekul udara sekeliling.
Vibration guitar strings vibrate the air molecules around.
• Molekul-molekul udara mengalami mampatan dan renggangan secara berselang seli.
The molecules of air undergoes compression and rarefaction alternately.

• Tenaga dipindahkan ke telinga pendengar / Energy is transferred to the listener's ears.
• Gegendang telinga bergetar / Eardrum vibrates.

(b) Gitar itu akan digunakan dalam suatu persembahan. Cadang dan terangkan pengubahsuaian yang
perlu dibuat ke atas gitar tersebut supaya ia boleh menghasilkan bunyi secara lebih berkesan dan
bunyi gitar akan kedengaran lebih jelas oleh pendengar yang duduk jauh darinya. Cadangan anda perlu
U merangkumi aspek-aspek berikut:
N The guitar will be used in a presentation. Suggest and explain modifications to be made to the guitar so that it can
I
T generate sounds more effectively and guitar sound will be heard more clearly by the listener sitting far away from it.
5 Your suggestion should include the following aspects:
(i) Magnitud daya yang digunakan untuk memetik tali gitar.
The magnitude of the force used to pluck the strings of a guitar.
(ii) Tindakan pelarasan melalui tombol gitar.
Action of adjustment made through the knob.
(iii) Saiz kotak kayu.
The size of the wooden box.
(iv) Pengubahsuaian lain.
Other modifications.

© Nilam Publication Sdn. Bhd. 152


MODUL • Fizik TINGKATAN 4

Aspek cadangan / Suggestion aspect Sebab / Reason

Magnitud daya yang digunakan untuk memetik


tali gitar mestilah lebih besar • Amplitud lebih besar / Larger amplitude
The magnitude of the force used to pluck the strings of • Tenaga lebih besar / Larger energy
a guitar must be larger • Lebih nyaring / Louder

Tindakan pelarasan melalui tombol gitar iaitu


tombol diputar untuk mengetatkan tali-tali gitar
• Frekuensi lebih tinggi / Higher frequency
Action of adjustment made through the knob which is
• Lebih langsing / Higher pitch
the knob is rotated to tighten the strings of the guitar

• Lebih banyak molekul udara bergetar


Saiz kotak mesti lebih besar More air molecules vibrates
Size of the box must be larger • Lebih banyak tenaga dipindahkan
More energy transferred

• Tali tidak mudah terputus


Pengubahsuaian-pengubahsuaian lain:
Strings are not easily broken
Other modifications:
• Frekuensi lebih tinggi / lebih langsing
• Tali yang lebih kuat / Stronger strings
Higher frequency / Higher pitch
• Tali yang lebih halus / Thinner strings

Teknik menjawab [Format Kertas 2: Bahagian B] / Answering technique [Paper Format 2: Part B]

Rajah di bawah menunjukkan sebuah sistem radar yang digunakan untuk menentukan kedudukan sebuah kapal
terbang di lapangan terbang.
The diagram below shows a radar system used to determine the position of a plane at the airport.

Kapal terbang
Pemantul parabola Plane
Parabolic reflector Cadang dan terangkan bagaimana untuk
menghasilkan radar yang dapat menentukan
kedudukan sebuah kapal terbang dengan efektif.
Suggest and explain how to produce a radar that can
Pemancar / Penerima
Transmitter / Receiver determine the position of a plane effectively.
[10 markah / marks] U
U
N
N
II
T
T
Perhatian / Attention : 15
Jawapan boleh diolah dalam bentuk poin. / Answers can be processed in the form of points.
Untuk soalan dengan panduan aspek cadangan, pastikan ada sekurang-kurangnya lima aspek cadangan.
For questions with guidance aspects of the suggestion, make sure there are at least five aspects of suggestion.
Jika tidak cukup lima aspek cadangan, gunakan kreativiti sendiri untuk mencukupkannya supaya menjadi sekurang-kurangnya
lima aspek cadangan.
If not enough five aspects of suggestion, use your own creativity to suffice in order to be at least five aspects of suggestion.
Jika tidak diberi sebarang panduan aspek cadangan, anda perlu memikirkan sekurang-kurangnya lima aspek cadangan atas kreativiti
sendiri. / If not given any aspect of the guidelines, you should think of at least five aspects of suggestion on your own creativity.
Boleh beri lebih daripada satu sebab atau keterangan. / May give more than one reason or explanation.

153 © Nilam Publication Sdn. Bhd.


MODUL • Fizik TINGKATAN 4

Cadangan Sebab
Suggestion Reason

Jenis gelombang yang digunakan: / Wave type used:


Gelombang mikro • Tenaga lebih tinggi / Higher energy
Gelombang mikro [M1] • Frekuensi lebih tinggi / Higher frequency [M2]

Luas permukaan pemantul parabola: / The surface area of a parabolic reflector:


Lebih besar Menerima lebih banyak isyarat gelombang pantulan
Larger [M3] Receive more reflected signals [M4]

Jenis pemantul parabola: / Type of parabolic reflector:


Cekung Menumpukan isyarat gelombang pantulan yang dikesan
Concave [M5] To converge the reflected wave signals detected [M6]

Kedudukan pemancar: / Position of transmitter:


Pada titik fokus Isyarat gelombang tuju dapat dipancarkan ke jarak yang lebih
At focal point [M7] jauh / Incident wave signal can be transmitted to longer distances
[M8]

Kedudukan penerima: / Position of receiver:


Pada titik fokus Isyarat gelombang terpantul ditumpukan pada penerima
At focal point [M9] Reflected signals wave is focused on a receiver [M10]

Lokasi sistem radar dipasang: / The location of the radar system installed:
Tempat yang lebih tinggi dan terbuka • Kurang halangan / Less obstacle
Higher and open place [M11] • Tiada gangguan isyarat / There is no signal interruption [M12]

Amplitud gelombang isyarat tuju: / The amplitude of the incident signal:


Lebih besar Isyarat gelombang pantulan dapat dikesan dengan lebih mudah
Larger [M13] Reflected signals can be detected easily [M14]

U Jenis permukaan pemantul parabola: / Type of surface for parabolic reflector:


N
I
T Berkilat Pemantul yang lebih baik
5 Shiny [M15] Good reflector [M16]

© Nilam Publication Sdn. Bhd. 154


MODUL • Fizik TINGKATAN 4

Unit
CAHAYA DAN OPTIK
6 LIGHT AND OPTICS

6.1 PEMBIASAN CAHAYA / REFRACTION OF LIGHT SK 6.1

SP 6.1.1 Memerihalkan fenomena pembiasan cahaya

Berikan maksud • Pembengkokan lintasan cahaya yang berlaku di sempadan yang membahagikan dua
pembiasan cahaya. medium lutsinar dengan ketumpatan yang berbeza.
Give the definition of The bending of pathway of light at the boundary that separating two transparent medium of
light refraction. different optical density.
• Arah dan halaju cahaya berubah semasa mengalami pembiasan cahaya.
Direction and velocity of light change during refraction of light.
Terminologi dalam pembiasan cahaya:
Terminology in refraction of light:
(i) AO : Sinar tuju / Incident ray
N
A (ii) N : Normal / Normal
(iii) OB : Sinar biasan / Refracted ray
i (iv) Sudut tuju, i = sudut di antara normal dan sinar tuju.
Udara / Air Angle of incidence, i = angle between normal and
Blok kaca O
incident ray.
Glass block (v) Sudut biasan, r = sudut di antara normal dan sinar
r biasan.
Angle of refraction, r = angle between normal and
refracted ray.
B (vi) Sudut tuju dan sudut biasan MESTI diukur dari
normal.
Angle of incidence and angle of refraction MUST be
measured from normal.

Apabila cahaya merambat Penerangan


When light travels Explanation
1 Dari medium kurang tumpat ke medium lebih • Sinar cahaya membengkok mendekati normal.
tumpat. Light ray bend towards normal.
From less dense medium to denser medium. • Halaju cahaya berkurang.
Velocity of light decreases.
Sinar tuju
N
• Sudut tuju, i lebih besar daripada sudut
Incident ray biasan, r.
i Angle of incidence, i is bigger than angle of
Udara / Air
refraction, r.
Blok kaca / Glass block
Sinar biasan • Panjang gelombang berkurang. U
Refracted ray Wavelength decreases. N
r I
• Frekuensi malar. T
Frequency is constant. 6

155 © Nilam Publication Sdn. Bhd.

B6FizikF4(155-187)csy5p.indd 155 09/01/2020 9:56 AM


MODUL • Fizik TINGKATAN 4

2 Dari medium lebih tumpat ke medium kurang • Sinar cahaya membengkok menjauhi normal.
tumpat. / From denser medium to less dense medium. Light ray bends away from normal.
N • Halaju cahaya bertambah.
Sinar tuju
Incident ray Velocity of light increases.
i • Sudut tuju, i lebih kecil daripada sudut biasan r.
Blok kaca / Glass block Angle of incidence, i is smaller than angle of refraction, r.
Udara / Air Sinar biasan • Panjang gelombang bertambah.
r
Refracted ray Wavelength increases.
• Frekuensi malar. / Frequency is constant.
3 Bersudut tegak dengan sempadan dua medium. • Sinar cahaya dibiaskan tanpa kesan pembengkokan.
Perpendicular to the boundary of two mediums. Light ray is refracted without the bending effect.
N • Halaju cahaya bertambah.
Sinar tuju
Incident ray Velocity of light increases.
• Panjang gelombang bertambah.
Blok kaca / Glass block Wavelength increases.
Udara / Air
• Frekuensi malar.
Sinar biasan
Refracted ray Frequency is constant.

SP 6.1.2 Menerangkan indeks biasan, n

Berikan maksud • Indeks biasan, n ialah darjah pembengkokan alur cahaya apabila cahaya merambat
indeks biasan, n. dari vakum ke suatu medium. / Refractive index, n is the degree of light ray bends when
Give the definition travels from vacuum to a medium.
of refractive index, • Indeks biasan ialah nisbah laju cahaya di dalam vakum kepada laju cahaya di dalam
n. medium: / The refractive index is the ratio of the speed of light in a vacuum to the speed of
light in the medium:
laju cahaya dalam vakum / speed of light (in vacuum)
n=
laju cahaya dalam medium / speed of light (in medium)
c
n =
v
[di mana c ialah kelajuan cahaya / where c is the speed of light = 3 × 108 m s–1]
Hitung indeks Laju cahaya dalam
biasan, n, bagi Medium medium, v (m s–1) Indeks biasan, n
medium berikut Medium Speed of light in the Refractive index, n
di dalam jadual. medium, v (m s–1)
Calculate the
Vakum c 8 –1
refractive index, n, 3 × 108 n = v = 3 × 108 m s–1 = 1.00
for the following Vacuum 3 × 10 m s
mediums in the 8 –1
table.
Air
2.26 × 108 n = 3 × 10 8m s –1 = 1.33
c Water 2.26 × 10 m s
n= v; 8 –1
Minyak zaitun
2.05 × 108 n = 3 × 10 8m s –1 = 1.46
[c = 3 × 108 m s–1] Olive oil 2.05 × 10 m s
8 –1
Kaca
1.97 × 108 n = 3 × 10 8m s –1 = 1.52
Glass 1.97 × 10 m s
8 –1
Intan
1.24 × 108 n = 3 × 10 8m s –1 = 2.42
U Diamond 1.26 × 10 m s
N
I
T Apabila indeks biasan, n semakin besar: / When the refractive index, n becomes larger:
• Halaju cahaya dalam medium berkurang. / Velocity of light in medium decreases.
6 • Ketumpatan optik bahan bertambah. / The optical density of material increases.

© Nilam Publication Sdn. Bhd. 156

B6FizikF4(155-187)csy5p.indd 156 09/01/2020 9:56 AM


MODUL • Fizik TINGKATAN 4

SP 6.1.3 Menkonsepsikan Hukum Snell

Hukum pembiasan cahaya: / Law of refraction of light:


Hukum (a) sinar tuju, sinar biasan dan normal berada dalam satah yang sama.
pembiasan the incident ray, the refracted ray and the normal lie in the same plane.
cahaya (b) nisbah sin i kepada sin r adalah suatu pemalar
Law of refraction of the ratio of sin i to sin r is a constant
light sin i
= pemalar/constant
sin r
Daripada rumus: / From the formula of:
n1 sin θ1 = n2 sin θ2

n2 sin θ1
n1 = sin θ di mana / where, n1 ialah indeks biasan medium 1
2 where n1 is the refractive index for medium 1
n2 ialah indeks biasan medium 2
where n2 is the refractive index for medium 2
θ1 ialah sudut tuju bagi medium 1
Hukum Snell where θ1 is the angle of incidence for medium 1
Snell’s Law θ2 ialah sudut biasan bagi medium 2
where θ2 is the angle of refraction for medium 2

Apabila medium 1 adalah udara (n1 = 1) dan medium 2, n2 = n


When medium 1 is air (n1 = 1) and medium 2, n2 = n
sin i
n= di mana / where, i ialah sudut tuju dalam udara / i is the angle of incidence in air
sin r
r ialah sudut biasan dalam medium 2
r is the angle of refraction in medium 2

SP 6.1.4 Mengeksperimen untuk menentukan indeks biasan, n bagi blok kaca atau perspeks.

Eksperimen Hubungan antara sudut tuju, i dan sudut biasan, r


Experiment Relationship between the angle of incidence, i and the angle of refraction, r

Inferens Sudut biasan bergantung pada sudut tuju.


Inference Angle of refraction depends on angle of incidence.
Hipotesis Apabila sudut tuju, i, bertambah, sudut biasan, r, juga bertambah.
Hypothesis When angle of incidence, i, increases, angle of refraction, r, also increases.
Tujuan Untuk mengkaji hubungan antara sudut tuju dan sudut biasan.
Aim To investigate the relationship between angle of incidence and angle of refraction.
1 Pemboleh ubah dimanipulasikan: / Manipulated variable:
Sudut tuju, i / Angle of incidence, i

Pemboleh ubah 2 Pemboleh ubah bergerak balas: / Responding variable:


Variables Sudut biasan, r / Angle of refraction, r

3 Pemboleh ubah dimalarkan: / Constant variable:


Indeks biasan / Refractive index
U
Senarai radas N
Kertas putih, jangka sudut, kotak sinar dengan plat celah tunggal, blok, kaca, pensel dan I
dan bahan T
pembaris.
List of apparatus
and materials
White paper, protractor, ray box with single slit, glass block, pencil and ruler. 6

157 © Nilam Publication Sdn. Bhd.

B6FizikF4(155-187)csy5p.indd 157 09/01/2020 9:56 AM


MODUL • Fizik TINGKATAN 4

Kotak sinar
Normal
Ray box
Normal
Sinar tuju
Incident ray i

Sinar biasan
Susunan radas Refracted ray
Arrangement of the r
apparatus
Udara Blok kaca
Air Glass block

1 Radas disusun seperti dalam rajah di atas.


The apparatus is set up as shown in the diagram above.
2 Garis luar blok kaca dilukiskan di atas kertas putih. Lima garis pada sudut tuju yang
berbeza, i = 10º, 20º, 30º, 40º dan 50º dilukis pada kertas menggunakan jangka sudut.
An outline of glass block is traced on a white paper. Five lines at different angle of incidence,
i = 10º, 20º, 30º, 40º and 50º is drawn by using a protractor.

3 Sinar cahaya dari kotak sinar dituju ke arah tengah blok kaca pada sudut tuju,

Prosedur i = 10º.
Procedure
A light ray from ray box is incident at the centre of the glass block at an angle of incidence,
i = 10º.

4 Sudut biasan, r diukur dan direkodkan.


The angle of refraction, r is measured and recorded.
5 Langkah 3 dan langkah 4 diulang dengan sudut tuju, i = 20º, 30º, 40º dan 50º.

Step 3 and step 4 are repeated with angles of incidence, i = 20º, 30º, 40º and 50º.
6 Nilai sin i dan sin r dihitung dan direkodkan.
The values of sin i and sin r are calculated and recorded.

Sudut tuju, i (º) Sudut biasan, r (º)


sin i sin r
Angle of incidence, i (º) Angle of refraction, r (º)

10

Keputusan 20
Results
30
U
N 40
I
T
50
6

© Nilam Publication Sdn. Bhd. 158

B6FizikF4(155-187)csy5p.indd 158 09/01/2020 9:56 AM


MODUL • Fizik TINGKATAN 4

1 Dengan menggunakan kertas graf, plotkan sebuah graf untuk sin r melawan sin i.
By using a piece of graph paper, plot a graph of sin r against sin i.
2 Kecerunan, m, bagi graf sin r melawan sin i dihitung (tunjuk pengiraan anda).
The gradient, m, of the graph of sin r against sin i is calculated (show your calculation).
1 1
Analisis data 3 Tentukan nilai —. / Determine the value of — .
m m
Analysis of data
4 Oleh yang demikian, tentukan nilai indeks pembiasan bagi blok kaca daripada
eksperimen ini.
Hence, determine the value of the refractive index of the glass block from this experiment.
5 Adakah hipotesis diterima?
Is the hypothesis accepted?

Kesimpulan Apabila sudut tuju, i, bertambah, sudut biasan, r, juga bertambah.


Conclusion When angle of incidence, i, increases, angle of refraction, r, also increases.

SP 6.1.5 Menerangkan dalam nyata dan dalam ketara

Nyatakan definisi
Dalam nyata, H ialah jarak objek dari permukaan air.
dalam nyata. Real depth, H is the distance of object from the water surface.
Define real depth.
Nyatakan definisi
Dalam ketara, h ialah jarak imej dari permukaan air.
dalam ketara.
Apparent depth, h is the distance of image from the water surface.
Define apparent depth.

1 Sinar cahaya yang merambat dari ikan (objek) terbias


menjauhi normal apabila ia merambat dari air (medium N N
lebih tumpat) ke udara (medium kurang tumpat). Udara
Air
Light rays coming from the fish (object) are bent away from the
normal as they passing through from water (denser medium) to Air Dalam ketara, h
Water Apparent depth, h
air (less dense medium).
I
2 Apabila sinar cahaya sampai ke mata, ia seolah-olah datang Dalam nyata, H
Real depth, H
dari ikan maya (imej) yang berada di atas O
ikan sebenar.
When the light rays reach the eye, they appear to come from a
virtual fish (image) which is above the real fish.

3 Dalam ketara, h ialah jarak dari permukaan air ke imej, I.


The apparent depth, h is the distance from the surface of water to the image, I.

4 Dalam nyata, H ialah jarak dari permukaan air ke objek, O.


The real depth, H is the distance from the surface of water to the object, O.

5 Hubungan indeks biasan, n, dalam nyata dan dalam ketara ialah:


The relationship between of refractive index, n, to real depth and apparent depth is:
U
Dalam nyata/Real depth H N
n= = h I
Dalam ketara/Apparent depth T

159 © Nilam Publication Sdn. Bhd.

B6FizikF4(155-187)csy5p.indd 159 09/01/2020 9:56 AM


MODUL • Fizik TINGKATAN 4

SP 6.1.6 Mengeksperimen untuk menentukan indeks biasan menggunakan dalam nyata dan dalam ketara.

Eksperimen Hubungan antara dalam nyata dan dalam ketara


Experiment Relationship between real depth and apparent depth

Inferens Dalam ketara dipengaruhi oleh dalam nyata.


Inference The apparent depth influenced by the real depth.

Hipotesis Apabila dalam nyata, H suatu objek bertambah, dalam ketara, h turut bertambah.
Hypothesis When the real depth, H of an object increases, its apparent depth, h also increases.

Tujuan Untuk mengkaji hubungan antara dalam nyata dan dalam ketara.
Aim To investigate the relationship between the real depth and the apparent depth.
1 Pemboleh ubah dimanipulasikan: / Manipulated variable:
Dalam nyata, H / Real depth, H

Pemboleh ubah 2 Pemboleh ubah bergerak balas: / Responding variable:


Variables Dalam ketara, h / Apparent depth, h

3 Pemboleh ubah yang dimalarkan: / Constant variable:


Indeks biasan air, n / Refractive index of water, n

Senarai radas
dan bahan 2 pin, gabus, pita selofan, bikar (1 000 ml), air, pembaris meter dan kaki retort
List of apparatus 2 pins, cork, cellophone tape, 1 000 ml beaker, water, metre rule and retort stand
and materials
Pemerhati
Air / Water Observer

Pembaris meter
Susunan radas Bikar Metre rule
Arrangement of the Beaker
apparatus H h
Gabus / Cork
Imej pin O
Image of pin O Pin I
Pin I
Pin O / Pin O

1 Radas disusun seperti dalam rajah di atas.


The apparatus is set up as shown in the diagram above.
2 Pin O diletakkan pada dasar bikar menggunakan pita selofan.
Pin O is placed at the base of the beaker using cellophone tape.
3 Pin yang lain I diletakkan pada penutup gabus yang diapit oleh kaki retort. Ini
digunakan sebagai pengesan imej pin O. / The other pin I is fixed on a cork that is held
by the clamp of a retort stand. It is used as a detection of image of pin O.
4 Bikar diisi dengan air pada ketinggian, H = 20 cm (dalam nyata).
Prosedur
The beaker is filled with water to a height of H = 20 cm (real depth).
Procedure
5 Imej pin O dilihat secara menegak dari atas permukaan bikar.
The image of the pin O is viewed vertically from above of the beaker.
6 Imej pin O yang kelihatan menerusi air dilabelkan dengan pin I. Kedudukan imej,
U h diukur menggunakan pembaris meter. / The image of pin O which is seen through the
N water is lebelled with pin I. The position of image, h is measured using the metre rule.
I
T 7 Langkah 4 hingga 6 diulang dengan H = 30 cm, 40 cm, 50 cm dan 60 cm serta
6 direkodkan.
Steps 4 to 6 are repeated with H = 30 cm, 40 cm, 50 cm and 60 cm and recorded.

© Nilam Publication Sdn. Bhd. 160

B6FizikF4(155-187)csy5p.indd 160 09/01/2020 9:56 AM


MODUL • Fizik TINGKATAN 4

Dalam nyata Dalam ketara


Real depth Apparent depth
H (cm) h (cm)
Keputusan 20.0
Results 30.0
40.0
50.0
60.0

1 Dengan menggunakan kertas graf, plotkan sebuah graf bagi H melawan h.


Using a graph paper, plot a graph of H against h.
2 Kecerunan graf H melawan h dihitung (tunjuk pengiraan anda).
Analisis data The gradient of the graph of H against h is calculated (show your calculation).
Analysis of data
3 Apakah nilai indeks pembiasan air daripada eksperimen ini?
What is the value of the refractive index of water from this experiment?
4 Adakah hipotesis diterima? / Is the hypothesis accepted?

Kesimpulan Apabila dalam nyata, H suatu objek bertambah, dalam ketara, h turut bertambah.
Conclusion When the real depth, H of an object increases, its apparent depth, h also increases.

SP 6.1.7 Menyelesaikan masalah yang berkaitan dengan pembiasan cahaya

Latihan / Exercises
1 Tentukan indeks pembiasan 3 Berapakah halaju cahaya dalam medium dengan
kaca. 60° Udara indeks pembiasan 2.2?
Air
Determine the refractive index of What is the speed of light in a medium with a refractive
the glass. index of 2.2?
Kaca 35°
Penyelesaian / Solution: Glass
Penyelesaian / Solution:
sin i sin 60°
n= = = 1.51 n = 2.2
sin r sin 35°
c = Halaju dalam vakum / Speed in vacuum
= 3.0 × 108 m s–1
2 Halaju cahaya yang melintasi sesuatu medium ialah
1.6 × 108 m s–1. Kirakan indeks pembiasan medium Halaju cahaya di udara, c
tersebut. / The speed of light passing through a medium is Speed of light in air, c
8 –1
1.6 × 10 m s . Calculate the refractive index of the n =
Halaju cahaya dalam medium, v
medium. Speed of light in medium, v

Penyelesaian / Solution:
8 –1
Halaju dalam vakum / Speed in vacuum, 2.2 = 3.0 × 10 m s
c = 3.0 × 108 m s–1 v
Halaju dalam medium / Speed in medium,
v = 1.6 × 108 m s–1 3.0 × 108 m s–1
v = = 1.36 × 108 m s–1
Halaju cahaya di udara, c 2.2
Speed of light in air, c
n =
Halaju cahaya dalam medium, v
Speed of light in medium, v U
N
I
8 –1 T
n = 3.0 × 10 m s
1.6 × 108 m s–1
= 1.88
6

161 © Nilam Publication Sdn. Bhd.

B6FizikF4(155-187)csy5p.indd 161 09/01/2020 9:56 AM


MODUL • Fizik TINGKATAN 4

4 Rajah di bawah menunjukkan seekor ikan di 5 Dalam rajah di bawah, lukiskan rajah sinar dari
bahagian bawah kolam berada pada kedudukan titik Z ke mata pemerhati untuk menunjukkan objek
50 cm dari permukaan air. Berapakah dalam ketara kelihatan lebih tinggi.
ikan itu? In the diagram below, draw a ray diagram from point
The diagram below shows a fish is at the bottom of pond Z to the eye's observer to show how the object appears
50 cm from the water surface. What is the apparent higher up.
depth of the fish? Pemerhati
(Indeks biasan air / Refractive index of water = 1.33) Observer

N N
Udara
Air
Air
Water h

I 50 cm N

Imej
O Image

Z
Objek / Object

Penyelesaian / Solution:

n = Dalam nyata/Real depth = H


Dalam ketara/Apparent depth h
0.5 m

1.33 =
h
0.5 m
h =
1.33
= 0.38 m

6.2 PANTULAN DALAM PENUH / TOTAL INTERNAL REFLECTION SK 6.2

SP 6.2.1 Menerangkan sudut genting dan pantulan dalam penuh

Fenomena yang berlaku hanya apabila cahaya merambat dari medium


Berikan definisi bagi
lutsinar yang lebih tumpat ke medium lutsinar yang kurang tumpat.
pantulan dalam penuh.
A phenomenon which occurs only when light travels from a denser transparent
Define total internal reflection.
medium to a less dense transparent medium.

Sudut genting, c, ialah sudut tuju dalam medium yang lebih tumpat
apabila sudut biasan, r, dalam medium yang kurang tumpat ialah sama
Apakah itu sudut genting, c?
What is a critical angle, c? dengan 90º.
The critical angle, c, is the angle of incidence in the denser medium when the
angle of refraction, r, in the less dense medium is equal to 90º.
U
N
I
T

© Nilam Publication Sdn. Bhd. 162

B6FizikF4(155-187)csy5p.indd 162 09/01/2020 9:56 AM


MODUL • Fizik TINGKATAN 4

Jadual menunjukkan penerangan tentang perambatan sinar cahaya dari air (medium lebih tumpat) ke udara
(medium kurang tumpat) dalam sinar tuju, i yang berbeza.
The table below show explanation on light rays travel from water (denser medium) to air (less dense medium) in different
angles of incidence, i.

Kes/Case: i < c (i) Sinar cahaya dibiaskan menjauhi normal.


Sinar Light is refracted away from normal.
biasan
r Refracted (ii) Sebahagian kecil sinar cahaya dipantulkan ke dalam medium yang
Udara / Air ray tumpat.
Air / Water Small portion of light is reflected to the denser medium.
i
(iii) Sudut genting, c > sudut tuju, i
Sinar tuju / Incident ray Critical angle, c > incident angle, i

Kes/Case: i = c (i) Sudut tuju, i, bertambah sehingga sinar biasan betul-betul berada di
atas sempadan pembahagi udara-air, iaitu sudut biasan menjadi,
r Sinar biasan r = 90º.
Refracted ray
Udara / Air The angle of incidence, i, increases where the refracted ray is exactly on the
Air / Water boundary separating air-water, the angle of refraction, becomes r = 90º.
i=c (ii) Sudut tuju, i, yang sepadan pada ketika ini dikenali sebagai sudut
Sinar tuju Sinar pantulan genting, (i = c).
Incident ray Reflected ray The corresponding angle of incidence at this moment is known as critical
angle, (i = c).
(iii) Sinar pantulan menjadi lebih jelas. / Reflected ray becomes clearer.
Kes/Case: i > c (i) Apabila sudut tuju, i, dalam air meningkat sehingga melebihi sudut
genting, c, sinar cahaya akan dipantulkan sepenuhnya ke dalam air.
When the angle of incidence, i, in water increases until it exceeds the critical
Udara / Air
angle, c, light will be reflected totally into water.
Air / Water
i (ii) Fenomena ini dikenali sebagai pantulan dalam penuh.
This phenomenon is known as total internal reflection.
Sinar tuju Sinar pantulan
Incident ray Reflected ray

Nyatakan dua syarat untuk Dua syarat untuk pantulan dalam penuh berlaku ialah:
pantulan dalam penuh Two conditions for total internal reflection to occur are:
berlaku. (a) Cahaya merambat dari medium yang lebih tumpat ke medium kurang
State two conditions for total tumpat. / The light travels from a denser medium to a less dense medium.
internal reflection to occur.
(b) Sudut tuju, i, adalah lebih besar daripada sudut genting, c (i > c).
The angle of incidence, i, is greater than the critical angle, c (i > c).

1
SP 6.2.2 Menghubung kait sudut genting dengan indeks biasan, n = sin c

sin i (dalam udara / in air)


Udara n=
Air sin r (dalam kaca / in glass)

Kaca sin 90°


n=
Glass sin c
c U
1 N
n= I
sin c T

163 © Nilam Publication Sdn. Bhd.

B6FizikF4(155-187)csy5p.indd 163 09/01/2020 9:56 AM


MODUL • Fizik TINGKATAN 4

Contoh / Examples

1 Kira sudut genting kaca yang mempunyai indeks 2 Rajah di bawah menunjukkan satu sinar cahaya, P,
pembiasan, n = 1.5. ditujukan ke dalam blok kaca. Sudut genting kaca
Calculate the critical angle for glass with refractive ialah 42°. Ke arah manakah cahaya bergerak dari
index, n = 1.5. titik Q?
Penyelesaian / Solution: The diagram below shows a light ray, P, directed into
the glass block. The critical angle of the glass is 42°. In
1 which direction does the light move from point Q?
= sin c
n
A B
1 C
1.5 = sin c
sin c = 0.6667
Q D
c = 41.8°
Blok kaca
42˚ Glass block

Penerangan: Sudut tuju = sudut genting = 42°, maka cahaya


merambat di sepanjang sempadan kaca-udara.
Explanation: Incident angle = critical angle = 42°, thus
light travels along to the boundary of glass-air.

SP 6.2.3 Berkomunikasi untuk menerangkan fenomena semula jadi dan aplikasi pantulan dalam penuh dalam kehidupan harian.

Aplikasi Pantulan dalam Penuh dalam Kehidupan Harian


Applications of Total Internal Reflection in Daily Life

1 Pembentukan pelangi / Formation of rainbow

• Pembentukan pelangi ialah fenomena yang terhasil


daripada pembiasan, penyebaran dan pantulan dalam
penuh. Cahaya
The formation of rainbow is a phenomenon caused by matahari
Sunlight
refraction, dispersion and total internal reflection.
• Cahaya matahari yang menembusi titisan air hujan akan
terbias kali pertama dan tersebar pada warna-warna yang
berbeza.
Sunlight that passes through a raindrop undergoes first
refraction and dispense into different colours.
• Pantulan dalam penuh akan berlaku pada permukaan
titisan air hujan lalu terbias dan tersebar kali kedua
apabila keluar ke medium udara. Pemerhati
Observer
Total internal reflection occurs on the surface of raindrop thus
undergoes second refraction and dispersion when it goes out
into the medium of air.
U
N
• Hasilnya, pelangi akan terbentuk dalam bentuk
I melengkung.
T
As a result, a curved-shape rainbow is occur.
6

© Nilam Publication Sdn. Bhd. 164

B6FizikF4(155-187)csy5p.indd 164 09/01/2020 9:56 AM


MODUL • Fizik TINGKATAN 4

2 Logamaya / Mirage

(a) Lapisan udara di atas: Lebih sejuk . / Upper layer of air: Cooler .
(b) Ketumpatan optik: Lebih tumpat . / Optical density : Denser .
(c) Lapisan udara di permukaan: Lebih panas . / Layer of air on the surface: Hotter .
(d) Ketumpatan optik: Kurang tumpat . / Optical density: less dense .

Udara sejuk (lebih tumpat)


Cool air (denser)
Udara panas (kurang tumpat)
Warm air (less dense)
Permukaan
panas
Hot surface
Imej
Image

• Cahaya yang merambat dari lapisan atas akan terbias ke lapisan bawah beransur-ansur menjauhi normal
sehingga sudut tuju lebih besar daripada sudut genting.
Light that travels from upper layer to lower layer will gradually refracted away from normal until the incident
angle is bigger than the critical angle.
• Pantulan dalam penuh berlaku di permukaan tanah.
Total internal reflection
occurs on the surface.
• Pemerhati melihat imej sebagai lopak air pada permukaan.
The observer see the image as a puddle on the surface.

3 Periskop berprisma / Prism periscope

• Untuk melihat objek di sebalik penghalang. Dalam kapal selam,


periskop digunakan untuk memerhatikan kapal-kapal pada 45°
permukaan laut. Objek
Object
To view objects behind obstacles. In submarines, periscopes are used to
observe ships on the surface of the sea.
• Terdiri daripada dua buah prisma bersudut tegak yang dipasang
pada hujung tiub.
Consist of two right angle prisms fitted at both ends of long tube. 45°
Imej
Image
• Sinar cahaya dari objek akan memasuki dan melalui prisma pertama
tanpa pembiasan. Sudut tuju, 45° lebih besar daripada sudut genting
prisma, iaitu 42°. Pantulan dalam penuh berlaku dan cahaya akan terpantul ke bawah.
Light ray from the object will enter and pass through the first prism without refraction. The angle of incidence
which is 45° is larger than the critical angle, 42°. Total internal reflection occur and light will be reflected
downwards.
• Apabila sinar cahaya melalui prisma kedua, sekali lagi pantulan dalam penuh berlaku dan cahaya keluar U
N
tanpa pembiasan ke mata pemerhati. Imej yang terhasil adalah tegak dan sama saiz. I
T
When the light ray pass through the second prism, again total internal reflection happen and light ray will enter
the eye of the observer. The image formed is upright and same size as the object. 6

165 © Nilam Publication Sdn. Bhd.

B6FizikF4(155-187)csy5p.indd 165 09/01/2020 9:56 AM


MODUL • Fizik TINGKATAN 4

4 Gentian optik / Optical fibre

• Suatu gentian optik terdiri daripada teras dalam yang


mempunyai indeks biasan yang tinggi dan dikelilingi oleh
penyalut yang mempunyai indeks biasan yang rendah. Lapisan luar
Outer cladding
An optical fibre consists of inner core of high refractive index Normal
and surrounded by layer of cladding of lower refractive index. Normal
• Apabila sinar cahaya merambat ke dalam teras dalam pada
satu hujungnya, pantulan dalam penuh berlaku berturut-
turut sehingga ke hujung yang lagi satu. Teras dalam
When a light ray moves into the inner core from one end, total Inner core
internal reflections occur continuously until it reach the other Sinar cahaya
Light ray
end.
Perambatan cahaya dalam gentian optik
Kelebihan menggunakan kabel gentian optik: Propagation of light in an optical fibre
The advantages of using optical fibre cable:
• Lebih nipis dan lebih ringan.
Much thinner and lighter.
• Membawa lebih banyak maklumat (isyarat) dengan pantas.
Carry more information (signal) quickly.
• Tiada gangguan daripada medan elektromagnet dan menghasilkan sambungan yang lebih jelas.
No interference from electromagnet field and result in clearer connections.
• Tiada rintangan elektrik.
No electrical resistance.
• Tidak berlakunya kejutan elektrik jika kabel terputus.
No electric shock happen if the cable breaks.

5 Pemantul ‘Cat’s eye’ / Cat’s eye reflector

• Merupakan peranti keselamatan untuk penanda bagi pengguna jalan raya pada waktu malam.
A Cat’s eye reflector is a safety device as a marker for road users at night.
• Sfera berpantul diletakkan di dalam pemantul.
Reflective spheres are set into the reflector.
Cahaya masuk
Light enter Pantulan dalam penuh
Total internal reflection occurs

Cahaya keluar
Light exit

• Apabila cahaya daripada lampu kereta merambat pada pemantul mata jalan, prisma di dalam alat
tersebut akan memantul kembali kepada mata pengguna jalan raya.
When the light ray from the headlight enter the reflector, the prism inside the device will reflect back to the eye of
the road users.

U
N
I
T

© Nilam Publication Sdn. Bhd. 166

B6FizikF4(155-187)csy5p.indd 166 09/01/2020 9:56 AM


MODUL • Fizik TINGKATAN 4

SP 6.2.4 Menyelesaikan masalah yang melibatkan pantulan dalam penuh

Latihan / Exercises
1 Rajah di bawah menunjukkan lintasan sinar cahaya yang tidak lengkap memasuki suatu prisma kaca. Indeks
biasan kaca ialah 1.47.
The diagram below shows an incomplete light ray path entering a glass prism. The refractive index of glass is 1.47.

45°

Prisma kaca 45°


Glass prism

Sinar cahaya / Light ray


(a) Hitung sudut genting prisma kaca itu. / Calculate the critical angle of the glass prism.
1
n =
sin c
1
1.47 = sin c
sin c = 0.6803
c = 42.86o

(b) Pada rajah di atas, lengkapkan lintasan sinar cahaya dari titik O.
On the diagram above, complete the light ray path from point O.

(c) Berdasarkan jawapan di (b), namakan fenomena cahaya yang terlibat.


Based on the answer in (b), name the light phenomenon involved.
Pantulan dalam penuh. / Total internal reflection.

2 Intan mempunyai indeks biasan, n = 2.4. Apakah sudut genting bagi intan?
Diamond has a refractive index, n = 2.4. What is the critical angle of diamond?
1
n =
sin c
1
2.4 = sin c
c = 24.6o

3 Rajah di bawah menunjukkan laluan sinar cahaya pada satu bongkah kaca. Apakah indeks biasan bagi kaca
tersebut? / The diagram below shows a ray of light in a glass slab. What is the refractive index of the glass?
55o

1
n = sin c U
N
I
1 T
= sin 55o
6
= 1.22

167 © Nilam Publication Sdn. Bhd.

B6FizikF4(155-187)csy5p.indd 167 09/01/2020 9:56 AM


MODUL • Fizik TINGKATAN 4

6.3 PEMBENTUKAN IMEJ OLEH KANTA / IMAGE FORMATION BY LENSES SK 6.3

SP 6.3.1 Mengenal pasti kanta cembung sebagai kanta penumpu dan kanta cekung sebagai kanta pencapah

Apakah itu kanta cembung? Paksi kanta / Axis of lens


What is convex lens?

Kanta cembung = kanta penumpu =


kanta positif F
Convex lens = converging lens
Paksi utama F
= positive lens Principal axis
Kanta cembung adalah lebih tebal di
tengah-tengah daripada di sisinya.
Convex lenses are thicker in the
centre than at the edge.
f f

• Sinar cahaya yang selari dengan paksi utama dibiaskan untuk


menumpu di titik fokus.
Light rays parallel to principal axis are refracted to converge to the
focal point.

Apakah itu kanta cekung? Paksi kanta / Axis of lens


What is concave lens?

Kanta cekung = kanta pencapah =


kanta negatif F
Concave lens = diverging
F Paksi utama
lens = negative lens Principal axis
Kanta cekung adalah lebih nipis di
tengah-tengah daripada di sisinya.
Concave lenses are thinner in the
centre than at the edge. f f

• Sinar cahaya yang selari dengan paksi utama dibiaskan untuk


mencapah dari titik fokus.
Light rays parallel to the principal axis are refracted to diverge
from the focal point.

Pusat optik, O Titik pada pusat kanta. Sinar yang merambat melalui pusat optik dalam garis
Optical centre, O lurus.
The geometric centre of the lens. Light rays passed through the optical centre in a
straight line.

Paksi utama Garis lurus yang melalui pusat optik suatu kanta dan bersambung dengan
Principal axis pusat-pusat kelengkungan dua permukaan kanta itu.
Straight line which passes through the optical centre of a lens and centre of curvature
of both surfaces of the lens.

Paksi kanta Garis lurus yang menerusi pusat optik dan berserenjang dengan paksi utama.
Axis of lens Straight line along the optical centre and perpendicular to principal axis.

Jarak objek, u Jarak di antara pusat optik, O, dengan objek.


U Object distance, u Distance between the optical centre, O, to the object.
N
I
T Jarak imej, v Jarak antara pusat optik, O, dengan imej.
6 Image distance, v Distance between the optical centre, O, to the image.

© Nilam Publication Sdn. Bhd. 168

B6FizikF4(155-187)csy5p.indd 168 09/01/2020 9:56 AM


MODUL • Fizik TINGKATAN 4

Titik fokus, F Titik pada paksi utama kanta.


Focal point, F Point located at the principal axis of lens.
• Untuk kanta cembung, sinar cahaya yang selari dengan paksi utama
akan menumpu pada titik ini melalui kanta.
For convex lens, light rays parallel to the principal axis will converge at this
point after passing through the lens
• Untuk kanta cekung, sinar cahaya yang selari dengan paksi utama akan
mencapah dari titik ini selepas melalui kanta.
For concave lens, light rays parallel to the principal axis appear to diverge
from this point after passing through the lens.

Panjang fokus, f Jarak di antara titik fokus, F dan pusat optik kanta.
Focal length, f The distance between the focal point, F and optical centre of the lens.

SP 6.3.2 Menganggar panjang fokus bagi suatu kanta cembung menggunakan objek jauh

Aktiviti : Menganggar panjang fokus bagi suatu kanta cembung


Activity : Estimate the focal length of a convex lens

Kanta cembung / Convex lens

Skrin putih
White screen
Imej tajam
Sharp image
Pemegang kanta
Lens holder

Objek jauh
(di luar makmal) Pembaris
Distant object setengah meter 50 40 30 20 10 0
50 40 30 20 10 0 cm
(outside the laboratory) Half meter rule cm
f
f
Arahan / Instructions:
1 Radas disusun seperti yang ditunjukkan dalam rajah.
The apparatus is set up as shown in the diagram.
2 Kanta cembung diletakkan pada pemegang kanta dan dihalakan pada tingkap atau ruang yang terbuka.
Convex lens is placed on the lens holder and towards an open window or area.
3 Dengan menggunakan objek di luar (contoh: pokok) sebagai objek jauh, kedudukan skrip putih diubah
sehingga imej yang jelas terbentuk pada skrin.
By using an object outside of the building (example: tree) as a distant object, the position of the white screen is adjusted
until a clear image is formed on the screen.
4 Panjang fokus kanta, iaitu jarak antara pusat optik kanta dengan skrin diukur dan direkod.
Focal length of the lens, which is the distance between the optical centre of the lens and the screen is measured and
recorded.

Keputusan / Results:
U
Kedudukan kanta pada pembaris, / Position of the lens on metre rule, a = N
I
Kedudukan skrin pada pembaris, / Position of the screen on lens, b = T

Panjang fokus kanta, / Length of focal length, f = b – a = 6

169 © Nilam Publication Sdn. Bhd.

B6FizikF4(155-187)csy5p.indd 169 09/01/2020 9:56 AM


MODUL • Fizik TINGKATAN 4

SP 6.3.3 Menentukan kedudukan imej dan ciri-ciri imej yang dibentuk oleh: (i) kanta cembung (ii) kanta cekung

Cara Melukis Rajah Sinar bagi Kanta Cembung dan Kanta Cekung
Way to Draw a Ray Diagram for Convex Lens and Concave Lens

Kanta cembung / Convex lens

(a) (b) (c)

Objek O Objek O F O F F O FF O F F O
Object Object

1 Sinar cahaya melalui pusat optik, O kanta secara lurus tanpa terbias.
A ray of light passes through the optical centre, O of the lens without being refracted.
2 Sinar cahaya bergerak selari dengan paksi utama akan terbias dan melalui titik fokus, F.
A ray of light moves parallel to the principal axis is refracted and passes through the focal point, F.
3 Sinar cahaya yang melalui titik fokus, F akan terbias dan bergerak selari dengan paksi utama.
A ray of light that passes through the focal point, F is refracted and moves parallel to the principal axis.
4 Mana-mana titik pertemuan antara sinar cahaya akan menunjukkan kedudukan dan ciri-ciri imej.
Any intersection point between the rays of lights will shows the position and characteristics of the image.

Kanta cekung / Concave lens

(a) (b) (c)

O Objek O
Object F O F F O FF O F F O F

1 Sinar cahaya melalui pusat optik, O kanta secara lurus tanpa terbias.
A ray of light passes through the optical centre, O of the lens without being refracted.
2 Sinar cahaya bergerak selari dengan paksi utama akan terbias dan muncul daripada titik fokus yang sama
sisi dengan kanta.
A ray of light moves parallel to the principal axis is refracted and passes through the focal point, F on the
same side of the lens.
3 Sinar cahaya yang melalui titik fokus, F di sisi lain sebelah kanta akan terbias dan bergerak selari dengan
paksi utama.
A ray of light that travels towards the focal point, F on the other side of the lens is refracted and moves
parallel to the principal axis.
4 Mana-mana titik pertemuan antara sinar cahaya akan menunjukkan kedudukan dan ciri-ciri imej.
Any intersection point between the rays of lights will shows the position and characteristics of the image.

U
N
I
T

© Nilam Publication Sdn. Bhd. 170

B6FizikF4(155-187)csy5p.indd 170 09/01/2020 9:56 AM


MODUL • Fizik TINGKATAN 4

1 Rajah sinar untuk kanta cembung / Ray diagrams for convex lens

Kedudukan objek Rajah sinar Ciri-ciri imej


Object position Ray diagram Characteristics of image

(a) Objek, O di infiniti


Object, O at infinity • Songsang / Inverted
F F • Nyata / Real
• Diperkecil / Diminished

(b) Objek, O lebih jauh


daripada 2F O
Object, O beyond 2F • Songsang / Inverted
(u > 2f ) 2F F 2F
• Nyata / Real
F
I • Diperkecil / Diminished
u v
O

2F F F 2F
(c) Objek, O pada 2F • Songsang / Inverted
Object, O at 2F O
(u = 2f ) I • Nyata / Real
2F F F
• Sama saiz dengan objek
u v Same size as the object

F F
(d) Objek, O di antara F dan 2F
Object, O between F and 2F O • Songsang / Inverted
(f < u < 2f ) I
• Nyata / Real
2F F F 2F

u v • Diperbesar / Magnified

(e) Objek, O pada F


• Tegak / Upright
Object, O at F O
(u = f ) • Maya / Virtual
2F F F 2F

• Diperbesar / Magnified

(f) Objek, O di antara F dan I


pusat optik
Object, O between F and • Tegak / Upright
optical centre
(u < f ) • Maya / Virtual
2F FO F 2F
• Diperbesar / Magnified
v u U
N
I
T

171 © Nilam Publication Sdn. Bhd.

B6FizikF4(155-187)csy5p.indd 171 09/01/2020 9:56 AM


MODUL • Fizik TINGKATAN 4

2 Rajah sinar untuk kanta cekung / Ray diagrams for concave lens

(a) Objek, O lebih jauh daripada 2F (u > 2f ) ( b) Objek, O di antara F dan 2F (f < u < 2f )
Object, O beyond 2F (u > 2f) Object, O between F and 2F (f < u < 2f)

O O

2F F I F 2F
2F F I F 2F

Ciri-ciri imej: / Characteristics of image:


Diperkecil, tegak, maya / Diminished, upright, virtual

v
SP 6.3.4 Menyatakan pembesaran linear, m sebagai: m =
u

Pembesaran linear, m / Linear magnification, m

Maksud / Meaning
– Nisbah tinggi imej terhadap tinggi objek. / The ratio of image height to object height.
– Nisbah jarak imej terhadap jarak objek. / The ratio of image distance to object distance.

Pembesaran linear, m = Tinggi imej / Image height = hi   Jarak imej / Image distance v
atau / or   =
Linear magnification, Tinggi objek / Object height ho Jarak objek / Object distance u
hi v
m = =
ho u
* Tiada unit. / No unit.
* Sentiasa bernilai positif. / Always in positive value.

Latihan / Exercise
1 Rajah di sebelah menunjukkan kedudukan imej, I, bagi objek, 100 cm
O yang terhasil oleh kanta cembung. Ketinggian objek, ho
ialah 6 cm. Berapakah tinggi imej, hi? I
The diagram on the right shows the position of an image, I, for O
object, O formed by a convex lens. The height of the object, ho is
6 cm. What is the image height, hi? 70 cm

Penyelesaian / Solution:
Jarak imej / Image distance, v = 70 cm
Jarak objek / Object distance, u = 100 cm – 70 cm = 30 cm
ho = 6 cm
h v
m = i =
ho u
h 70 cm
U i = 30 cm
N 6 cm
I
70 cm
T hi = × 6 cm
30 cm
6
= 14 cm

© Nilam Publication Sdn. Bhd. 172

B6FizikF4(155-187)csy5p.indd 172 09/01/2020 9:56 AM


MODUL • Fizik TINGKATAN 4

6.4 FORMULA KANTA NIPIS / THIN LENS FORMULA SK 6.4

Mengeksperimen untuk:
(i) mengkaji hubungan jarak objek, u dan jarak imej, v bagi satu kanta cembung
SP 6.4.1 (ii) menentukan panjang fokus kanta nipis dengan menggunakan
1 1 1
Formula Kanta: = +
f u v

Eksperimen Hubungan antara jarak objek dan jarak imej


Experiment The relationship between object distance and image distance

Inferens Jarak imej dipengaruhi oleh jarak objek.


Inference The image distance is influenced by the object distance.

Hipotesis Apabila jarak objek, u berkurang, jarak imej, v bertambah.


Hypothesis When the object distance, u decreases, the image distance, v increases.

Untuk mengkaji hubungan antara jarak objek dan jarak imej bagi kanta cembung.
Tujuan
To investigate the relationship between the object distance and the image distance for a convex
Aim
lens.

1 Pemboleh ubah dimanipulasikan:


Manipulated variable:
Jarak objek, u. / Object distance, u.

2 Pemboleh ubah bergerak balas:


Pemboleh ubah Responding variable:
Variables
Jarak imej, v. / Image distance, v.

3 Pemboleh ubah yang dimalarkan:


Constant variable:
Panjang fokus kanta, f. / Focal length of the lens, f.

Senarai radas Kanta cembung, skrin putih, pemegang kanta, pembaris meter, mentol, bekalan kuasa,
dan bahan kadbod dengan lubang kecil berbentuk segi tiga, blok kayu.
List of apparatus Convex lens, white screen, lens holder, metre rule, bulb, power supply, cardboard with a small
and materials triangular shaped hole, wooden block.

Kanta cembung
Convex lens
Skrin putih
White screen
Kadbod
Cardboard
Susunan radas Pembaris meter
Arrangement of the Metre rule
apparatus Ke bekalan kuasa
To power supply
v
u
U
N
I
T

173 © Nilam Publication Sdn. Bhd.

B6FizikF4(155-187)csy5p.indd 173 09/01/2020 9:56 AM


MODUL • Fizik TINGKATAN 4

1 Radas disusun seperti dalam rajah.


The apparatus is set up as shown in the diagram.
2 Jarak objek ditentukan pada 15 cm dan mentol dihidupkan.
The object distance is set up at 15 cm and bulb is switched on.
3 Skrin putih dilaraskan untuk mendapatkan imej yang tajam (jelas).
Prosedur
The screen is adjusted to obtain a sharp image.
Procedure
4 Jarak imej diukur menggunakan pembaris dan direkodkan.
The image distance is measured using the ruler and recorded.
5 Langkah 2 hingga 4 diulang dengan jarak objek, u = 20.0 cm, 25.0 cm, 30.0 cm dan
35.0 cm.
Steps 2 to 4 are repeated with object distance, u = 20.0 cm, 25.0 cm, 30.0 cm and 35.0 cm.

1 1
u (cm) v (cm) –1 (cm–1)
Keputusan u (cm ) v
Results
15.0
20.0
25.0
30.0
35.0

1 1
1 Graf v melawan u diplot dengan menggunakan kertas graf.
1 1
Graph of against is plotted by using a graph paper.
v u

2 Kecerunan graf, m ditentukan. (tunjukkan pengiraan)


Gradient of the graph, m is determined. (show the calculation)
Analisis data
Analysis of data 3 Dengan menggunakan formula kanta dan dari graf yang diplot, nilai panjang fokus
kanta adalah .
By using the lens formula and from the graph plotted, the focal length of the lens is
.

4 Adakah hipotesis diterima?


Is the hypothesis accepted?

Kesimpulan Apabila jarak objek, u berkurang, jarak imej, v bertambah.


Conclusion When the object distance, u decreases, the image distance, v increases.

U
N
I
T

© Nilam Publication Sdn. Bhd. 174

B6FizikF4(155-187)csy5p.indd 174 09/01/2020 9:56 AM


MODUL • Fizik TINGKATAN 4

Hubungan antara u, v dan f / Relationship between u, v and f

1 1 1
= +
f u v

Jenis kanta Kanta cembung Kanta cekung


Type of lens Convex lens Concave lens
Panjang fokus, f Positif (+) Negatif (–)
Focal length, f Positive (+) Negative (–)
Jarak imej, v • Imej nyata / Real image • Imej maya / Virtual image
Image distance, v • Di sebelah kanta yang bertentangan • Di sebelah kanta yang sama dengan
objek objek
On the opposite side of object
On the same side of object

SP 6.4.2 Menyelesaikan masalah yang melibatkan formula kanta nipis bagi kanta cembung dan kanta cekung

Latihan / Exercises
1 Satu objek dengan tinggi 8 cm diletakkan pada 2 Satu sumber cahaya diletakkan di sebelah kiri kanta
jarak 20 cm dari kanta cekung. Panjang fokus ialah pada panjang fokus 12 cm. Imej yang terbentuk
10 cm. Berapakah dibesarkan enam kali di sebelah kanan kanta.
An object height that is 8 cm is placed at a distance of A light source is placed on the left side of the lens of
20 cm from a concave lens. Its focal length is 10 cm. focal length 12 cm. The image formed is magnified six
What is the times on the right side of the lens.
(a) jarak imej? (a) Apakah jenis kanta yang digunakan?
image distance? What is the type of lens used?
(b) saiz imej? (b) Apakah jarak objek tersebut?
image size? What is the object distance?
Penyelesaian / Solution: Penyelesaian / Solution:
(a) ho = 8 cm (a) Kanta cembung kerana imej yang terhasil
u = +20 cm berada bertentangan dengan kedudukan kanta
f = –10 cm dan objek. Imej yang terbentuk adalah imej
1 1 1 nyata.
= + Convex lens because the image formed is at the
f u v
opposite side of the lens and object. The image
1 1
= + 1 formed is a real image.
–10 cm 20 cm v v
v = –6.7 cm (b) m = u
hi v v
(b)
= 6 = u
ho u
6.7 cm v = 6u
hi = × 8 cm
20 cm
1 1 1
= 2.68 cm u + v = f
1 1 1
u + 6u = 12
7 1 U
6u = 12 N
I
u = 14 cm T

175 © Nilam Publication Sdn. Bhd.

B6FizikF4(155-187)csy5p.indd 175 09/01/2020 9:56 AM


MODUL • Fizik TINGKATAN 4

6.5 PERALATAN OPTIK / OPTICAL INSTRUMENTS SK 6.5

SP 6.5.1 Mewajarkan penggunaan kanta dalam peralatan optik iaitu kanta pembesar, mikroskop majmuk dan teleskop

1 Kanta pembesar / Magnifying glass


• Jarak objek, u di antara F dan pusat optik (u < f )
Object distance, u between F and optical centre (u < f)

Ciri-ciri imej:
I
The characteristics of the image:
Diperbesar, tegak, maya
Magnified, upright, virtual F O F

Mata
Eye

2 Mikroskop majmuk / Compound microscope


• Mikroskop majmuk digunakan untuk melihat objek yang sangat kecil dan halus.
A compound microscope is used to see a very small and fine objects.
• Terdiri daripada dua kanta cembung:
Consists of two convex lens:
(a) Kanta mata dengan panjang fokus, fe
Eyepiece with focal length, fe
(b) Kanta objek dengan panjang fokus yang pendek, fo
Objective lens with shorter focal length, fo

Kanta mata
Eyepiece

Kanta objek
Objective lens
Objek
Object
Fo = titik fokus kanta objek
Pemerhati
Fo Fo Fe I1 Fe Observer focal point of objective lens
Fe = titik fokus kanta mata
focal point of eyepiece
I2

Ciri-ciri imej pertama, I1


Characteristics of first image, I1
Songsang, nyata, diperbesar
Ciri-ciri imej terakhir, I2
Characteristics of final image, I2 Inverted, real, magnified
Songsang, maya, diperbesar
Inverted, virtual, magnified
U
N
I
T

© Nilam Publication Sdn. Bhd. 176

B6FizikF4(155-187)csy5p.indd 176 09/01/2020 9:56 AM


MODUL • Fizik TINGKATAN 4

3 Teleskop / Telescope
• Teleskop ialah alat optik untuk melihat objek yang sangat jauh seperti bintang dan
planet.
Telescope is an optical instrument that is used to see very far objects such
as stars and planets.
• Terdiri daripada dua kanta cembung:
Consists of two convex lens:
(a) Kanta mata dengan panjang fokus yang pendek, fo.
Eyepiece with shorter focal length, fo.
(b) Kanta objek dengan panjang fokus yang panjang, fe.
Objective lens with longer focal length, fe.
Kanta mata
Eyepiece lens

Kanta objek
Objective lens
fo Fo, Fe fe
Pemerhati Fo = titik fokus kanta objek
Cahaya selari dari Observer focal point of objective lens
objek jauh Fe = titik fokus kanta mata
Parallel rays focal point of eyepiece
from distant
object

I1
Imej, I2, di infiniti
Image, I2, is formed
at infinity

Ciri-ciri imej terakhir, I2 Ciri-ciri imej pertama, I1


Characteristics of final image, I2 Characteristics of first image, I1
Maya, songsang, diperbesar Nyata, songsang, diperkecil
Virtual, inverted, magnified Real, inverted, diminished

SP 6.5.2 Mereka bentuk dan membina mikroskop majmuk dan teleskop

Aktiviti : Mereka bentuk mikroskop majmuk dan teleskop


Activity : Design a compound microscope and telescope

Lakukan aktiviti ini secara berkumpulan, kumpul maklumat mengenai mikroskop majmuk dan teleskop berdasarkan
perkara berikut:
Do this activity in group, collect information about compound microscope and telescope based on the following criteria:

• Jenis dan fungsi kanta yang digunakan. / Type of lens and its function.
• Kriteria pemilihan kanta objek dan kanta mata. / Criteria in the selection of objective lens and eyepiece lens.
• Lukisan gambar rajah sinar bagi pembentukan imej yang terbentuk.
Draw ray diagrams for the images formed.

U
Kemudian, reka, lakar dan bina satu bentuk mikroskop majmuk dan teleskop mengikut ciri-ciri yang dikehendaki. N
I
Bentangkan rekaan mikroskop majmuk dan teleskop kumpulan anda di dalam kelas. T
Then, design, sketch and build a form of compound microscope and telescope according to the desired characteristics. Present
the design of the compound microscope and telescope in the classroom. 6

177 © Nilam Publication Sdn. Bhd.

B6FizikF4(155-187)csy5p.indd 177 09/01/2020 9:56 AM


MODUL • Fizik TINGKATAN 4

Perbandingan antara mikroskop majmuk dan teleskop


Comparisons between a compound microscope and a telescope

Persamaan / Similarities
1 Terdiri daripada dua kanta cembung
Consists of two convex lenses
2 Imej yang pertama adalah nyata dan songsang serta bertindak sebagai objek bagi kanta mata
The first image is real and inverted and acts as the object for the eyepiece
3 Kanta mata bertindak sebagai kanta pembesar
The eyepiece acts as a magnifying lens
4 Imej akhir adalah maya, songsang dan diperbesar
The final image is virtual, inverted and magnified

Perbezaan / Difference

Mikroskop majmuk Aspek Teleskop


Compound microscope Aspects Telescope

Kanta cembung berkuasa


Jenis kanta dan kuasa kanta rendah dan kanta cembung
Dua kanta cembung berkuasa tinggi
Types of lenses and power of berkuasa tinggi
Two high-powered convex lenses
lenses A low powered convex lens and a
high powered convex lens

Panjang fokus
fo < fe fo > fe
Focal length

Imej pertama adalah besar Imej pertama Imej pertama adalah kecil
First image is magnified First image First image is diminished

Kedudukan dekat dengan


Kedudukan imej terakhir Pada kedudukan infiniti
mata pemerhati
Position of final image At infinity
At the near point of the observer’s eye

Jarak antara dua kanta


D > (fo + fe) Distance between lenses D = (fo + fe)
(D)

Pembesaran linear, m fo
m = mo × me m=
Linear magnification, m
fe

U
N
I
T

© Nilam Publication Sdn. Bhd. 178

B6FizikF4(155-187)csy5p.indd 178 09/01/2020 9:56 AM


MODUL • Fizik TINGKATAN 4

SP 6.5.3 Berkomunikasi untuk menerangkan aplikasi kanta bersaiz kecil dalam teknologi peralatan optik

Aplikasi Kanta Bersaiz Kecil dalam Teknologi Peralatan Optik


Applications of Small Lenses in the Technology of Optical Devices

1 CCTV (Televisyen litar-tertutup) 2 Kamera dalam telefon pintar


CCTV (Closed-circuit television) Cameras in smartphones

Kanta
Lens

Saiz
Medan penglihatan sensor
Vision field Size of
sensor
Panjang
fokus
Jarak objek Focal
Object distance length

(a) Imej yang terbentuk – nyata, songsang dan diperkecil.


Image formed – real, inverted and diminished.
(b) Jarak minimum di antara sensor dengan pusat kanta mestilah sama dengan panjang fokus kanta kamera.
The minimum distance between the sensor and the optical lens must be the same as the focal length of the camera lens.
(c) Contoh ini menerangkan hubungan antara ketebalan telefon pintar dan panjang fokus kanta kameranya:
panjang fokus sebenar sebuah kanta sudut-lebar dalam telefon pintar adalah 4.25 mm. Jadual di bawah
menunjukkan beberapa model telefon pintar yang sebenar dan ketebalannya.
This example explains the relationship between the thickness of a smartphone and the focal length of its camera lens:
the actual focal length of the wide-angle lens of a smartphone is 4.25 mm. The table below shows the thickness of a
few actual models of smartphones.

Ketebalan telefon pintar


Telefon pintar
Thickness of smartphone
Smartphone
(mm)
Model A 9
Model B 8
Model C 6
Model D 7
U
Model E 10.5 N
I
Model F 7 T

Model G 8 6

179 © Nilam Publication Sdn. Bhd.

B6FizikF4(155-187)csy5p.indd 179 09/01/2020 9:56 AM


MODUL • Fizik TINGKATAN 4

6.6 PEMBENTUKAN IMEJ OLEH CERMIN SFERA


IMAGE FORMATION BY SPHERICAL MIRRORS SK 6.6

SP 6.6.1 Menentukan kedudukan imej dan ciri-ciri imej yang dibentuk oleh: (i) cermin cekung (ii) cermin cembung

Garis lurus yang melalui pusat kelengkungan, C dan kutub cermin sfera, P.
Paksi utama
Straight line which passes through the centre of curvature, C and pole of spherical
Principal axis
mirror, P.

Pusat kelengkungan, C Pusat sfera di mana cermin cekung atau cermin cembung terbentuk.
Centre of curvature, C Centre of sphere of which the concave or convex mirror is formed.

Kutub cermin, P Titik tengah pada cermin bulat dalam garis lurus.
Pole of mirror, P The centre point on the spherical mirror.

Titik pada paksi utama:


The point on the principal axis:
Titik fokus, F (a) mencapah daripada titik itu selepas dipantulkan oleh cermin cembung.
Focal point, F diverge from the point after reflected by convex mirror.
(b) menumpu pada titik itu selepas dipantulkan oleh cermin cekung.
converge to the point after reflected by concave mirror.

Jarak objek, u Jarak antara objek dengan kutub cermin sfera, P.


Object distance, u Distance between object and pole of spherical mirror, P.

Jarak imej, v Jarak antara imej dengan kutub cermin sfera, P.


Image distance, v Distance between image and pole of spherical mirror, P.

Panjang fokus, f Jarak di antara titik fokus, F dengan kutub cermin sfera, P.
Focal length, f Distance between focal point, F and pole of spherical mirror, P

Jejari kelengkungan cermin, r Jarak di antara kutub cermin sfera, P dengan pusat kelengkungan, C.
Radius of curvature of mirror, r Distance between the pole of mirror, P with the centre of curvature, C.

Peraturan rajah sinar bagi cermin cekung / Rules of ray diagrams for concave mirror

1 Sinar yang selari dengan paksi utama menumpu pada titik


Titik fokus, F yang dipanggil titik fokus, F pada paksi utama.
Focal point, F Ray parallel to the principal axis converges to a point, called
the focal point, F on the principal axis.
C
P PF = panjang fokus, f / focal length, f
F
Pusat lengkungan, C = jarak antara kutub cermin sfera, P ke titik fokus, F
Centre of curvature, C
distance between pole of spherical mirror, P and focal
point, F
2 Contoh, / Example, f = +10 cm

U
N
I
T

© Nilam Publication Sdn. Bhd. 180

B6FizikF4(155-187)csy5p.indd 180 09/01/2020 9:56 AM


MODUL • Fizik TINGKATAN 4

① ② ③

O C F P O C F P O C F P

Sinar cahaya yang melintasi C akan Sinar cahaya selari dengan paksi Sinar cahaya yang melintasi F akan
terpantul kembali sepanjang cahaya utama akan terpantul melalui F. dipantul selari dengan paksi utama.
tuju (garis lurus). / Light ray passing Light ray parallel to principal axis will Light ray passing through F will be
through C will reflect back along the reflect to F. reflected parallel to the principal axis.
incident ray (straight line).

Ciri-ciri imej pada cermin cekung / Characteristics of image for concave mirror

Jarak objek / Object distance Ciri-ciri imej / Characteristics of image


1 Pada infiniti / At infinity

• Nyata / Real
F
P
• Songsang / Inverted
C I
• Diperkecil / Diminished

2 Lebih jauh dari C ( u > 2f )


Further than C ( u > 2f )
O
• Nyata / Real
• Songsang / Inverted
C I F P • Diperkecil / Diminished

3 Pada C (u = 2f ) / At C (u = 2f)
O
• Songsang / Inverted
• Nyata / Real
C F P
• Sama saiz dengan objek / Same size as the object
I

4 Antara F dan C (f < u < 2f )


Between F and C (f < u < 2f)

O • Songsang / Inverted
• Nyata / Real U
P N
C F • Diperbesar / Magnified I
I T

181 © Nilam Publication Sdn. Bhd.

B6FizikF4(155-187)csy5p.indd 181 09/01/2020 9:56 AM


MODUL • Fizik TINGKATAN 4

5 Pada F (u = f )
At F (u=f)

O • Maya / Virtual
• Tegak / Upright
C F P • Diperbesar / Magnified

6 Antara F dan P (u < f )


Between F and P (u < f)
I
• Tegak / Upright
O • Maya / Virtual
P • Diperbesar / Magnified
C F

Peraturan untuk rajah sinar bagi cermin cembung / Rules for ray diagrams for convex mirror

1 Sinar yang selari dengan paksi utama seolah-olah terpesong dari satu titik di belakang cermin.
Rays parallel to the principal axis appear to diverge from a point behind the mirror.

Cermin cembung
Convex mirror

P F C

2 Titik ini yang terletak pada paksi utama dipanggil titik fokus.
This point which lies on the principal axis is called the focal point.

PF = panjang fokus, f / focal length, f
= jarak antara titik fokus, F dan kutub cermin sfera, P
Distance between focal point, F and the pole of spherical mirror, P

Video
U
N
I
T

6 Kanta Cembung
Convex mirror

© Nilam Publication Sdn. Bhd. 182

B6FizikF4(155-187)csy5p.indd 182 09/01/2020 9:56 AM


MODUL • Fizik TINGKATAN 4

① ② ③

O P F C O P F C O P F C

Sinar cahaya yang melintasi C akan Sinar cahaya selari dengan paksi Sinar cahaya yang melintasi F yang
dipantulkan semula sepanjang sinar utama akan memantul dari F. terpantul adalah selari dengan paksi
cahaya tuju (garis lurus). Light rays parallel to the principal utama.
Light rays passing through, C, is axis will reflect from F. Light rays passing through F which
reflected back along the incident light reflect are parallel to the principal
rays (straight line). axis.

Ciri-ciri cermin cembung / Charateristics of image for convex mirror

Jarak objek Ciri-ciri imej


Object distance Characteristics of image
1 Lebih jauh dari F ( u > f )
Further than F ( u > f )

• Diperkecil / Diminished
• Tegak / Upright

O I
• Maya / Virtual
F P F C

2 Antara F dan P (u < f )


Between F and P (u <f)

• Diperkecil / Diminished
• Tegak / Upright

O I • Maya / Virtual
F P F C

Tip / Tips
✦ Ciri-ciri imej bagi cermin cembung adalah 'DUV'.
Characteristics of the image for convex mirrors is 'DUV'.
D = Diperkecil / Diminished
U
U = Tegak / Upright N
V = Maya / Virtual I
T
Apabila objek mendekati cermin, saiz imej akan membesar.
6

When the object is nearer to the mirror, the size of image increases.

183 © Nilam Publication Sdn. Bhd.

B6FizikF4(155-187)csy5p.indd 183 09/01/2020 9:56 AM


MODUL • Fizik TINGKATAN 4

SP 6.6.2 Berkomunikasi menerangkan aplikasi cermin cekung dan cermin cembung dalam kehidupan

Aplikasi Cermin Cekung / Applications of Concave Mirrors

Cermin pergigian Pemantul / Reflector Cermin solek


Dental mirror ✦ Boleh digunakan dalam lampu suluh dan Cosmetic mirror
✦ Cermin cekung lampu kereta. ✦ Cermin cekung
digunakan dalam cermin Can be used in flashlights and car dengan apertur
peralatan pergigian untuk headlights. yang luas dan jejari
melihat kedudukan gigi ✦ Mentol cahaya ditetapkan pada kelengkungan yang
dengan lebih jelas. kedudukan yang tetap pada titik fokus besar digunakan.
Concave mirrors are used kanta cekung. A wide-aperture
in dental mirrors to see Light bulbs are fixed at fixed positions at concave mirror
the position of teeth more the focal point of concave lens. with large radius of
clearly. ✦ Menghasilkan sinar cahaya yang selari curvature is used.
✦ Imej yang terbentuk supaya boleh mengekalkan keamatan ✦ Imej yang terbentuk
adalah imej yang tegak yang seragam bagi jarak yang lebih adalah imej maya,
dan diperbesar. jauh. tegak dan diperbesar.
The image formed is an Generates parallel light rays to maintain a The image formed is
upright and magnified uniform intensity for further distances. virtual, upright and
image. magnified.
Mentol pada
titik fokus
F
Bulb at focal
point
Cermin parabola
Parabolic mirror

Aplikasi Cermin Cembung / Applications of Convex Mirrors

Cermin pandang belakang Cermin keselamatan dalam Cermin keselamatan jalan


Rear-view mirror bangunan Blind spot mirror
✦ Untuk memberi pandangan Security mirror in buildings ✦ Biasanya dipasang di
belakang yang lebih luas ✦ Biasa digunakan di pasaraya selekoh yang tajam bagi
kepada pemandu kereta. untuk tujuan pengawasan. pemandu untuk melihat
To give a wide-angle view Specially used in the kereta yang akan datang
behind them for drivers. supermarket for surveillance pada sebelah titik buta.
purposes. Usually mounted at sharp
corners for drivers to see
oncoming cars on the blind
side.

U
N
I
T

© Nilam Publication Sdn. Bhd. 184

B6FizikF4(155-187)csy5p.indd 184 09/01/2020 9:56 AM


MODUL • Fizik TINGKATAN 4

Praktis SPM / SPM Practice


Soalan Objektif / Objective Questions
1 Rajah 1 menunjukkan sinar cahaya yang merambat 4 Antara berikut, yang manakah adalah ciri-ciri imej
dari air ke udara. yang terbentuk di atas cermin satah?
Diagram 1 shows light ray travels from water to air. Which of the following are the characteristics of the
image formed on a plane mirror?
A Songsang, sama saiz dengan objek dan nyata.
p Udara Inverted, same size as the object and real.
q Air
B Tegak, sama saiz dengan objek dan nyata.
r Air
s Water Upright, same size as the object and real.
C Tegak, diperbesar dan songsang sisi.
Upright, magnified and laterally inverted.
Rajah 1 / Diagram 1 D Songsang sisi, sama saiz dengan objek dan
Indeks biasan bagi air itu ialah maya.
Refraction index of water is Laterally inverted, same size as the object and
virtual.
sin r sin s
A ——–
sin q C ——–
sin p
5 Rajah 3 menunjukkan suatu objek diletakkan di
sin p sin p
——–
B ——– D hadapan satu kanta cembung. / Diagram 3 shows
sin s sin r
an object placed in front of a convex lens.

2 Imej manakah yang dihasilkan oleh kanta penumpu O


pada skrin?
Which image is obtained by a convex lens on the
screen? 2F F F 2F
A Songsang dan nyata.
Inverted and real.
Rajah 3 / Diagram 3
B Songsang dan maya.
Inverted and virtual. Apakah ciri-ciri imej yang terbentuk?
What are the characteristics of the image formed?
C Tegak dan nyata.
A Nyata, songsang, diperkecil
Upright and real.
Real, inverted, diminished
D Tegak dan maya.
B Nyata, songsang, sama saiz
Upright and virtual.
Real, inverted, same size
C Maya, tegak, diperbesar
3 Rajah 2 menunjukkan sinar cahaya X ditujukan ke
Virtual, upright, magnified
dalam blok kaca. Sudut genting kaca ialah 42°.
Diagram 2 shows a light ray, X, directed into a glass D Nyata, songsang, diperbesar
block. The critical angle of the glass is 42°. Real, inverted, magnified

B D 6 Rajah 4 menunjukkan suatu sinar cahaya merambat


C
dalam satu blok semi bulatan yang lutsinar.
Diagram 4 shows a ray of light propagating in a
transparent semi-circular block.
A Y

Kaca 35°
Glass 48°

X
Rajah 4 / Diagram 4 U
N
Apakah indeks biasan bagi blok lutsinar itu? I
Rajah 2 / Diagram 2 T
What is the refractive index of the transparent block?
Ke arah manakah sinar merambat dari titik Y? A 1.00 C 1.74 6
In which direction does the light move from point Y? B 1.35 D 2.80
185 © Nilam Publication Sdn. Bhd.

B6FizikF4(155-187)csy5p.indd 185 09/01/2020 9:56 AM


MODUL • Fizik TINGKATAN 4

Soalan Struktur / Structure Questions


1 Dalam satu eksperimen dalam makmal, pelajar mengkaji hubungan antara jarak objek, u dan jarak imej, v bagi
kanta cembung. Daripada data yang diperoleh, graf diplot seperti di bawah.
In a laboratory experiment, students investigate the relationship between the object distance, u and the image distance, v
for a convex lens. From the data obtained, a graph is plotted as shown below.
1
(cm–1)
v

0.100

0.075

0.050

0.025

1
0
u (cm )
–1
0.025 0.050 0.075 0.100
Rajah 1 / Diagram 1
1 apabila —1
(a) (i) Nyatakan nilai pintasan di paksi — u = 0 dan, cari nilai panjang fokus kanta.
v
State the value of the intercept at the —1 axis when —
1 = 0 and, find the value of focal length of the lens.
v u
Apabila / When —1 = 0
u
1 = 0.10 cm–1

v
∴ 1 1 1
— = — + —
f u v
1
— = 0.10 cm–1
f
1
f = ———–– = 10.0 cm
0.10 cm–1
1 1
(ii) Namakan kuantiti yang diperoleh dari pintasan di paksi — —
u apabila v = 0.
1 axis when —
Name the quantity obtained from the reciprocal of the intercept at the — 1 = 0.
u v
Panjang fokus / Focal length

(b) Kirakan kecerunan graf, m.


Calculate the gradient of the graph, m.
(0 – 0.10) cm–1
m = —––––––––––
(0.10 – 0) cm–1
–0.10
= —–––
0.10
= –1.0

U
N
I
T

© Nilam Publication Sdn. Bhd. 186

B6FizikF4(155-187)csy5p.indd 186 09/01/2020 9:56 AM


MODUL • Fizik TINGKATAN 4

2 Rajah 2 menunjukkan seorang budak lelaki menggunakan kanta untuk memfokuskan cahaya pada kertas yang
seiras. Selepas beberapa ketika, kertas pada Rajah 2 (a) lebih cepat daripada kertas di Rajah 2 (b).
Diagram 2 show a boy using a lens to focus light on an identical paper. After a while, the paper in Diagram 2 (a) burns
faster than paper in Diagram 2 (b).
Kanta
Lens
Kanta
Lens

Kertas terbakar Kertas terbakar


Paper burned Paper burned
(a) (b)

Rajah 2 / Diagram 2

Diameter kanta Rajah (a) Diameter kanta Rajah (b)


Diameter of lens of Diagram (a) Diameter of lens of Diagram (b)

Bentuk keratan rentas kanta Bentuk keratan rentas kanta


Rajah (a) Rajah (b)
Cross sectional shape of lens Cross sectional shape of lens
of Diagram (a) of Diagram (b)

(a) Banding: / Compare:


(i) Diameter kanta / Diameter of lens
Diameter kanta di kedua-dua rajah adalah sama / Diameter of lens in both diagram are the same.

(ii) Ketebalan kanta / Thickness of lens


Ketebalan kanta di Rajah (a) > Ketebalan kanta di Rajah (b)

Thickness of lens in Diagram (a) > Thickness of lens in Diagram (b)


(iii) Panjang fokus kanta / Focal length of lens
Panjang fokus kanta di Rajah (a) < Panjang fokus kanta di Rajah (b)
Focal length of lens in Diagram (a) < Focal length of lens in Diagram (b)
(iv) Kuasa kanta / Power of lens
Kuasa kanta di Rajah (a) > Kuasa kanta di Rajah (b)
Power of lens in Diagram (a) > Power of lens in Diagram (b)

(b) Hubung kaitkan: / Relate:


(i) Ketebalan kanta dan panjang fokus kanta / Thickness of lens and focal length of lens
Ketebalan kanta semakin bertambah, panjang fokus semakin berkurang.
As the thickness of lens increases, the focal length of lens decreases.
U
(ii) Panjang fokus kanta dan kuasa kanta / Focal length of lens and power of lens N
I
T
Panjang fokus semakin berkurang, kuasa kanta semakin bertambah.
6
As the focal length decreases, the power of lens increases.

187 © Nilam Publication Sdn. Bhd.


B6FizikF4(155-187)csy5p.indd 187 09/01/2020 9:56 AM


FORMULA PENTING / IMPORTANT FORMULA

Maklumat berikut mungkin berfaedah. Simbol-simbol mempunyai makna yang biasa.


The following information may be useful. The symbols have their usual meaning.

v–u 20* Haba / Heat, Q = mcΔθ


1* a =
t
21* Haba / Heat, Q = ml
2* v 2 = u 2 + 2as
22* P1 V1 = P2 V2
1 2
3* s = ut + at
2
P1 P2
23* =
4* Momentum = mv T1 T2

V1 V2
5* F = ma 24* =
T1 T2
6* Daya impuls, mv – mu
F= 25* v = f λ
Impulsive force, t
ax
7* Berat / Weight, W = mg 26* λ =
D
sin i
8 Tenaga kinetik 1 27* n = (n1 = udara / air)
= mv 2 sin r
Kinetic energy 2

9 Tenaga keupayaan graviti dalam nyata / real depth H


= mgh 28* n = =
Gravitational potential energy dalam ketara / apparent depth h

1 1 1
10 Tenaga keupayaan kenyal 1 29* = +
= Fx f u v
Elastic potential energy 2

tenaga / energy 30* Pembesaran linear, hi v


11 Kuasa / Power, P = m= =
masa / time Linear magnification, ho u

Gm1 m2
12* Daya graviti / Gravitational force, F = 31 Q = It
r2

mv2 32 E = VQ
13* Daya memusat / Centripetal force, F =
r
4π2r3 33 V = IR
14* Jisim Bumi / Mass of Earth, m =
GT 2
35 Kuasa / Power, P = IV
15* Laju linear satelit, GM
v= 35 g = 10 m s–2
Linear speed of satellite, r
Ns Vs
36 =
16* Halalu lepas, 2GM  Np Vp
v=
Escape velocity, r
Is Vs
37 Kecekapan / Efficiency = × 100%
Ip Vp
m
17 ρ =
V 38 E = mc 2
F
18 Tekanan / Pressure, P = 39 c = 3.0 × 108 m s –1
A
40 1 u = 1.66 × 10–27 kg
19 Tekanan / Pressure, P = hρg

*Rumus untuk Tingkatan 4 / Formulae for Form 4

Modul Fizik F4(formula)csy3p.indd 1 09/01/2020 8:35 AM

You might also like